ROSH: Internal Med ALL Q's

Lakukan tugas rumah & ujian kamu dengan baik sekarang menggunakan Quizwiz!

A 2-week-old boy presents to the emergency department with his mother. The child was born at 37 weeks gestation, and his mother had sporadic prenatal care. The child presents with fever, lethargy, and vomiting. Vital signs are 101.5°F, pulse 155 beats per minute, respirations 30/min, blood pressure 95/60 mm Hg, and oxygen saturation of 98% on room air. Physical exam reveals a lethargic baby with dry mucous membranes. What cerebrospinal fluid analysis confirms the most likely diagnosis? A20 cm opening pressure, clear appearance, 30 mg/L protein, 75 g/dL glucose, and 15 cells/HPF white blood cells B25 cm opening pressure, fibrin webs visible, 100 mg/L protein, 45 mg/dL glucose, and 100 cells/HPF white blood cells C35 cm opening pressure, turbid appearance, 200 mg/L protein, 35 mg/dL glucose, and 80,000 cells/HPF white blood cells D40 cm opening pressure, clear appearance, 150 mg/L protein, 80 g/dL glucose, and 2,000 cells/HPF white blood cells

35 cm opening pressure, turbid appearance, 200 mg/L protein, 35 mg/dL glucose, and 80,000 cells/HPF white blood cells This patient has bacterial meningitis. The most common pathogens of bacterial meningitis depend on the age of the patient. Group B Streptococcus and Listeria monocytogenes are the most common pathogens in infants less than 1 month old. Neisseria meningitidis and Streptococcus pneumoniae are the most common pathogens from 1 month to 50 years of age. Streptococcus pneumoniae and Listeria monocytogenes are the most common pathogens in patients greater than 50 years old. Patients will present with the classic triad of fever, headache, and altered mental status. They may also report nausea, vomiting, and symptoms of meningeal irritation. The physical exam will reveal fever, altered mental status, Kernig sign (the inability to straighten the knee with hip flexion), and Brudzinski sign (flexion of the neck produces hip and knee flexion). A lumbar puncture is the gold standard for definitive diagnosis. Evaluation of cerebrospinal fluid will reveal elevated polymorphic neutrophils, decreased glucose, increased protein, and elevated cerebrospinal fluid opening pressure. If the patient has focal neurologic findings, papilledema, or a history of other central nervous system diseases, then a CT scan should be considered prior to lumbar puncture to assess for midline shift, which increases the risk of herniation when lumbar puncture is performed. A 20 cm opening pressure, clear appearance, 30 mg/L protein, 75 g/dL glucose, and 15 cells/HPF white blood cells (A) are indicative of viral meningitis. A 25 cm opening pressure, fibrin webs visible, 100 mg/L protein, 45 mg/dL glucose, and 100 cells/HPF white blood cells (B) are indicative of fungal meningitis. A 40 cm opening pressure, clear appearance, 150 mg/L protein, 80 g/dL glucose, and 2,000 cells/HPF white blood cells (D) is not associated with a known pattern of infectious meningitis. Question: What infectious agent of bacterial meningitis are nonblanching petechiae and a purpuric rash associated with? Answer: Neisseria meningitidis.

A 45-year-old man is screened for tuberculosis with a tuberculin skin test. He had recent close contact with an individual with active and contagious tuberculosis. Which of the following induration measurements is the minimum induration for a positive tuberculin skin test in this patient? 10 millimeters 15 millimeters 2 millimeters 5 millimeters

5 mm Tuberculosis is an infectious disease caused by Mycobacterium tuberculosis. Tuberculosis may be clinically active or latent. Latent tuberculosis means that the individual has had prior tuberculosis infection but does not have active tuberculosis disease. In many cases of latent tuberculosis, the infection is contained or "walled off" by host defenses but not cleared. Latent tuberculosis may become active at any point, but this activation is most likely to occur in immunocompromised patients. Risk factors for acquiring tuberculosis include living in or traveling to endemic areas, intravenous drug use, homelessness, and occupational exposures, such as working in health care. The most common clinical manifestations of tuberculosis include systemic and pulmonary symptoms. Pharmacologic treatment is recommended to treat latent tuberculosis. The preferred regimen is four months of rifampin. A result of 10 millimeters (A) is used as the threshold for patients who have an increased risk of reactivation, such as chronic renal failure requiring dialysis, diabetes mellitus, some malignancies (leukemias, lymphomas, carcinoma of the head, neck, or lung), or patients who are underweight; injection drug users; residents or employees in prisons, jails, or health care facilities; and workers and people who stay in homeless shelters. A result of 15 millimeters (B) is incorrect and is only used as a threshold for a positive test in healthy individuals who are at least 4 years old without the above risk factors. A result of 2 millimeters (C) is incorrect. The only time a tuberculin skin test is considered positive at less than 5 millimeters is in patients with HIV infection and a close contact with an active contagious case. Question: Which alternative medication can be used to treat latent tuberculosis? Answer: Isoniazid.

Which of the following patients is at risk for having an atypical presentation of an acute myocardial infarction? 21-year-old man with a history of asthma 37-year-old man with a history of obesity 72-year-old woman with a history of diabetes and hypertension 75-year-old man with a family history of coronary artery disease

72-year-old woman with a history of diabetes and hypertension A 21-year-old man with a history of asthma (A) is less likely to have a myocardial infarction since the patient is a younger age and asthma is not a risk factor for myocardial infarction. A 37-year-old man with a history of obesity (B) is more likely to have a typical presentation in the setting of being young and without any predisposing conditions that would cause an atypical presentation. A 75-year-old man with a family history of coronary artery disease (D) is also more likely to have the typical presentation in the setting of coronary artery disease and being a man. Question: What is the Levine sign? Answer: Clenched fist placed over the heart in the setting of chest pain.

Which of the following patients meets the diagnostic criteria of diabetes mellitus, according to the American Diabetes Association? A patient with a fasting glucose level of 100 mg/dL and an A1C value of 5.5% A patient with polyuria, excessive thirst, and a random glucose level of 240 mg/dL An asymptomatic patient with a fasting glucose level of 136 mg/dL (day 1) and 110 mg/dL (day 2) An asymptomatic patient with a random glucose level of 220 mg/dL

A patient with polyuria, excessive thirst, and a random glucose level of 240 mg/dL Type 2 diabetes is characterized by hyperglycemia that is often due to a progressive loss of insulin secretion from the beta cells superimposed on the background of insulin resistance, leading to relative insulin deficiency. At first presentation, most patients with type 2 diabetes are asymptomatic and only have serum evidence of hyperglycemia. As the disease progresses, the classic symptoms of hyperglycemia (polyuria, polydipsia, nocturia, blurred vision, and weight loss) develop. Rarely, patients with type 2 diabetes present in a hyperosmolar hyperglycemic state, a condition that is characterized by significant hyperglycemia, severe dehydration, and obtundation but without ketoacidosis. DKA is rarely the first presentation of type 2 diabetes but can occur in the setting of severe infection or acute illness and in non-Caucasian ethnic groups. According to the American Diabetes Association (ADA) and the World Health Organization, three diagnostic criteria exist for diabetes: symptomatic hyperglycemia, asymptomatic hyperglycemia, and prediabetes. Diabetes can be easily diagnosed when a patient is symptomatic (has polyuria, polydipsia, thirst, weight loss) and has a random blood glucose of ≥ 200 mg/dL. Both type 1 and type 2 diabetes can be diagnosed this way. In an asymptomatic patient, diabetes (often type 2) can be diagnosed by one of the following criteria: fasting blood glucose ≥ 126 mg/dL on more than one occasion (7.0 mmol/L), two-hour plasma glucose value of ≥ 200 mg/dL (11.1 mmol/L) during a 75 g oral glucose tolerance test (OGTT), or A1C values ≥ 6.5% (48 mmol/mol). In the absence of unequivocal symptomatic hyperglycemia, the diagnosis of diabetes must be confirmed by a repeat test on a subsequent day. The ADA criteria to diagnose prediabetes are as follows: impaired fasting glucose with fasting plasma glucose of 110-125 mg/dL (6.1-6.9 mmol/L), impaired glucose tolerance with two-hour glucose value after a 75 g OGTT of 140-199 mg/dL (7.8-11.0 mmol/L), and A1C values of 5.7 to < 6.5% (39-48 mmol/mol). An asymptomatic patient with a fasting glucose level of 136 mg/dL (day 1) and 110 mg/dL (day 2) (C) has met the diagnostic criteria of prediabetes, which indicates the presence of impaired glucose tolerance with a high risk for developing diabetes mellitus in the future. Lifestyle modification, including regular exercise, diet low in refined sugars, and weight loss, should be strongly encouraged in this patient. Screening for diabetes in this patient should be repeated annually. An asymptomatic patient with a random glucose level of 220 mg/dL (D) should be tested on a subsequent day to either exclude or diagnose diabetes. The diagnosis of diabetes cannot be made after a single random glucose level in an asymptomatic patient. Question: What abnormal genes are often associated with type 1 diabetes? Answer: HLA-DR3-DQ2 haplotype and HLA-DR4 serotype.

A 56-year-old man presents to the office with complaints of anal pain with bowel movements for the past 2 weeks. He reports there is no pain at rest, but he often has residual pain after defecating. He has noted a small amount of bright red blood on the toilet paper after having a bowel movement. His past medical history is significant for a prior work-related back injury, for which he takes hydrocodone with acetaminophen on a daily basis. On physical examination, the finding demonstrated above is noted (open ulcer inferior to anus). A digital rectal exam is not performed due to the patient's significant pain. What is the most likely diagnosis based on the provided information? Anal fissure External hemorrhoid Internal hemorrhoid Perianal abscess

Anal Fissure An anal fissure is characterized as a superficial longitudinal tear at the posterior midline of the anus. Anal fissure is a common cause of anal pain and bleeding. Anal fissures can occur secondary to trauma, constipation, vaginal delivery, and anal intercourse. They can also occur secondary to other diseases, such as Crohn disease, human immunodeficiency virus, and sarcoidosis. Fissures are more common in middle age. The vast majority of anal fissures occur in the posterior midline of the anal canal. Patients may report acute pain at the anus with defecation. The pain associated with anal fissure is typically more severe than pain associated with hemorrhoids. Hematochezia may also be present. While an acute fissure appears similar to a laceration, a chronic fissure has raised edges and may expose the internal anal sphincter muscle fibers. Anal fissures are treated with conservative measures initially. These measures include increasing dietary fiber, sitz baths, topical analgesics for pain, and topical vasodilators. Patients with constipation should be started on a bowel regimen (stool softener). Patients with chronic anal fissures often need surgical intervention. Prevention includes adequate fluid intake, high-fiber diet, and avoidance of straining. External hemorrhoid (B) is incorrect. The pain of an external hemorrhoid is less severe and is observed on physical examination as a swollen vein or lump at the anal verge and perianal area. Internal hemorrhoid (C) is incorrect. Internal hemorrhoids do not present with pain, as they occur above the dentate line. They more commonly present with bleeding. Perianal abscess (D) is incorrect. A perianal abscess may be associated with erythema, tenderness, and swelling on physical examination. Rectal pain is often throbbing in nature and worsens with sitting and activities that promote straining (coughing, defecation). Question: What is the recommended dietary fiber intake? Answer: 20-35 grams per day.

A 61-year-old woman presents to the emergency department with fever and abdominal pain for the past 2 days. She reports mild nausea but no episodes of emesis. She notes a previous history of an "abnormal colonoscopy". On physical examination, she is febrile to 100.8°F and is tender in the left lower quadrant. A mass is palpable in the left lower quadrant. Which of the following diagnostic studies would have the highest sensitivity and specificity for confirming the diagnosis, given her most likely diagnosis and associated complication? Abdominal radiograph Abdominal ultrasound CT scan of the abdomen MRI of the abdomen

CT scan of the abdomen CT scan of the abdomen has a 94% and 99% respective sensitivity and specificity in the diagnosis of acute diverticulitis. Diverticulitis occurs when a diverticulum becomes inflamed or necrotic. Acute diverticulitis is associated with the development of complications, such as abscess formation, fistula, bowel obstruction, and perforation of the bowel. Diverticulitis occurs more frequently in older patients with a median age at diagnosis of 63 years. Clinical manifestations include abdominal pain (particularly involving the left lower quadrant), low-grade fever, nausea, and vomiting. Patients with complicated diverticulitis may have additional signs or symptoms suggesting hemodynamic instability or peritonitis. An abscess may result in a palpable mass in the left lower quadrant. Uncomplicated diverticulitis can be managed in the outpatient setting as long as the patient is not > 70 years of age and does not exhibit sepsis, high fever, significant comorbidities or immunosuppression, or inability to tolerate oral intake. Patients receiving outpatient therapy should be administered oral antibiotics for 7-10 days with close follow-up planned. Preferred antibiotic therapy includes a fluoroquinolone (ciprofloxacin, levofloxacin) with metronidazole. For patients who are admitted to the hospital for management, intravenous antibiotics should be initiated along with adequate fluid resuscitation and pain control. Surgical intervention may be necessary in the setting of acute complications. Patients who are at high risk for serious complications related to diverticulitis or those with recurrent diverticulitis may benefit from elective colectomy. Abdominal radiograph (A) is incorrect. An abdominal radiograph may reveal nonspecific findings associated with complications of diverticulitis, such as an ileus formation or perforation. Air-fluid levels or pneumoperitoneum may be observed. Abdominal ultrasound (B) is incorrect. Ultrasound may demonstrate an abscess, fistula, or nonspecific findings associated with peritonitis. MRI of the abdomen (D) is incorrect. MRI is not readily available and takes longer than CT scan. Further studies are needed to assess the sensitivity and specificity of MRI of the abdomen in the diagnosis of acute diverticulitis compared with CT scan. Question: What are risk factors for the development of diverticulitis? Answer: Cigarette smoking, nonsteroidal anti-inflammatory drugs, aspirin, obesity, excessive alcohol use, red meat ingestion, and a positive family history. Nuts, corn, and popcorn ingestion have been disproven as risk factors for the development of diverticulitis.

A 65-year-old man with a past medical history of chronic kidney disease stage IV presents to the clinic to discuss his lab results. The patient does not report any symptoms or concerns at this time. His BMP results include glucose 140 mg/dL, creatinine 3.86 mg/dL, eGFR 21 mL/min, sodium 142 mmol/L, potassium 7.2 mmol/L, chloride 108 mmol/L, carbon dioxide 21 mmol/L, and calcium 9.7 mmol/L. The patient's ECG is noted above. Which of the following should be administered as the most appropriate next step in this patient's management? Calcium gluconate Insulin Kayexalate Sodium bicarbonate

Calcium Gluconate The patient in the vignette above has acute hyperkalemia as noted in both his labs and ECG findings. Patients with hyperkalemia can be asymptomatic or can present with nonspecific symptoms such as muscle weakness, fatigue, or shortness of breath. Patients with advanced kidney disease are most commonly affected. Some causes of hyperkalemia include potassium supplements or use of certain medications, including potassium-sparing diuretics such as spironolactone, angiotensin-converting enzyme (ACE) inhibitors, angiotensin II receptor blockers (ARBs), and digoxin. Diagnosis of hyperkalemia is based on a serum potassium level greater than 5 mEq/L. An electrocardiogram is not a sensitive test to diagnose hyperkalemia, however, there are significant characteristic findings on an electrocardiogram that can point to the diagnosis. These findings include elevated or peaked T waves, widening of the QRS complex, and PR prolongation. The treatment for hyperkalemia involves multiple steps. The first and most important treatment is to immediately administer calcium gluconate or calcium chloride to stabilize the myocardium and increase the cardiac threshold. The second step is intended to decrease serum potassium by redistributing the potassium into the cells, which can be done by administering insulin, albuterol, sodium bicarbonate, or a combination of these. The final step in management is to remove the potassium from the body by administering loop diuretics or sodium polystyrene. Insulin (B) and sodium bicarbonate (D) can be used as the second step in management for patients with hyperkalemia. Both insulin and sodium bicarbonate facilitate the redistribution of potassium from the serum into the cells. Before redistributing the potassium, it is important to stabilize the patient's cardiac function. Kayexalate (C) is the final step in treatment for hyperkalemia and will decrease the body's total potassium after the heart has been stabilized and the potassium has been redistributed. Question: What are the two most common causes of hypokalemia? Answer: Gastrointestinal losses due to vomiting, diarrhea, or laxative use and renal losses due to use of loop or thiazide diuretics.

A 45-year-old woman presents to a primary care provider with fatigue, generalized joint pain and stiffness, and multiple nodules over the extensor surface of her forearms. A complete laboratory workup is ordered and is significant for elevated levels of anti-cyclic citrullinated peptide and C-reactive protein. She is started on appropriate pharmacologic therapy. Which of the following diagnostic studies will most likely be indicated as her condition progresses, in addition to routine preoperative tests for procedures requiring sedation? Cervical spine radiographs Dual-energy X-ray absorptiometry Pulmonary function testing Schirmer test

Cervical spine radiographs Rheumatoid arthritis (RA) is a chronic autoimmune inflammatory condition, typically affecting women between 30 and 50 years of age. Rheumatoid nodules may be present over bony prominences (e.g., the extensor surface of the forearms), bursae, or tendon sheaths.The anti-cyclic citrullinated peptide (anti-CCP) test is the most specific laboratory blood test for diagnosing RA. Serum rheumatoid factor levels may also be elevated but are not as specific as the anti-cyclic citrullinated peptide test. Since RA is an inflammatory condition, the erythrocyte sedimentation rate and C-reactive protein levels are frequently elevated. Radiography of the affected joints will initially show juxta-articular demineralization and soft tissue swelling. Erosions at the ulnar styloid and juxta-articular margins may occur as the disease progresses. Joint space narrowing may also become more prominent as the disease progresses. Cervical changes may result in atlantoaxial instability or subluxation, necessitating cervical spine radiographs in patients with advanced rheumatoid arthritis who require intubation and sedation. Dual-energy X-ray absorptiometry (B) is used to measure bone density and is useful for diagnosing osteopenia and osteoporosis. A T-score of −1.0 to −2.49 is diagnostic for osteopenia. A T-score of ≤ 2.5 is diagnostic for osteoporosis. While osteoporosis is a common extra-articular manifestation of RA, it will have no effect on intubation and sedation. Pulmonary function testing (C) is useful in the diagnosis, evaluation, and monitoring of respiratory processes such as asthma and chronic obstructive pulmonary disease. While pulmonary manifestations in RA include interstitial fibrosis, pulmonary nodules, and bronchiolitis obliterans, pulmonary function testing would not be indicated unless the patient was having respiratory complications. Schirmer test (D) is used to measure tear production and is useful for diagnosing Sjögren syndrome. Sjögren syndrome may be present in patients with RA, but testing is not necessary in the preoperative setting, as it has no effect on intubation and sedation. Question: What syndrome is characterized by splenomegaly, anemia, neutropenia, thrombocytopenia, and rheumatoid arthritis? Answer: Felty syndrome.

A 30-year-old man presents to a primary care provider with fatigue, decreased appetite, and a diffuse maculopapular rash involving his palms and soles. He reports that he is currently sexually active with both men and women and has had four different partners in the last year. He also reports that he had a painless sore on his penis about two months ago that healed spontaneously after a few weeks. A rapid plasma reagin test is positive. A follow-up fluorescent treponemal antibody absorption test is also positive. Which of the following physical exam findings is most consistent with the patient's diagnosis? Chancre Condyloma acuminata Condylomata lata Gummas

Condylomata lata The patient in the vignette above has secondary syphilis which is characterized by a diffuse maculopapular rash commonly found on the palms and soles, condylomata lata, and systemic signs and symptoms (e.g., fever, headache, sore throat, lymphadenopathy, decreased appetite). Secondary syphilis occurs several weeks to months after a primary syphilis infection. Primary syphilis is characterized by a painless genital ulcer (chancre) that heals spontaneously after three or four weeks. Tertiary syphilis occurs in about 15% of untreated patients and is characterized by small, benign tumors (gummas) that can be found in the brain, bone, skin, liver, or other tissues. Tertiary syphilis may also present with cardiovascular manifestation. Neurosyphilis can occur during any stage. The patient in the vignette above had a chancre (A) two months ago during the primary stage of syphilis. A chancre is a painless penile ulcer and is not typically seen in secondary syphilis. Condyloma acuminata (B) are anogenital warts that occur with human papillomavirus infection and have a cauliflower-like appearance. They are not present in syphilic infections. Gummas (D) are benign lesions that involve the skin, bones, and viscera and are seen in tertiary syphilis. Question: What is the contagious, nonsexually transmitted disease seen in tropical regions caused by Treponema pallidum subspecies pertenue that is characterized by granulomatous lesions? Answer: Yaws.

A 20-year-old man presents to his primary care provider in July with complaints of fever, chills, headache, and muscle aches for four days. The patient recalls removing a tick on his arm several days before his symptoms began and is unsure how long the tick had been there. The patient reports that he has been dog sitting for the past week in North Carolina and noticed that one of the dogs had multiple ticks attached. He brought one of the ticks with him to the office today. Which of the following physical exam findings is most consistent with the suspected diagnosis? Erythematous circular rash with central clearing Facial flushing and conjunctivitis Maculopapular rash on the trunk that spreads to the extremities Tic douloureux

Facial flushing and conjunctivitis The tick mentioned in the vignette above is an adult female American dog tick (Dermacentor variabilis), a vector for the gram-negative coccobacillus Rickettsia rickettsii, which causes Rocky Mountain spotted fever. The symptoms associated with Rocky Mountain spotted fever typically appear 2-14 days after infection and include fever, chills, headache, nausea, vomiting, myalgias, insomnia, and irritability. Facial flushing and conjunctival injection are common signs. The characteristic rash starts as a blanching and maculopapular rash that becomes petechial and begins on the wrists and ankles before spreading to the trunk. It typically appears two to four days after fever. An erythematous circular rash with central clearing (A) is also known as erythema migrans and is pathognomonic for Lyme disease. Lyme disease is caused by the spirochete Borrelia burgdorferi and can be transmitted by the black-legged deer tick (Ixodes scapularis). A maculopapular, sandpaper-like rash on the trunk that spreads to the extremities (C) is characteristic of scarlet fever. In contrast, Rocky Mountain spotted fever starts on the feet and ankles and spreads to the trunk. Scarlet fever is a complication of Streptococcus pyogenes infection. A "strawberry tongue" or Pastia lines (erythematous lines in body creases) may be seen in scarlet fever. Tic douloureux (D) is another name for trigeminal neuralgia and has nothing to do with ticks. The French translation of tic douloureux is "painful muscle spasm." Question: Why is doxycycline typically contraindicated in children? Answer: It can cause tooth discoloration and weakened dental enamel.

A 29-year-old man who has a history of chronic homelessness and IV drug use presents to you for hospital admission after arriving at the emergency room with fevers, chills, and rapid weight loss over the past five days. Vital signs show HR of 82 bpm, BP of 110/65 mm Hg, and temperature of 102.1°F. Physical exam reveals a rash on the patient's palms that is shown above (petechial rash on palmar surface). Which of the following is most likely to confirm the suspected diagnosis? Gram-positive cocci seen in the patient's blood culture Hemorrhagic exudates seen on the patient's retina Hyperpigmented linear lesions under the patient's nails Subcutaneous nodules seen over the patient's distal phalanx

Gram-positive cocci seen in the patient's blood culture Infective endocarditis is an acute or subacute bacterial infection of the endocardium and heart valves. Acute infective endocarditis is most commonly caused by Staphylococcus aureus bacteremia and is most prevalent in patients with a history of IV drug use. Acute Staphylococcus aureus bacteremia causing endocarditis in those who use IV drugs typically affects tricuspid valve. Subacute infective endocarditis is associated with bacteremia caused by the following organisms: HACEK group bacteria, Streptococcus viridans, and Enterococcus species. The Duke criteria are used to diagnose infective endocarditis. It uses known risk factors, signs, symptoms, laboratory data, and diagnostic studies to predict the probability of bacterial endocarditis. Major Duke criteria include blood cultures showing a bacteria that is common in infective endocarditis and evidence of endocardial involvement seen on either echocardiogram or heard as a new heart murmur on physical exam. Minor Duke criteria include predisposition condition (e.g., history of IV drug use), fever, immunologic phenomenon (e.g., Osler nodes, Roth spots, glomerulonephritis, positive rheumatoid factor), vascular phenomena (e.g., Janeway lesions, septic emboli, mycotic aneurysms, intracranial hemorrhage), microbiological evidence of bacteremia from an organism that is not typically associated with infectious endocarditis, and positive echocardiogram not meeting major criteria. The Duke criteria indicate infective endocarditis in the following cases: two major clinical criteria are met, one major and three minor clinical criteria are met, or when five minor criteria are met. In the case of this patient, three minor Duke criteria features are met (i.e., history of IV drug use, fever, and Janeway lesions). Thus, an additional major criterion is needed in order to diagnose infective endocarditis. Gram-positive cocci seen in the patient's blood culture is indicative of Staphylococcus aureus bacteremia and, therefore, an example of a major Duke criterion. Treatment of infective endocarditis is achieved by treating the underlying bacteremia with IV antibiotics. Hemorrhagic exudates seen on the patient's retina (B) (i.e., Roth spots) and subcutaneous nodules seen over the patient's distal phalanx (D) (i.e., Osler nodes) both represent immunologic phenomenon that are possible in infective endocarditis. Addition of any one of the above signs to the patient's presentation would be interpreted as meeting four minor Duke criteria. While this is suggestive of infective endocarditis, it is not diagnostic. Hyperpigmented linear lesions under the patient's nails (C) (i.e., splinter hemorrhages) are a sign of infective endocarditis. However, the presence of splinter hemorrhages is nonspecific for infective endocarditis and, therefore, is not the best answer choice. Question: What is the most common risk factor present in individuals with right-sided endocarditis? Answer: IV drug use.

A 40-year-old woman presents with widespread pain for four months. She also reports sleep disturbance, headaches, and fatigue. Laboratory studies are within normal limits. Which of the following physical exam findings is most likely to be seen in this patient? Gottron papules Musculoskeletal tenderness Sclerodactyly Tophi

Musculoskeletal tenderness Fibromyalgia syndrome is a central pain disorder characterized by chronic, widespread musculoskeletal pain accompanied by fatigue and several tender points. It may occur spontaneously or with rheumatoid arthritis, lupus, or Sjӧgren syndrome. Fibromyalgia more commonly develops in women aged 30 to 55 years. Fibromyalgia is often comorbid with several disorders, including headaches, irritable bowel syndrome, chronic fatigue syndrome, temporomandibular disorder, major depressive disorder, and panic disorder. Common triggers of fibromyalgia include Lyme disease, hypothyroidism, viral infections, and autoimmune disorders. Additionally, there is a correlation between fibromyalgia and emotional or physical trauma in childhood. Some patients with fibromyalgia have low levels of serotonin and norepinephrine, high levels of substance P, abnormalities in imipramine uptake receptor, low levels of growth hormone, and hypofunction of the hypothalamus-pituitary-adrenal axis. Patients typically present with nonarticular musculoskeletal aches, pain, fatigue, sleep disturbance, and multiple tender "trigger" points. Physical exam shows no signs of muscle inflammation or weakness, although the patient often reports musculoskeletal tenderness when pressure is applied at more than 11 of the 18 tender points. Diagnosis of fibromyalgia is made by recognizing the typical pattern of pain and other symptoms as well as by excluding underlying disorders such as hypothyroidism, hepatitis C, and vitamin D deficiency. The only drug that is FDA-approved for treating fibromyalgia is pregabalin. Gottron papules (A) are flat, erythematous papules and plaques that are commonly seen on the fingers. They are pathognomonic for dermatomyositis, an idiopathic inflammatory myopathy characterized by the presence of proximal muscle weakness and cutaneous manifestations such as heliotrope rash. Sclerodactyly (C), a part of the CREST syndrome (calcinosis, Raynaud phenomenon, esophageal dysmotility, sclerodactyly, and telangiectasia), is an abnormal, chronic hardening and tightening of the skin and connective tissues of the hands and toes. It is associated with limited cutaneous scleroderma. Tophi (D) are chalky deposits of urate crystals that are seen in chronic gout. They are not seen in patients with fibromyalgia. Question: What is the correct amount of pressure that is applied to tender points in a patient with fibromyalgia? Answer: 4 kg/cm, which can be quantified by pressing with a fingertip until the fingertip blanches.

A 44-year-old woman presents to the emergency department with chest pain, nausea, and diaphoresis. She reports multiple episodes of early morning chest pain for the past three weeks. She has a 20 pack-year smoking history and reports cocaine use in the last month. The initial ECG obtained shows ST elevations in the inferior leads. Serial troponins and CK-MB are negative. No stenotic lesions are seen on coronary angiography. Which of the following medications would be inappropriate in the management of the suspected diagnosis? Amlodipine Isosorbide dinitrate Lisinopril Propranolol

Propranolol Prinzmetal (or variant) angina is a rare form of angina that occurs due to coronary vasospasm and often occurs late at night or early in the morning. This condition most frequently occurs in women under 50 years of age. Chest pain symptoms may be identical to the chest pain experienced during a myocardial infarction. Risk factors for Prinzmetal angina include hypertension, smoking, cocaine use, diabetes, obesity, emotional stress, and certain medications (e.g., beta-blockers and triptan medications). Vasospasm at rest with preservation of exercise capacity is associated with Prinzmetal angina. The right main coronary artery (associated with the inferior ECG leads) is most commonly affected in Prinzmetal angina. Transient ST elevations during angina with return to baseline upon resolution of symptoms is characteristic. Cardiac biomarkers will be normal. Beta-blockers (e.g., propranolol, nadolol, sotalol, timolol) exacerbate coronary vasospasm and should be avoided in patients with Prinzmetal angina in the absence of coronary stenosis. Prinzmetal angina is responsive to amlodipine (A), a calcium channel blocker that reduces cardiac muscle oxygen demand. Isosorbide dinitrate (B) is a vasodilator and relieves angina caused by coronary vasospasm. Lisinopril (C) is an angiotensin-converting enzyme inhibitor that is associated with improved cardiovascular outcomes in patients with Prinzmetal angina. Question: How long does it take for elevated CK-MB levels to return to normal? Answer: 48-72 hours.

A 57-year-old man with a past medical history of recent right-sided hip replacement presents to the emergency department with sudden-onset chest pain, shortness of breath, cough, diaphoresis, and hemoptysis. Vitals show heart rate of 110 beats per min, blood pressure of 147/81 mm Hg, respiratory rate of 22 breaths per min, and O2 saturation of 97% on room air. Physical exam shows clear lung sounds in all fields and a new systolic murmur heard over the second intercostal space on the left side of the sternum. Which of the following is the diagnostic test of choice to confirm the diagnosis? AChest X-ray BComputed tomography angiogram of the chest CD-dimer DVentilation-perfusion scan

Pulmonary embolism is most commonly caused by a thrombus or embolus that obstructs flow in the pulmonary circulation. The most common cause of pulmonary embolism is a lower extremity deep vein thrombosis. Other causes are air emboli from central lines, amniotic fluid from active labor, and fat from long bone fractures or surgery. Risk factors for pulmonary embolism are a hypercoagulable state, venous stasis, vascular intimal inflammation or injury, recent surgery, cancer, oral contraceptives, trauma, and pregnancy. Symptoms may include pleuritic chest pain, dyspnea, cough, hemoptysis, and diaphoresis. Signs on evaluation are tachycardia, tachypnea, crackles, new-onset murmur, and low-grade fever. Gold standard diagnosis is made using CT angiogram of the chest to evaluate for filling defects. Treatment includes anticoagulation, most commonly with heparin in the acute setting, and thrombolytics in patients who are hemodynamically unstable. A vena cava filter can be placed in patients who are at high risk for recurrent pulmonary embolism and cannot tolerate anticoagulation. A chest X-ray (A) is nonspecific for a pulmonary embolism and should only be used to rule out other causes of symptoms such as pneumonia. Rarely seen X-ray findings suggestive of pulmonary embolism include a Hampton hump (a pleural-based wedge-shaped defect caused by infarcted lung parenchyma) and Westermark sign (paucity of vascular markings distal to the pulmonary embolism). A D-dimer (C) may be used in low-risk patients to rule out a pulmonary embolism, however, if it is positive, it requires additional testing to confirm the diagnosis. A ventilation-perfusion scan (D) is reserved for patients who cannot tolerate or have contraindications to a CT angiogram. Question: What is the Virchow triad? Answer: Hypercoagulable state, venous stasis, and vascular intimal inflammation or injury.

A 25-year-old man of African descent develops acute hemolytic anemia and hemoglobinuria after being treated with primaquine as a prophylaxis for malaria. On exam, he is noted to have generalized jaundice, dark urine, and pallor. Heinz bodies are seen on a peripheral blood smear. Which of the following is the most likely cause of his symptoms? Absence of decay-accelerating factor Antibody-mediated destruction of red cells Defective red cell membrane-tethering proteins Reduction in glutathione levels in red cells

Reduction in glutathione levels in red cells Glucose-6-phosphate dehydrogenase deficiency (G6PD) is an X-linked recessive disorder that is characterized by a reduction in the half-life of G6PD, thereby rendering red cells susceptible to oxidative stress. Reduced glutathione (activated) levels are restored by glutathione reductase, which requires the presence of NADPH generated by G6PD. Hence, a decrease in the half-life of G6PD leads to a decrease in NADPH levels, resulting in the reduction of glutathione levels in red cells and increased susceptibility to oxidative stress and eventual intravascular hemolysis. During a hemolytic episode, elevations in indirect bilirubin, LDH, and reticulocytes may be present. Peripheral blood smear demonstrates Heinz bodies and bite cells. The diagnosis can be confirmed by an enzyme assay, which will be low between hemolytic episodes. Absence of decay-accelerating factor (A) (DAF) is seen in patients with paroxysmal nocturnal hemoglobinuria, an acquired defect in myeloid stem cells resulting in absent glycosylphosphatidylinositol (GPI), thereby rendering red cells susceptible to destruction by complements. DAF inhibits C3 convertase and protects red cells against complement-mediated damage. DAF is securely anchored to the red cell membrane by GPI. Hence, the absence of GPI leads to the absence of DAF, which in turn causes episodic intravascular hemolysis at night during sleep. Antibody-mediated destruction of red cells (B) results in immune hemolytic anemia, characterized by IgG- or IgM-induced extravascular hemolysis. IgG inappropriately binds red cells in relatively warm temperatures of the central body (warm agglutinin), causing the consumption of the antibody-coated red cells by splenic macrophages, resulting in the formation of spherocytes. Immune hemolytic anemia is associated with lupus, chronic lymphocytic leukemia, and certain drugs, such as penicillin and cephalosporins. Defective red cell membrane-tethering proteins (C) results in hereditary spherocytosis, an inherited defect in cytoskeleton-membrane tethering proteins that maintain the biconcave shape of red blood cells. The loss of membrane proteins renders red cells spherical instead of biconcave. The defect most commonly involves ankyrin, spectrin, and band 3. Question: True or false: African variant of G6PD is more severe than the Mediterranean variant. Answer: False.

A 77-year-old man presents to a primary care provider with fever, malaise, and a pruritic rash for three days. The patient reports that he cared for his grandchildren two weeks ago and that they had similar symptoms that did not seem as severe. The patient states that the rash began on his head and body and then spread to his arms and legs. Physical examination reveals the presence of a scattered, erythematous maculopapular rash with a centripetal pattern. Sporadic vesicles, as seen in the image above, are present. A few of the vesicular lesions are crusted over. The patient states that he and his children are against vaccinations. What is the causative agent of the most likely diagnosis? Human herpesvirus-6 Rubella virus Rubeola virus Varicella zoster virus

Varicella zoster virus The varicella-zoster virus (VZV) is the causative agent of chickenpox (varicella) and shingles (herpes zoster). Varicella is characterized by a pruritic rash that follows a centripetal pattern. Lesions typically begin on the scalp, face, and trunk before spreading to the extremities. The initial lesions are maculopapular before erupting into superficial vesicles that have a "dewdrop on a rose petal" appearance. These vesicles eventually crust over. Lesions in multiple stages and morphologies are typically present on the skin. Fever and malaise are more pronounced in adults compared to children. Human herpesvirus-6 (A) is the causative agent of roseola infantum. It is mostly seen in children six months to three years of age. It is characterized by the appearance of a pink maculopapular rash after the fever resolves. The lesions in the image above are inconsistent with the roseola infantum rash. The rubella virus (B) causes German measles and is characterized by a fine maculopapular rash that progresses from the face to the trunk to the extremities. Posterior cervical and postauricular lymphadenopathy is common. Vesicular lesions are not seen with German measles. The rubeola virus (C) causes measles and is characterized by conjunctivitis, coryza, and cough. The measles rash begins on the face and behind the ears before spreading to the trunk and extremities. The palms and soles may be affected. Koplik spots are pathognomonic for measles. Question: The Hutchinson sign may be seen in what conditions? Answer: Ophthalmic herpes zoster (vesicles on the nose), intracranial mass (unresponsive, enlarged pupil), and subungual melanoma (longitudinal melanonychia).

A 50-year-old man presents to his primary care provider with complaints of fatigue, night sweats, weight loss, and bone pain. Laboratory testing is significant for a normochromic, normocytic anemia. The peripheral blood smear is shown in the image above. A comprehensive metabolic panel is significant for a blood urea nitrogen of 50 mg/dL, a creatinine of 3.5 mg/dL, and a total protein of 14.7 g/dL. Bence Jones proteins are present in the urine. Which of the following pathologic fractures is most commonly associated with the patient's diagnosis? Acetabular fracture Humeral neck fracture Humeral shaft fracture Vertebral fracture

Vertebral Fracture Multiple myeloma, also known as plasma cell myeloma, is a primary malignant bone tumor that occurs due to the hyperproliferation of plasma cells, which produce monoclonal immunoglobulins. It is slightly more common in men and typically occurs between 45-74 years of age. Patients will typically present with common cancer symptoms: fatigue, weakness, weight loss, low-grade fever, and night sweats. Like other bone cancers, bone pain, pathological fractures, radiculopathy, and paresthesias may occur. The most common pathological fractures associated with multiple myeloma are vertebral, pelvic, femoral neck, and rib fractures. These fractures are lytic in nature. Several characteristic diagnostic findings are associated with multiple myeloma. Rouleaux formation (as shown above) is commonly seen on peripheral smear. Serum protein electrophoresis will show a monoclonal protein spike ("church spike"). Bence Jones proteins (immunoglobulin light chains) in the urine are commonly seen with multiple myeloma but may also be seen with Waldenstrom macroglobulinemia. Radiographs of the skull may show "punched-out" lesions. Bone radiographs are useful in the initial detection of lytic lesions, which are mostly seen in the axial skeleton. MRI and PET scans may also be used and are more sensitive than bone radiographs. The CRAB acronym (elevated calcium, renal insufficiency, anemia, lytic bone lesions) is commonly used for multiple myeloma. Other findings associated with multiple myeloma include elevated blood urea nitrogen and creatinine levels, hypogammaglobulinemia, elevated serum total protein, > 10% bone marrow clonal plasma cells, and spinal cord compression. The typical initial treatment includes an immunomodulatory agent (e.g., lenalidomide), a proteasome inhibitor (e.g., bortezomib, carfilzomib), and high-dose dexamethasone. Question: What facial bone is associated with necrosis resulting from long-term bisphosphonate use? Answer: Jaw.

A 24-year-old woman presents to the office for intermittent dull abdominal pain that occurs about 20 minutes after eating certain meals. She occasionally has nausea but no fever, vomiting, or sweating. Her symptoms last for about two hours after onset and do not occur after every meal. Her diet is high in carbohydrates and fat. Which of the following is the best initial diagnostic study of choice for her condition? Abdominal ultrasound Computed tomography of the abdomen Magnetic resonance imaging of the abdomen Plain abdominal radiograph

Abdominal US Cholelithiasis is a condition where there are stones found in the biliary tract, particularly in the gallbladder. Most gallstones are composed of cholesterol. Risk factors of this condition include female sex, European or American Indian ancestry, increased age, obesity, and pregnancy. Gallstone disease can be both asymptomatic and symptomatic. Symptomatic patients may have nonspecific symptoms such as indigestion, belching, upset stomach, and bloating. Others may present with biliary colic, which is characterized by episodic epigastric or right upper quadrant pain radiating to the back. The pain typically begins after eating (typically after a fatty meal), is described as both intense and dull, then slowly wanes over the course of a few hours. There may also be associated nausea, vomiting, and sweating. The pain may also be unremitting and not relieved with taking antacids or having a bowel movement. Ultrasonography is the initial imaging modality of choice in disease involving the gallbladder or biliary tract. However, it is not as sensitive as computed tomography (CT) scans for detecting stones that may be located in the common bile duct, but CT exposes the patient to high levels of radiation. Magnetic resonance imaging (MRI) with magnetic resonance cholangiopancreatography (MRCP) is excellent at identifying stones in the biliary tract and is typically reserved for suspected choledocholithiasis. It is often recommended as the second-line imaging study of choice if ultrasonography does not provide a clear diagnosis. Endoscopic retrograde cholangiopancreatography (ERCP) and percutaneous transhepatic cholangiography are diagnostic as well as therapeutic, but both are invasive, require contrast, and have a high risk of complications. If the stones are relatively small and purely composed of cholesterol, ursodiol is a medication that may help to dissolve them over a period of many months (6 to 18 months of treatment). If this is not effective, definitive surgical removal of the gallbladder may be required in patients who have large gallstones and those who have evidence of a calcified gallbladder. Computed tomography (B) and magnetic resonance imaging of the abdomen (C) are good choices if there are suspected gallstones in the common bile duct. However, both are also more expensive and have more risks involved than ultrasonography (e.g., radiation exposure with CT scans). Plain abdominal radiographs (D) can be used to rule out other causes of abdominal pain, such as intestinal obstruction, but are not used to diagnose gallstone disease. Question: What are patients who have a calcified gallbladder at risk for? Answer: Gallbladder carcinoma.

A 10-year-old boy presents to the pediatrics clinic for evaluation of his asthma. He is currently only taking a rescue inhaler as needed. He has daytime symptoms four days per week, nighttime symptoms once per month, and uses a rescue inhaler four days per week. Which of the following medication regimens is most appropriate for this patient? Albuterol Albuterol and fluticasone Albuterol, fluticasone, and salmeterol Albuterol, fluticasone, salmeterol, and a short course of oral prednisone

Albuterol and fluticasone Asthma is classified according to severity to help guide treatment and monitor symptoms. The classifications include intermittent asthma and mild, moderate, or severe persistent asthma. Albuterol (A) is a short-acting beta-agonist bronchodilator. Albuterol is used as needed to treat intermittent asthma. The patient in this vignette has mild persistent asthma because he has symptoms four days per week. Albuterol, fluticasone, and salmeterol (C) is incorrect. This regimen includes a short-acting beta-agonist, inhaled corticosteroid, and long-acting beta-agonist. This regimen would be appropriate if the patient had moderate persistent asthma. Albuterol, fluticasone, salmeterol, and a short course of oral prednisone (D) is incorrect. Prednisone is sometimes indicated in severe persistent asthma and is often used in the treatment of acute asthma exacerbations. Question: What are the measurement criteria used to define reversible obstruction on spirometry? Answer: FEV1 that increases 12% and 200 cc after bronchodilation.

A 61-year-old woman presents to a primary care provider for routine management of hypertension, hyperlipidemia, and diabetes mellitus. Her vital signs are significant for a BP of 148/96 mm Hg. Her most recent hemoglobin A1C was 7.6%. While at the office, the patient suddenly exhibits slurred speech, left-sided facial weakness, and right arm weakness. The patient is immediately taken to the emergency department. While en route, the patient's symptoms resolve. A stroke protocol is initiated upon arrival to the ED. A noncontrast CT of the head is shown above. The patient's complete blood count, comprehensive metabolic panel, coagulation profile, and urinalysis are normal. Serial cardiac enzymes and ECGs are normal. The physical exam is significant for a left-sided carotid bruit. No focal neurologic deficits are present. What is the best next step in the management of this patient? Administer recombinant tissue plasminogen activator Admit and order a diffusion-weighted MRI Discharge on aspirin with outpatient evaluation within 72 hours Emergent left carotid endarterectomy

Admit and order a diffusion-weighted MRI A diffusion-weighted MRI is better than standard CT and MRI tests for evaluating ischemic damage and subsequent stroke risk after a TIA. The American Stroke Association recommends the use of the ABCD2 score to predict future stroke risk and determine the need for hospitalization. The score is composed of the following components: age ≥ 60 years (1 point); systolic BP ≥ 140 mm Hg or diastolic BP ≥ 90 mm Hg (1 point); clinical features, including unilateral weakness with or without speech impairment (2 points) or speech impairment without unilateral weakness (1 point); duration of TIA ≥ 60 minutes (2 points), 10-59 minutes (1 point), or less than 10 minutes (0 points); and diabetes mellitus (1 point). Patients presenting within 72 hours of TIA should be admitted if their ABCD2 score is greater than or equal to 4, if their ABCD2 score is less than 4 and there is uncertainty that outpatient evaluation can be completed within 48-72 hours, or if their ABCD2 score is less than 3 and there is evidence of cerebral infarction. There is no need to administer recombinant tissue plasminogen activator (A) since the patient in the vignette above does not have an ischemic stroke. Thrombolytic therapy is indicated within three hours of ischemic stroke symptoms if the patient has no contraindications. Aspirin should be avoided for 24 hours after thrombolytic therapy. If the patient had an ABCD2 score of less than 3, no evidence of cerebral infarction, and reliable follow-up, it would be appropriate to discharge on aspirin with outpatient evaluation within 72 hours (C). An emergent left carotid endarterectomy (D) is not indicated unless the patient is symptomatic and a carotid ultrasound demonstrates a stenosis > 70%. Question: True or false: the anterior circulation carries 80% of cerebral blood flow and is the location of 80% of transient ischemic attacks and strokes. Answer: True.

A 45-year-old man presents to a primary care provider with fever, fatigue, worsening dyspnea, cough, and right-sided pleuritic chest pain for one week. Physical examination is significant for diminished breath sounds over the right lower lobe and dullness to percussion over the same area. A chest radiograph is ordered and shows a right-sided 15 mm pleural effusion over 35% of the hemithorax. He is transferred to the ED where a thoracentesis is performed. Pleural fluid analysis reveals a pleural fluid protein to serum protein ratio of 0.7 and a pleural fluid LDH to serum LDH ratio of 0.8. The pleural fluid is free-flowing and has a pH of 7.4. Gram stain and culture of the pleural fluid are negative.Which of the following is also indicated in the treatment of this patient? Antibiotics and supportive treatment Tube thoracostomy Tube thoracostomy and intrapleural fibrinolytic Tube thoracostomy with intrapleural fibrinolytic and deoxyribonuclease

Antibiotics and supportive treatment The patient in the vignette above has an uncomplicated parapneumonic effusion secondary to a bacterial pneumonia. After administration of antibiotics, no further intervention is indicated. Parapneumonic pleural effusions are exudates that accompany bacterial pneumonias. A thoracocentesis is both therapeutic and diagnostic. Light criteria confirm an exudative effusion if one of the following three criteria are met: pleural fluid protein to serum protein ratio > 0.5; pleural fluid LDH to serum LDH greater than 0.6; pleural fluid LDH greater than two-thirds the upper limit of normal serum LDH. t. Category 1 effusions are free-flowing with a thickness of < 10 mm on radiographs. Category 2 effusions are free-flowing with a thickness of > 10 mm and cover < 50% of the hemithorax. Gram stain and culture of category 2 effusions are typically negative, and the pH is ≥ 7.2. Category 1 and 2 effusions are considered uncomplicated parapneumonic effusions and are treated with antibiotics. Tube thoracostomy is not necessary, and the prognosis is good. Category 3 effusions cover > 50% of the hemithorax, are loculated, or have a thickened parietal pleura. Gram stain and culture may be positive, the pH of the effusion is typically < 7.2, and the glucose level of the effusion is typically < 60 mg/dL. Category 3 effusions are considered complicated parapneumonic effusions and have a poor prognosis. Category 4 effusions are defined as empyemas and involve frank pus in the pleural space. They have the worst prognosis. Category 3 and 4 effusions require tube thoracostomy in addition to antibiotic treatment. Intrapleural fibrinolytics (e.g., tissue plasminogen activator) and deoxyribonuclease have been shown to improve clinical outcomes. A tube thoracostomy (B) is indicated for complicated parapneumonic effusions and empyemas to reduce the risk of permanent pulmonary impairment secondary to fibrous encasement of the lung. A tube thoracostomy and intrapleural fibrinolytic (C) have not been proven in clinical trials to improve the drainage of an empyema or a complicated parapneumonic effusion. However, tube thoracostomy with intrapleural fibrinolytic and deoxyribonuclease (D) has been shown to improve clinical outcomes (e.g., improved drainage, decreased length of hospital stay, decreased surgical referrals) compared to placebo or either agent alone. Question: What is the role of deoxyribonuclease in complicated parapneumonic effusions? Answer: Degrades biofilm formation within the pleural cavity by catalyzing extracellular DNA.

A 35-year-old man presents to the clinic complaining of a retrosternal burning sensation after eating, with an occasional acidic taste in his mouth. Which of the following lifestyle modifications should be recommended for this patient? Avoid caffeine, chocolate, and carbonated beverages Avoid eating a meal six hours before bedtime Avoid foods containing tomatoes, onions, and garlic Sleep in a prone position

Avoid caffeine, chocolate, and carbonated beverages Gastroesophageal reflux disease describes reentry of stomach contents into the distal esophagus, causing symptoms of retrosternal burning, postprandial (after lunch/dinner) chest discomfort, and acidic taste in the hypopharynx or mouth. Chronic cough or hoarseness can also result from chronic gastroesophageal reflux. Common causes of gastroesophageal reflux disease include gastroesophageal junction incompetence, impaired esophageal acid clearance, and impaired defenses against esophageal injury. Risk factors for the development of gastroesophageal reflux disease include obesity, pregnancy, exogenous estrogen treatment, and hiatal hernia. Certain dietary habits also contribute to gastroesophageal reflux disease, such as ingestion of foods high in caffeine or carbonation and eating immediately before lying down. Question: What is Barrett esophagus? Answer: Metaplastic epithelium displaces normal squamous epithelium of the distal esophagus and can lead to adenocarcinoma.

A 7-year-old boy is brought into the clinic for a cough. His mother states that she has noticed the cough has gotten more frequent over the past few months and is worse at night. The cough is dry. She has also noticed occasional wheezing and coughing when he has participated in baseball practice. He denies fever, rhinorrhea, headache, chest pain, or shortness of breath but does state that occasionally after practicing sports, his chest feels "tight." Which of the following is the most appropriate diagnostic test to order? Chest radiograph Complete blood count IgE antibody tests Pulmonary function studies

Asthma is characterized by intermittent cough, wheezing, and shortness of breath that is reversible with bronchodilating medications. The pathophysiology behind asthma is bronchial hyperresponsiveness. This causes inflammation of the airways and increased mucus production. Patients typically have an exaggerated response to a trigger. Triggers include allergens, infections, exercise, and inhalants. Exposure to these triggers causes wheezing and cough. Patients often complain of the cough being worse at night. Shortness of breath or difficulty breathing can occur with acute asthma exacerbations. Symptoms are usually relieved with avoidance of the triggers, time, or the use of bronchodilator medications. Occasionally, patients will complain of chest tightness or a heavy sensation in the middle of the chest, as in the vignette. Patients with asthma typically do not complain of sharp chest pain. The diagnosis is confirmed with pulmonary function studies that show a decreased forced expiratory volume in one second (FEV1) to forced vital capacity (FVC) ratio (FEV1/FVC). An obstructive pattern is present when the ratio is below 70% of the predicted normal value. A bronchodilator is then administered to see if the patient has a response. An FVC improvement of greater than 12% after bronchodilator administration is indicative of asthma. Chest radiographs (A) are only indicated when evaluating with new-onset symptoms in a patient over the age of 40 or individuals with symptoms that are not typical for asthma. These symptoms include chest pain, fever, weight loss, night sweats, hemoptysis, or clubbing of the fingers. A complete blood count (B) might be useful to look at the differential of white blood cells and examine the level of eosinophils if allergies are thought to be playing a role, however, this is not the best test, as it is nonspecific and may be normal in patients with asthma. IgE antibody tests (C) are used to diagnose specific allergies and, therefore, would not be useful as the most appropriate next step in diagnosing asthma in this scenario. Question: What stimuli can be used for bronchoprovocation testing? Answer: Inhaled methacholine, inhaled mannitol, exercise, or hyperventilation of dry air.

What surgical procedure is an alternative to ciprofloxacin to eradicate the carrier state for carriers of Salmonella? Billroth I Cholecystectomy Splenectomy Whipple procedure

Cholecystectomy Salmonella is a gram-negative rod of the Enterobacteriaceae family that can cause typhoid (enteric) fever, acute enterocolitis, and bacteremia. Salmonella causes infection via fecal-oral transmission and is most commonly acquired through contaminated food (e.g., chicken, eggs, beef, fruit) or drink. Salmonella enterica serovar Typhi can colonize the gallbladder, resulting in an asymptomatic carrier state. Asymptomatic carriers can continuously spread Salmonella bacteria. High-dose ciprofloxacin or ampicillin may be used to treat carriers in the absence of gallstones but is not a definitive treatment. Cholecystectomy has a higher cure rate and is the preferred treatment in the presence of gallstones but is still not 100% effective at eliminating the carrier state. A distal gastrectomy (antrectomy) is a procedure in which the antrum of the stomach is excised and is used to treat gastric cancers, gastric perforations, severe gastric ulcers, or complicated Helicobacter pylori infections. An antrectomy is further classified by the anastomoses to reestablish gastrointestinal continuity. A Billroth I (A) is a gastroduodenostomy fashioned in either an end-to-end or end-to-side manner. A Billroth II is a gastrojejunostomy. A splenectomy (C) is most commonly indicated for a ruptured spleen. Other indications include symptomatic splenomegaly, blood disorders refractory to other treatments, cancer (e.g., chronic lymphocytic leukemia, Hodgkin lymphoma, hairy cell leukemia), and splenic cysts or tumors. A Whipple procedure (D), also known as a pancreaticoduodenectomy, is most commonly indicated for the treatment of pancreatic cancer. It is also indicated for other pancreatic, intestinal, and biliary disorders. Gastrectomy, splenectomy, and Whipple procedures are ineffective for eradicating Salmonella carrier states because the gallbladder is the primary colonization site. Question: What bacteria should patients with pet turtles be aware of? Answer: Salmonella is commonly found on reptiles such as turtles, lizards, and snakes.

A 40-year-old woman presents to her primary care provider with right leg pain that has been persistent for the past seven months. The patient has a history of a right tibial shaft stress fracture that was diagnosed two years ago. The patient reports sensations of burning, tingling, and severe muscle aches over the site of the previous fracture. She reports her right leg seems to be a different color and is painful to move at times. On physical examination, the right leg is pale and dusky in appearance. The patient has pain out of proportion to light touch over the anterior tibialis muscle. Ankle dorsiflexion is limited due to pain. AP and lateral radiographs of the right tibia are normal. What is the most likely diagnosis? Arterial ulcer Chronic osteomyelitis Complex regional pain syndrome Exertional compartment syndrome

Complex regional pain syndrome The patient in the vignette above has complex regional pain syndrome. Complex regional pain syndrome is a diagnosis of exclusion characterized by localized pain of more than six months' duration. The pain is typically unilateral, severe, and out of proportion to any physical examination or diagnostic findings. A previous history of a fracture, sprain or strain, soft tissue injury, or surgery is common. Patients may present with sensations of burning, tingling, or myalgias. Complex regional pain syndrome is more common in women and has a peak incidence at 40 years old. The Budapest criteria are used to diagnose complex regional pain syndrome after other causes (e.g., fracture, compartment syndrome, infection, cancer) have been ruled out. The Budapest criteria require the presence of at least one symptom in three of the following categories and one sign in two of the following categories: sensory, vasomotor, edema or sudomotor, and motor or trophic. Sensory signs and symptoms include hyperesthesia or allodynia. Vasomotor signs and symptoms include changes in skin color, temperature, or symmetry. Edema or sudomotor signs and symptoms include changes in edema or sweating. Motor or trophic signs and symptoms include decreased motor function, decreased range of motion, or trophic changes. The treatment of complex regional pain syndrome includes a combination of psychological therapy, physical therapy, and pharmacologic treatments (e.g., nonsteroidal anti-inflammatory drugs, gabapentin, corticosteroids, antidepressants, bisphosphonates, topical lidocaine, capsaicin cream). Referral to a pain clinic may be necessary for patients who do not respond to initial treatments. An arterial ulcer (A) results from ischemia to an area of skin (typically on the lower extremities) and is more common in patients with a history of diabetes, smoking, hyperlipidemia, or hypertension. The affected area may appear pale, blue, or shiny and feel cool to the touch. A loss of hair and faint or absent pulses is consistent with an arterial ulcer. Physical examination would reveal the presence of an ulcer. Chronic osteomyelitis (B) occurs when bacteria in necrotic tissue causes a recurrence of infection after the original infection has been treated with antibiotics or surgery. Radiographs would show signs of cortical destruction and periosteal reaction. The radiograph in the vignette above is normal. Exertional compartment syndrome (D) typically presents with pain, swelling, tightness, weakness, or numbness that progressively worsens with activity and improves with rest. It is typically bilateral. Pale and dusky appearance of the skin is not typically seen in exertional compartment syndrome. The patient in the vignette above has had persistent, unilateral pain that is more indicative of complex regional pain syndrome. Question: What is the difference between sequestrum and involucrum? Answer: Sequestrum is dead bone, while involucrum is newly formed bone.

A 32-year-old man presents to the clinic for treatment of newly diagnosed, asymptomatic HIV infection. His viral load is 200,000 copies/mL, and his CD4 cell count is 110 cells/microL. He is currently taking no medications and is allergic to sulfa drugs (generalized urticaria). A chest X-ray reveals no active pulmonary disease, and tuberculosis skin and sputum tests are negative. Which of the following antimicrobials should be prescribed prophylactically for this patient in addition to antiretroviral medication? Azithromycin 1,200 mg once weekly Dapsone 100 mg daily Rifabutin 300 mg daily Trimethoprim-sulfamethoxazole DS daily

Dapsone 100 mg daily When CD4 cell counts fall under 200 cells/microL in a patient with HIV infection, prophylactic treatment to prevent pneumocystis pneumonia should be administered. First-line prophylaxis is with double-strength trimethoprim-sulfamethoxazole once daily. Prophylactic treatment should continue until CD4 cell counts are over 250 cells/microL or between 100 and 200 cells/microL in a patient who has had an undetectable viral load for six months or more. In patients who have a sulfa allergy, dapsone 100 mg daily is the prophylactic treatment of choice. Azithromycin 1,200 mg once weekly (A) has been the prophylactic regimen of choice for HIV-positive patients with CD4 cell counts under 50 cells/microL. However, newer research shows very few cases of Mycobacterium avium complex in patients being treated with antiretroviral therapy. Therefore, current recommendations call for prophylaxis against Mycobacterium avium complex only in patients with CD4 cell counts under 50 cells/microL who are unable or unwilling to begin prompt antiretroviral therapy. Once antiretroviral therapy has begun, prophylactic azithromycin should be discontinued. Rifabutin 300 mg daily (C) is an alternative choice of prophylaxis against Mycobacterium avium complex for patients who are intolerant of or allergic to macrolide antibiotics. Like azithromycin, rifabutin should only be used as prophylaxis for Mycobacterium avium complex in HIV-positive patients with CD4 cell counts under 50 cells/microL who are unable or unwilling to begin prompt antiretroviral treatment, and it should be discontinued once antiretrovirals are begun. All patients being considered for receipt of prophylaxis for Mycobacterium avium complex should be tested for active infection before prophylaxis is administered. Trimethoprim-sulfamethoxazole DS daily (D) is the preferred prophylactic regimen for prevention of Pneumocystis pneumonia in HIV-positive patients with CD4 cell counts under 200 cells/microL. The patient in the above vignette, however, is allergic to sulfa drugs and should not be prescribed trimethoprim-sulfamethoxazole. If a patient has a mild, localized dermatologic allergic reaction to sulfa drugs, gradual desensitization to trimethoprim-sulfamethoxazole can be attempted, however, this should not be attempted in patients with generalized or severe hypersensitivity to sulfa. Question: What is the prophylactic treatment for Cryptococcus in HIV-infected individuals? Answer: None is recommended due to treatment toxicity and cost. Symptomatic patients with positive serologic testing for Cryptococcus can be treated with nitazoxanide.

A 55-year-old man presents to a primary care provider for concerns about erectile dysfunction. He reports that he has had difficulty maintaining an erection for several months. He is currently on medication for depression, anxiety, benign prostatic hyperplasia, and angina. Which of the following medications is the most likely cause of his erectile dysfunction? Options: Duloxetine, Nitroglycerin, Prazosin, Selegiline

Duloxetine. Antidepressant medications such as selective serotonin reuptake inhibitors (e.g., fluoxetine, paroxetine, sertraline), serotonin and norepinephrine reuptake inhibitors (e.g., duloxetine, venlafaxine), tricyclics (e.g., amitriptyline, nortriptyline), and older monoamine oxidase inhibitors (e.g., isocarboxazid, phenelzine) have the potential to cause erectile dysfunction. Other classes of medications that may cause erectile dysfunction include diuretics, certain beta-blockers, and opioids.

A 23-year-old man presents to your clinic with a two-day history of worsening parasternal chest pain. The patient reports the pain is sharp and made worse with breathing and lying down. Additional symptoms include a nonproductive cough, which the patient states started several days before the onset of his chest pain. Vital signs are within normal limits. Cardiopulmonary exam is unremarkable. You obtain an ECG and decide to treat this patient with high-dose aspirin. Which of the following ECG findings is most indicative of the suspected diagnosis? Diffuse ST segment elevations Normal sinus rhythm Sinus tachycardia T wave inversions

Diffuse ST segment elevations Acute pericarditis occurs when the pericardial sac becomes inflamed. The most common reason for this in the United States is inflammation of the pericardial sac after an acute viral illness such as an upper respiratory tract infection. Other etiologies include uremia, an autoimmune inflammatory process, cancer, early-onset myocardial infarction, and postmyocardial infarction pericarditis (Dressler pericarditis). Classically, a patient will present with pleuritic sharp chest pain that is made worse by lying down and improved with sitting up or leaning forward. Vital signs are often normal in mild disease, however, a fever may be present. A pericardial friction rub is a sensitive and specific physical exam finding in patients with acute pericarditis, but it is not always present in this disease. In the first hours to days of acute pericarditis, an ECG will often reveal diffuse ST segment elevations and PR segment depressions. One week after disease onset, the ECG will usually become isoelectronic, after which diffuse T wave inversions may develop. Complete resolution of abnormal ECG findings ordinarily occurs after two to six weeks. Echocardiogram showing a pericardial effusion supports the diagnosis of acute pericarditis. However, a pericardial effusion is often absent in patients with acute pericarditis. NSAIDs or colchicine are typically used in the treatment of acute pericarditis. Normal sinus rhythm (B) is common in patients with musculoskeletal or gastrointestinal etiologies of chest pain. Patients with musculoskeletal chest wall pain usually present with chest pain that is tender to palpation and positional. Chest pain that is described as burning or gnawing and is made worse with food suggests a gastrointestinal etiology. Sinus tachycardia (C) is the most common ECG finding in patients with pulmonary embolism. Patients with pulmonary embolism often present with signs and symptoms of deep vein thrombosis, pleuritic chest pain, and cough. T wave inversions (D) are a common finding in patients with acute pericarditis, however, T wave inversions usually develop one to two weeks after disease onset. Question: What are the most common viral etiologies associated with acute pericarditis? Answer: Coxsackievirus, echovirus, and cytomegalovirus.

A 62-year-old obese woman with diabetes presents to the office complaining of worsening dyspnea and dizziness. She has a longstanding history of alcohol use and currently reports decreasing her alcohol consumption to 12 beers per week. On physical exam, she appears to be in no acute distress. Cardiovascular exam reveals jugular venous distention and rales bilaterally on auscultation of the lungs. Which initial diagnostic study is most helpful for eliminating a valvulopathy as the possible cause of her symptoms? Cardiac magnetic resonance imaging Chest radiography Echocardiography Electrocardiography

Echocardiogram Dilated cardiomyopathy is a broad category of conditions in which the heart muscle enlarges, causing left ventricular systolic dysfunction defined as an ejection fraction of less than 50%. There is no wall hypertrophy. Dilated cardiomyopathy is the third most common cause of heart failure and the most frequent reason for heart transplantation. Cardiac MRI (A) allows for detailed examination of myocardial anatomy, function, and perfusion. It may also help identify different pathologies. However, in instances of suspected dilated cardiomyopathy, cardiac MRI is not the preferred initial diagnostic imaging modality. Chest radiography (B) may allow for visualization of the heart valves but is not preferred over echocardiography. Chest radiography is an excellent way to visualize the heart borders and identify cardiomegaly. It also provides an excellent view of the lungs. Electrocardiography (D) is not the preferred diagnostic modality for determining a valvulopathy as the cause for dilated cardiomyopathy. Electrocardiography is helpful, however, in determining if there are any underlying dysrhythmias contributing to dilated cardiomyopathy. Question: What law describes how stretching of the cardiac muscle produces a greater amount of cardiac output force (which is an aspect of compensation of low cardiac output)? Answer: The Frank-Starling Law.

A 26-year-old man who uses intravenous drugs presents with an acute onset of fever, chills, malaise, and anorexia. On physical exam, his temperature is 101.4°F, BP 98/50 mm Hg, HR 120 beats per minute, and RR 22 breaths per minute. Splinter hemorrhage and Osler nodes are present. Which of the following diagnostic tests would confirm the suspected diagnosis? Chest radiography Computed tomography Echocardiography Electrocardiogram

Echocardiography Welcome Chelsy Gentry! Question 60 (of 155) Highlight Strike Erase Flag Normal LabValues «Previous Next» Your Answer is Incorrect A 26-year-old man who uses intravenous drugs presents with an acute onset of fever, chills, malaise, and anorexia. On physical exam, his temperature is 101.4°F, BP 98/50 mm Hg, HR 120 beats per minute, and RR 22 breaths per minute. Splinter hemorrhage and Osler nodes are present. Which of the following diagnostic tests would confirm the suspected diagnosis? AChest radiography BComputed tomographyYour Answer CEchocardiographyCorrect Answer DElectrocardiogram Correct Answer ( C ) Explanation: Acute bacterial endocarditis refers to infection of the endocardial surface of the heart that may involve one or more heart valves or an intracardiac device. Risk factors include previous bacterial endocarditis, preexisting valvular disease, intravenous drug use, an indwelling intravenous catheter, immunosuppression, and recent dental or surgical procedure. It is most commonly caused by Streptococcus viridans, Staphylococcus aureus, and enterococci. In those who use intravenous drugs, S. aureus is the most common cause, and the tricuspid valve is most frequently affected. Prosthetic valve endocarditis is often caused by S. aureus, gram-positive organisms, or fungi if the disease develops during the first two months after implantation. In later disease, streptococci or staphylococci are predominantly causative. Acute onset of fever is the most common presenting symptom and is often accompanied by chills, anorexia, and weight loss. Other symptoms include malaise, headache, myalgia, arthralgia, night sweats, abdominal pain, and dyspnea. Cardiac murmur, splenomegaly, and cutaneous manifestations (petechiae and splinter hemorrhage) may be seen on physical exam. Relatively uncommon clinical manifestations that are highly suggestive of bacterial endocarditis include Janeway lesions (painless red lesions on the palms and soles), Osler nodes (painful, violaceous raised lesions of the fingers, toes, and feet), and Roth spots (exudative lesions of the retina). The diagnosis of bacterial endocarditis is established based on clinical diagnosis, blood cultures, and echocardiography. The acceptable diagnostic criteria for bacterial endocarditis are the modified Duke criteria. A patient must have two major criteria, one major and three minor criteria, or five minor criteria for the diagnosis of bacterial endocarditis to be made. Major criteria include two positive blood cultures of a typical causative microorganism and echocardiographic evidence of endocardial involvement, i.e., new valvular regurgitation. Minor criteria include predisposing factor (intravenous drug use), fever > 100.4°F, vascular phenomena (embolic disease or pulmonary infarction), immunologic phenomena (glomerulonephritis, Osler node, Roth spots), and positive blood culture not meeting major criteria. Empiric antimicrobial therapy should include coverage for staphylococci, streptococci, and enterococci. Gentamicin plus ceftriaxone or vancomycin is an appropriate initial therapy for patients with suspected bacterial endocarditis. Chest radiograph (A) does not confirm the diagnosis of bacterial endocarditis but may reveal underlying pulmonary infiltrates, congestive heart failure, the presence of septic pulmonary emboli, and alternative causes of fever and systemic symptoms. Computed tomography (B) of the chest, abdomen, and pelvis is useful to evaluate sites of metastatic infection such as splenic infarct, renal infarcts, or psoas abscess that may require localized drainage. Electrocardiogram (D) has no specific diagnostic feature but may show the presence of a heart block or conduction delay, providing evidence of paravalvular extension of infection to the valve annulus and adjacent septum. Additionally, ST elevation or depression may suggest the presence of emboli of the coronary circulation. Question: Which antibiotics can be used for preventing endocarditis prior to dental procedures in a patient who is allergic to penicillin? Answer: Azithromycin, clarithromycin, cephalexin, or clindamycin.

A 68-year-old man presents with a substernal chest tightness that occurred while he was walking his dog this morning. The pain lasted approximately five minutes and was relieved by rest. Physical examination and laboratory studies are unremarkable. Resting ECG is normal. Which of the following is the most likely next step in diagnosis? Exercise ECG Myocardial perfusion scintigraphy Radionuclide angiography Stress echocardiography

Exercise ECG Stable angina pectoris is precordial chest pain typically precipitated by stress or exertion and rapidly relieved by rest or nitrates. It is often due to atherosclerotic heart disease but can also be caused by coronary vasospasm, congenital anomalies, emboli, arteritis, or dissection. Additionally, angina can occur in the absence of coronary artery obstruction as a result of severe myocardial cardiomyopathy, severe aortic stenosis or regurgitation, hyperthyroidism, or severe anemia. Patients present with chest discomfort that most commonly occurs during activity and is relieved by rest or nitrate. Exercise ECG is the most commonly used noninvasive procedure for evaluating inducible ischemia in patients with stable angina. Exercise ECG can be combined with imaging (nuclear or echocardiography). Question: What is the most common exercise protocol used in exercise ECG? Answer: The Bruce protocol, which increases the treadmill speed and elevation every three minutes until limited by symptoms.

A 72-year-old woman presents to the office complaining of worsening aching, throbbing, and heaviness in both legs. She also reports frequent night cramps in her legs and feeling like she cannot get comfortable while falling asleep. On physical exam, both lower extremities appear edematous with scattered patches of reddish-brown hyperpigmentation. On her distal lower legs bilaterally, there is a fibrosing panniculitis with hyperpigmentation and induration involving most of the leg circumferentially. Her left medial ankle is edematous and has a 7 mm shallow ulcer with a red base and irregular borders. Doppler ultrasound reveals a reflux of 0.9 seconds in the left great saphenous vein and 0.7 seconds in the right. Considering this information, what would be the most effective clinical intervention for this patient? Endovenous laser ablation Sclerotherapy Surgical vein stripping Venous reconstruction

Endovenous laser ablation Chronic venous insufficiency is a disease of variable severity caused by venous hypertension and incompetent valves leading to dilated veins, discoloration, swelling, pain, skin changes, and ulceration. Chronic venous insufficiency affects 2.5 million people in the United States. Chronic venous insufficiency involves both superficial and deep venous systems. In order for venous return to function appropriately, a series of bicuspid valves within the veins and muscle pumps in the foot, calf, and thigh must prevent backflow toward the feet. Chronic venous insufficiency results from an interruption of this blood flow within the system. The interruption can be caused by a number of factors, including valvular reflux, thrombotic obstruction of the valves, or a combination of the two. As a result, there is an increase in inflammation that leads to endothelial damage. Over time, this inflammation may result in dilated and tortuous capillaries, increased edema, decreased oxygenation to surrounding tissue, stasis changes of the overlying skin, thromboses, and ulceration. Risk factors for chronic venous insufficiency include prolonged standing, pregnancy, prior deep vein thrombosis, obesity, advancing age, and previous leg injury. The most common physical exam findings in patients with chronic venous insufficiency are edema, pain, overlying skin changes, and leg heaviness. Initial conservative measures are recommended for most patients with chronic venous disease. Leg elevation, exercise, and compression therapy will improve oxygen transport to the skin and subcutaneous tissues, decrease edema, reduce inflammation, and compress dilated veins. Currently, this technique has been replaced by percutaneous endovenous thermal ablation. This procedure is performed on an ambulatory basis with local anesthesia. Patients are typically completely ambulatory following treatment, and the recovery time is short. Two types of thermal ablation procedures exist: endovenous laser ablation and radiofrequency ablation. Significant reflux, defined as reflux greater than 0.5 seconds in deep veins (such as the great saphenous vein described in the above vignette), documented on Doppler ultrasound examination is an indication for endovenous laser ablation. Chronic venous insufficiency is associated with significant morbidity and mortality and may indicate the presence of other serious vascular conditions. One serious complication of chronic venous insufficiency is venous ulcers. Due to the low rate of healing and high rate of recurrence, venous ulcers can be debilitating and cause a significant decrease in quality of life. For telangiectasias and reticular veins that are asymptomatic but cosmetically bothersome, sclerotherapy (B) or laser therapy may be effective. The results of sclerotherapy are generally thought to be superior to those of laser therapy. Patients with isolated varicose veins without truncal reflux are typically treated with sclerotherapy or excision depending on the size, location, and number of veins involved. Surgical vein stripping (C), also known as high ligation and stripping, has been largely replaced by endovenous laser ablation due to the ease and relatively low-risk profile of the latter procedure. Venous reconstruction (D) is typically reserved for patients with severe vascular damage, particularly after a deep vein thrombosis. It is not necessary in this case. Question: What are two common chemical solutions used for vein sclerotherapy? Answer: Sodium tetradecyl sulfate and morrhuate sodium.

A 40-year-old man presents to his primary care provider with unilateral scrotal discomfort. He describes the discomfort as dull, heavy, and achy. He is up-to-date on all of his immunizations. He has been in a monogamous relationship for the past 15 years and is not currently using condoms. His family history is significant for a father who was diagnosed with leukemia at 65 years of age. Physical examination is significant for swelling and tenderness along the posterosuperior portion of the right testicle. Elevation of the right testicle relieves the pain. Urinalysis is significant for pyuria and bacteriuria. Urine culture is positive for an organism that grows readily on eosin methylene blue agar and has a metallic green sheen. Nucleic acid amplification testing is pending. What is the most likely diagnosis? Epididymitis due to Chlamydia trachomatis Epididymitis due to Escherichia coli Leukemia-induced epididymitis Viral epididymitis

Epididymitis due to Escherichia coli Epididymitis is an inflammation of the epididymis that often presents as dull, heavy, aching unilateral scrotal discomfort. Epididymitis is typically caused by an infectious disease, though trauma and autoimmune diseases may also cause it. Physical examination will reveal swelling, induration, or marked tenderness over the epididymis or entire scrotum. The Prehn sign (relief of pain with elevation of the scrotum) is associated with epididymitis. A clinical diagnosis can be made from the history and physical examination. Urinalysis may reveal pyuria and bacteriuria, and a urine culture can confirm certain bacterial sources such as N. gonorrhoeae and E. coli. E. coli characteristically grows on eosin methylene blue agar and has a metallic green sheen. Epididymitis due to Chlamydia trachomatis (A) is most common in patients < 35 years of age who are sexually active. Urethral discharge or crusting may be present. C. trachomatis will not grow on urine culture. Leukemia-induced epididymitis (C) has not been reported, however, leukemia patients are at higher risk of viral and mycotic epididymitis because of their immunocompromised state. Viral epididymitis (D) is the most common cause of epididymitis in prepubescent boys and may be caused by mumps, coxsackievirus, echovirus, or adenovirus. Viral and mycotic epididymitis are rarely seen except in immunocompromised patients. Question: What is the treatment of pelvic inflammatory disease in a nonpregnant patient? Answer: Ceftriaxone and doxycycline.

A 64-year-old man presents to the ED for worsening dyspnea on exertion and is admitted for a COPD exacerbation. His medical history is significant for coronary artery disease, hypertension, hyperlipidemia, diabetes mellitus, and obstructive sleep apnea. He has a 40 pack-year smoking history and a history of cocaine use. He was discharged from the hospital nine days ago after being admitted for a myocardial infarction. During the first night of his current admission, the patient begins experiencing chest pain. Vital signs are T 102°F (38.9°C), BP 132/78 mm Hg, HR 105 bpm, RR 22 breaths per minute, and pulse oximetry 96% on 2 L via nasal cannula. His WBC count is 14.7 × 109/L. His rhythm strip is shown above. Which of the following characteristics most likely indicates a diagnosis of postmyocardial infarction syndrome rather than an acute myocardial infarction? Dyspnea Fever and leukocytosis ST elevation in the anterior leads Tachycardia

Fever and leukocytosis Dressler syndrome (also known as postmyocardial infarction syndrome) is a pericarditis usually seen one to two weeks after an acute myocardial infarction or cardiac procedure. Patients must be evaluated carefully to distinguish Dressler syndrome from acute myocardial infarction. Dressler syndrome is most likely due to an autoimmune inflammatory response following transmural myocardial necrosis and may be recurrent even when treated. Typical signs and symptoms associated with Dressler syndrome include pleuritic chest pain relieved when leaning forward, a pericardial friction rub, and tachycardia. Fever and leukocytosis are associated with the inflammatory response seen in Dressler syndrome. Dyspnea (A) is incorrect because it may be seen in both Dressler syndrome and myocardial infarction. ST elevation in the anterior leads (C) is more indicative of an inferior wall myocardial infarction and is commonly caused by thrombosis of the left anterior descending artery. Tachycardia (D) may be seen in both Dressler syndrome and acute myocardial infarction. Question: What is the point between the QRS complex and the ST segment called? Answer: The J point.

A 65-year-old woman presents to a primary care provider with complaints of jaw claudication, throat pain, and scalp tenderness. She also reports soreness in her neck, shoulders, and pelvic girdle. She denies vision loss. Physical examination reveals a pulsatile temporal artery. Her C-reactive protein is elevated. A temporal artery biopsy is obtained. What is the best next step in the management of this patient? High-dose prednisone and low-dose aspirin IV methylprednisolone IV methylprednisolone and high-dose aspirin Treatment with corticosteroids if temporal artery biopsy is abnormal

Giant cell arteritis, also known as temporal arteritis, is a systemic, inflammatory syndrome that affects the medium and large vessels, particularly the temporal artery. It is most common in patients > 50 years of age. Presenting symptoms include a tender, inflamed, or pulsatile temporal artery; scalp tenderness; jaw claudication; throat pain; amaurosis fugax; diplopia; fatigue; fever; and symptoms of polymyalgia rheumatica. Polymyalgia rheumatica is a condition that frequently coexists with giant cell arteritis and is characterized by neck, shoulder, torso, or pelvic girdle pain and stiffness. Erythrocyte sedimentation rate and C-reactive protein are elevated in patients with giant cell arteritis. CBC will reveal a normochromic, normocytic anemia and thrombocytosis. A temporal artery biopsy is used for definitive diagnosis and should be performed promptly because untreated giant cell arteritis can result in blindness. Treatment is high-dose corticosteroid therapy (e.g., prednisone 40-60 mg/day) and should be administered immediately in all patients with suspected giant cell arteritis even before biopsy results are finalized. Patients with vision loss should receive IV methylprednisolone. Low-dose aspirin is also recommended to decrease the risk of cerebrovascular accidents and vision loss. Symptoms resolve within three days of starting corticosteroid therapy. High-dose corticosteroids will also treat coexisting polymyalgia rheumatica. Polymyalgia rheumatica in the absence of giant cell arteritis is treated with low-dose corticosteroids (e.g., prednisone 10-20 mg/day). IV methylprednisolone (B) is not necessary in this patient because she does not have vision loss. IV methylprednisolone and high-dose aspirin (C) is incorrect because low-dose aspirin and high-dose prednisone are sufficient for decreasing the risk of cerebrovascular accidents and vision loss. Treatment with corticosteroids if temporal artery biopsy is abnormal (D) is incorrect. Patients should be started on high-dose corticosteroids without waiting for biopsy results if giant cell arteritis is suspected. Question: What laboratory test is helpful in differentiating polymyalgia rheumatica from polymyositis? Answer: Creatine kinase, which will be normal in polymyalgia rheumatica and elevated in polymyositis.

A 52-year-old man presents to a primary care provider for severe joint pain in his great toe. He reports an acute onset of pain yesterday and denies any injury or trauma. He has had several similar presentations intermittently throughout the past two decades. Vital signs are T 39.1°C, BP 122/72 mm Hg, HR 86 bpm, RR 18 breaths per minute, and pulse oximetry 99% on room air. Physical examination reveals a great toe that is swollen and erythematous and exquisitely tender and warm to the touch. His WBC count is 14,600/mcL, and his uric acid level is 5.9 mg/dL. A radiograph of his toes is shown above. Which of the following is the most likely diagnosis? Gouty arthritis Pseudorheumatoid arthritis Rheumatoid arthritis Septic arthritis

Gouty Arthritis Gouty arthritis, or gout, is a crystal deposition disease caused by uric acid crystal accumulation in synovial fluid. It is most common in men and postmenopausal women. Acute gout is usually monoarticular and most often affects the metatarsophalangeal (MTP) joint of the great toe. Swelling, exquisite tenderness over the affected joint, and erythema are typical signs of gout. The feet, ankles, knees, and hands may also be affected. The underlying pathophysiology of gout is due to altered purine metabolism, resulting in increased uric acid production or decreased renal excretion of uric acid. Polyarticular involvement is more common in chronic gout. In patients with chronic gout who have had repeated flare-ups, the chalky deposits of uric acid crystals in the soft tissue of the affected joints are known as tophi. Definitive diagnosis is made by identification of negatively birefringent needle-shaped uric acid crystals in synovial fluid. Plain radiographs taken during a flare-up may show punched-out lesions ("rat bite" erosions), periarticular tophi (soft tissue masses), or intraosseous lesions. The WBC count is often elevated during a gout attack. The patient in the vignette above is likely taking urate-lowering medications, which can result in normal uric acid levels even during gout flare-ups. Treatment for acute gout flare-ups includes nonsteroidal anti-inflammatory drugs (NSAIDs), colchicine, and intra-articular corticosteroids. Allopurinol should not be started in a patient with an acute gout attack because acute changes in serum uric acid levels can exacerbate gout symptoms. Pharmacologic management includes avoiding certain medications such as thiazide diuretics and aspirin. Xanthine oxidase inhibitors (e.g., allopurinol and febuxostat) prevent excess formation of uric acid and are frequently used in gout prophylaxis. Uricosuric agents (e.g., probenecid) increase urinary excretion of uric acid. Pegloticase is an intravenous medication administered every two weeks and may be used in patients with refractory chronic tophaceous gout. Septic arthritis (D) has an acute onset and frequently involves a single joint (mostly the knee, hip, shoulder, or wrist). It is most common in young, sexually active adults. The presentation of septic arthritis is very similar to gouty arthritis, and patients will often present with fever. Septic joints are typically exquisitely tender, erythematous, warm, and edematous. Radiographs will only show soft tissue swelling and will not show the characteristic lesions seen with gout. Given the patient's history of recurrent episodes and his polyarticular presentation, gouty arthritis is the most likely diagnosis. Question: The chronic ingestion of what substance causes saturnine gout? Answer: Lead.

A 50-year-old man presents to a primary care provider for his annual examination and review of bloodwork. His vital signs are T 37.1°C, BP 150/96 mm Hg, waist circumference 41 inches, body mass index 39 kg/m2, HR 70 bpm, RR 18 breaths per minute, and pulse oximetry 99% on room air. Which of the following additional findings would give this patient a diagnosis of metabolic syndrome? Fasting blood sugar level of 90 mg/dL HDL level of 35 mg/dL LDL level of 150 mg/dL Triglyceride level of 120 mg/dL

HDL level of 35 mg/dL Metabolic syndrome, also known as syndrome X or insulin resistance syndrome, is diagnosed by the presence of at least three of the following criteria: a waist circumference ≥ 35 inches for women or ≥ 40 inches for men, a triglyceride level ≥ 150 mg/dL or on medication for hypertriglyceridemia, an HDL level < 50 mg/dL for women or < 40 mg/dL for men, a BP ≥ 130/85 mm Hg or on antihypertensive medication, and a fasting blood sugar level ≥ 100 mg/dL or on medication to treat hyperglycemia. Question: What syndrome is an autosomal recessive neurodegenerative disorder seen in children with diabetes insipidus, diabetes mellitus, optic atrophy, and deafness? Answer: Wolfram syndrome, the components of which can be remembered by the mnemonic DIDMOAD.

A 64-year-old man is diagnosed with pneumonia and admitted into the hospital. Which of the following findings supports Legionella as the pathogen causing pneumonia? Hypoglycemia Hyponatremia Infiltrate in the upper lobes Pleuritic chest pain

Hyponatremia. There are no findings on history, exam, or radiographic imaging that are specific enough to distinguish Legionella pneumonia from other causes of pneumonia. However, gastrointestinal symptoms, such as nausea, vomiting, and diarrhea, are more common in Legionella pneumonia. In patients who have laboratory testing obtained, hyponatremia supports the diagnosis of Legionella pneumonia.

A 33-year-old woman from Texas presents to a primary care provider with pain and decreased vision in her right eye, difficulty walking, and tremors in both hands. She reports that she has had multiple similar episodes in the past two years that seem to occur whenever the weather gets really hot. During her previous episode last summer, she felt very tired, experienced urinary urgency, and had a feeling of an electrical shock radiating down her neck and back into her extremities whenever she bent her head forward. Her symptoms typically resolve spontaneously after a few days, but they seem to be lasting longer now. Her mother had similar symptoms that were treated with injections of glatiramer acetate. Which of the following is the best test for confirming the suspected diagnosis? Brain biopsy Computed tomography Lumbar puncture Magnetic resonance imaging

MRI Multiple sclerosis is a disorder of the central nervous system that results from demyelination of the white matter in the brain and spinal cord, degeneration of axons and neurons, and astrocytic sclerosis. The most common age group affected is between 20-50 years old, and women are affected more than men. An increase in distance from the equator is associated with an increased risk of multiple sclerosis. Relapsing-remitting pattern. The Uhthoff phenomenon is a worsening of symptoms with high temperatures and is associated with multiple sclerosis. Symptoms typically associated with high temperatures include fatigue, concentration problems, urinary urgency, and Lhermitte sign (an electrical sensation from the neck that radiates down the spine and into the extremities that occurs with neck flexion). (MRI) is the single most useful test for diagnosing multiple sclerosis. White matter plaques are most commonly seen in the ventricles as perpendicular projections (Dawson fingers) or ovoid lesions. Lesions in the dorsal column of the cervical spine are also commonly seen. A brain biopsy (A) is the only definitive method for differentiating subtypes of multiple sclerosis but is not used for an initial diagnosis because it is an invasive test. A brain biopsy may be indicated in atypical presentations in which it is difficult to differentiate multiple sclerosis from other neurological disorders. Computed tomography (B) is less sensitive than MRI for detecting lesions and cerebral atrophy and is no longer included in the workup of multiple sclerosis. While lumbar puncture (C) may be used as an adjunct to MRI to detect the presence of oligoclonal bands, immunoglobulin G index, and myelin basic protein, MRI is far more specific than CSF analysis in the diagnosis and monitoring of multiple sclerosis. Question: What are the components of Charcot neurological triad that are associated with multiple sclerosis? Answer: Nystagmus, intention tremor, and scanning speech.

A 62-year-old man presents to a primary care provider with exertional weakness in his shoulders and thighs that improves with rest. He also complains of ptosis, double vision, and difficulty chewing his food. Physical exam reveals 2+ reflexes in the upper and lower extremities. Two-point discrimination and pinprick sensation are intact. Laboratory testing is positive for acetylcholine receptor antibodies and negative for muscle-specific tyrosine kinase antibodies. His CT scan is shown above. What is the most likely diagnosis? Amyotrophic lateral sclerosis Lambert-Eaton syndrome Multiple sclerosis Myasthenia gravis

Myasthenia gravis Myasthenia gravis is a chronic autoimmune neuromuscular disorder caused by antibodies against acetylcholine receptors and muscle-specific tyrosine kinase. It is characterized by proximal muscle weakness (e.g., shoulders, thighs) and fatigability that improves with rest. Common initial symptoms include ptosis, diplopia, blurred vision, difficulty in chewing or swallowing, and respiratory muscle weakness. Sensation and reflexes are normal. A relapsing-remitting pattern is common. Young women who are HLA-DR3 positive and older men with a thymoma (as seen on the CT scan above) are at an increased risk of myasthenia gravis. A diagnosis of myasthenia gravis may be made with a positive edrophonium (Tensilon) challenge. Edrophonium is a short-acting anticholinesterase that will transiently improve muscle weakness symptoms in patients with myasthenia gravis. Cholinesterase inhibitors (e.g., neostigmine, pyridostigmine) are the mainstay of treatment. Myasthenic crisis is the most serious complication of myasthenia gravis and is characterized by extreme weakness of the respiratory muscles. Amyotrophic lateral sclerosis (A) is a progressive neurodegenerative disease that affects the central nervous system and results in muscle weakness. Unlike myasthenia gravis, symptoms do not improve with rest. Acetylcholine receptor and muscle-specific tyrosine kinase antibodies are not associated with amyotrophic lateral sclerosis. Lambert-Eaton syndrome (B) is an autoimmune neuromuscular condition associated with small cell lung cancer that affects the proximal muscles. Unlike myasthenia gravis, it is characterized by morning weakness that improves with muscle use and typically spares the extraocular muscles. Multiple sclerosis (C) is a disorder of the central nervous system resulting from autoimmune demyelination. It is associated with HLA-DR2-positive patients. Like myasthenia gravis, multiple sclerosis typically follows a relapsing-remitting pattern and presents with ocular symptoms including diplopia. However, multiple sclerosis is more likely to present with sensory loss (e.g., paresthesias), speech dysfunction, and bladder dysfunction. Multiple sclerosis is not associated with thymoma or anti-acetylcholine antibodies. Question: What is Lhermitte sign? Answer: An electrical sensation down the back of the neck to the spine, trunk, and extremities occurring with neck flexion that is associated with multiple sclerosis.

A 62-year-old woman with a history of hypertension presents to the emergency department for new-onset headache and blurry vision that started one hour ago. Vital signs are BP 220/120 mm Hg, HR 100 bpm, and RR 16 breaths per minute. Physical exam is significant for flame hemorrhages on funduscopy. Lab work reveals anemia, thrombocytopenia, and an acutely elevated serum creatinine. The patient is admitted to the ICU for blood pressure stabilization. Which of the following is the most appropriate pharmacologic therapy? Captopril Clonidine Nicardipine Nifedipine

Nicardipine Acute severe hypertension (defined as a blood pressure greater than 180/110-120 mm Hg) has two different clinical presentations: hypertensive emergency and hypertensive urgency. The most common risk factor for acute severe hypertension is noncompliance with an established antihypertensive regimen. Other possible precipitants of acute severe hypertension include drugs (e.g., NSAIDs, cocaine, corticosteroids), anxiety, stroke, heart attack, preeclampsia or eclampsia, pheochromocytoma crisis, or iatrogenesis (e.g., stopping antihypertensives and IV normal saline infusions). Hypertensive urgency is defined as blood pressure of greater than 180/110-120 mm Hg with no signs of acute target organ damage. Patients with hypertensive urgency may present with symptoms such as headache, atypical chest pain, epistaxis, and dizziness. Hypertensive emergency is defined as blood pressure greater than 180/110-120 mm Hg with evidence of acute target organ damage. Acute target organ damage includes hemorrhagic or ischemic stroke, acute coronary syndrome, aortic dissection, diffuse microvascular injury (i.e., malignant hypertension), and hypertensive encephalopathy. The presence of anemia, thrombocytopenia, acute kidney injury, or new-onset retinopathy is indicative of acute microvascular disease, which is the case with the patient in the vignette above. Nondihydropyridine calcium channel blockers (e.g., nicardipine and clevidipine) and labetalol have been shown to be the most effective at lowering blood pressure in most clinical scenarios. Captopril (A) and clonidine (B) are oral antihypertensive agents used in the treatment of hypertensive urgency and are not first-line treatment in hypertensive emergency. Nifedipine (D) is a dihydropyridine calcium channel blocker and is a not recommended first-line medication in hypertensive emergency or urgency. Question: What are the first-line antihypertensive agents in patients with acute aortic dissection? Answer: Esmolol or labetalol.

A 66-year-old woman presents with a sensation of pulsation in her neck and abdomen. The patient reports that she has also had progressively worsening dyspnea on exertion and peripheral edema that began two months ago. She had a pacemaker placement nine months ago for a chronic bifascicular block. Physical exam is significant for distended, pulsatile neck veins, hepatomegaly, and 1+ generalized pitting edema. Palpation of the liver results in increased distension of the neck veins. Which of the following findings on physical exam would most likely correlate with the patient's condition? Harsh midsystolic crescendo-decrescendo murmur radiating to the left shoulder Loud midsystolic murmur best heard with the patient sitting and leaning forward Pansystolic murmur that becomes louder with inspiration Pansystolic murmur with prolonged apical impulse

Pansystolic murmur that becomes louder with inspiration Tricuspid regurgitation is a valvular disorder that occurs when there is retrograde blood flow from the right ventricle to the right atrium during systole. The underlying pathophysiology is a right-sided pressure overload leading to right-sided heart failure. Common causes of tricuspid regurgitation include congenital abnormalities of the tricuspid valve, structural abnormalities resulting from infection, and chronic pulmonary hypertension. Pacemaker lead placement is an increasingly common iatrogenic cause of tricuspid regurgitation. As tricuspid regurgitation persists, right-sided cardiomegaly, systemic venous congestion, and eventually right-sided heart failure ensue. Signs of severe tricuspid regurgitation are associated with systemic venous congestion and include distended, pulsating neck veins, a pulsatile enlarged liver, and anasarca. On cardiac auscultation, tricuspid regurgitation is a pansystolic murmur that becomes louder with inspiration and reduced with expiration or Valsalva maneuver. It is best heard at the left lower sternal border and radiates to the right lower sternal border. A harsh midsystolic crescendo-decrescendo murmur radiating to the left shoulder (A) and neck that is best heard at the second to third left intercostal space is associated with pulmonic stenosis. An early pulmonic ejection sound is common. A loud midsystolic murmur best heard with the patient sitting and leaning forward (B) is associated with aortic stenosis. Aortic stenosis is best heard at the second right intercostal space and radiates to the neck and left sternal border. A pansystolic murmur with prolonged apical impulse (D) that is best heard at the apex and radiating to the left axilla is associated with mitral regurgitation. Question: What is the Carvallo sign? Answer: The murmur in tricuspid regurgitation becomes louder with inspiration.

A 68-year-old man with a past medical history of diabetes mellitus and hypertension presents to the emergency department with acute onset of chest pain. The ECG is shown above. The initial troponin is elevated. Which of the following is the best treatment? Coronary artery bypass graft surgery Fibrinolysis Percutaneous coronary intervention Pharmacologic treatment without further intervention

Percutaneous coronary intervention Myocardial infarction is defined by myocardial injury occurring due to myocardial ischemia. Risk factors for myocardial infarction include advanced age, hypertension, hyperlipidemia, diabetes mellitus, obesity, smoking cigarettes, and prior cardiovascular disease, such as ischemic strokes, peripheral artery disease, or coronary artery disease. Patients with suspected myocardial infarction should have an ECG obtained and troponin drawn within 10 minutes of first medical contact. ST segment elevation myocardial infarction is defined by a myocardial infarction in which there is new ST segment elevation at the J point in two contiguous leads. Troponin is a biomarker that is sensitive and specific for cardiac injury. Serial troponin assays are indicated in patients who present in the first couple of hours after symptom onset. Patients diagnosed with ST segment elevation myocardial infarction should have continuous cardiac monitoring and intravenous access. Reperfusion can be achieved with fibrinolytic agents or directly with percutaneous coronary intervention. High-quality percutaneous coronary intervention is the preferred reperfusion method when it is available. Percutaneous coronary intervention is performed in the catheterization lab. It is important for percutaneous coronary intervention to be performed in a timely manner. The first medical contact time to percutaneous coronary intervention goal is 90 minutes for patients arriving at a facility that performs percutaneous coronary intervention, and the goal is 120 minutes for patients who initially arrive at a facility that does not perform percutaneous coronary intervention. Coronary artery bypass graft surgery (A) is infrequently performed emergently in patients with ST elevation myocardial infarction because coronary artery bypass graft surgery is associated with an increased risk of mortality when performed within seven days following ST segment elevation myocardial infarction. In most cases, it is best to allow myocardial recovery following acute ST segment elevation myocardial infarction prior to coronary artery bypass graft surgery. After allowing time for myocardial recovery, patients with critical anatomy should have coronary artery bypass graft surgery performed during the initial hospitalization. One exception is patients who fail initial treatment with fibrinolysis or percutaneous coronary intervention. These patients may require immediate treatment with coronary artery bypass graft surgery prior to the period of myocardial recovery. Fibrinolysis (B) is generally indicated in patients who present with acute ST segment elevation myocardial infarction who cannot have percutaneous coronary intervention within 120 minutes of first medical contact and have symptom duration of less than 12 hours. In these cases, fibrinolytic therapy should be administered within 30 minutes of hospital arrival. Furthermore, fibrinolytic therapy may be considered in patients presenting 12-24 hours after symptom onset who are still having chest pain and do not have access to percutaneous coronary intervention. Percutaneous coronary intervention is preferred when available because studies have shown lower rates of intracranial hemorrhage and recurrent myocardial infarction compared to fibrinolytic therapy. Pharmacologic treatment without further intervention (D) is incorrect because reperfusion is the most important part of the treatment of acute ST segment elevation myocardial infarction, however, pharmacologic therapy, which may include antiplatelet therapy, anticoagulant therapy, beta-blockers, statins, nitroglycerin, and morphine, also has a role in treatment. Question: What are other causes of elevated troponin in addition to acute myocardial infarction? Answer: Myocarditis, pericarditis, rapid atrial fibrillation, heart failure, pulmonary embolism, aortic dissection, and sepsis.

A 65-year-old man with a past medical history of non-small cell lung cancer who was treated with radiation therapy and chemotherapy presents to the emergency department with a three-day history of shortness of breath, fatigue, and swelling in his legs. Physical exam reveals hypotension, elevated jugular venous distension, and 2+ pitting edema bilaterally. Echocardiogram reveals a thickened pericardium. Which of the following physical exam findings would you expect to find, given the suspected diagnosis? Displaced point of maximal impulse Muffled heart sounds Pericardial knock S3 gallop

Pericardial Knock This patient presents with constrictive pericarditis. Constrictive pericarditis is a thickened, fibrotic pericardium that restricts ventricular diastolic filling. This presentation may be the result of chronic inflammation of the pericardium, connective tissue disorders, uremia, cardiac surgery, or radiation therapy. Patients with constrictive pericarditis present with dyspnea, fatigue, and signs of right-sided heart failure, such as peripheral edema. Physical exam will reveal elevated regular venous distention, hepatojugular reflux, and a pericardial knock. The pericardial knock is heard when ventricular filling suddenly ceases due to the thickened and noncompliant pericardium. Atrial fibrillation is a common complication of constrictive pericarditis that may also be noted on physical exam. An echocardiogram may reveal a thickened pericardium, abnormal septal motion, variation of ventricular filling with respiration, and dilated inferior vena cava and hepatic veins. Treatment of hemodynamically stable patients is similar to pericarditis treatment and includes colchicine and a nonsteroidal anti-inflammatory agent. Diuretics are also commonly utilized in the management of constrictive pericarditis. Treatment of hemodynamically unstable patients or patients with persistent symptoms involves a pericardiectomy. This procedure has high morbidity and mortality associated with it, so the patient should be medically optimized as best as possible prior to the procedure. Displaced point of maximal impulse (A) is a sign of cardiomegaly. Muffled heart sounds (B) are common in pericardial tamponade. Pericardial tamponade will also present with dyspnea and peripheral edema. An echocardiogram would reveal the presence of an effusion and evidence of diastolic collapse of heart chambers. An S3 gallop (D) is commonly seen in patients with heart failure with reduced ejection fraction. This extra sound is due to blood striking a compliant ventricle. Question: What electrocardiogram findings suggest acute pericarditis? Answer: Diffuse ST elevation and PR depression.

A 57-year-old man presents to a primary care provider with fatigue and stocking-glove paresthesias. Physical examination reveals a smooth tongue, cheilosis, and abnormal vibration and joint position sense. A CBC shows leukopenia, anemia, and thrombocytopenia. The MCV is 120 fL. Hypersegmented neutrophils and macro-ovalocytes are present on the manual differential. Serum homocysteine and methylmalonic acid levels are increased. What is the most likely diagnosis? Aplastic anemia Folate deficiency G6PD deficiency Pernicious anemia

Pernicious anemia Pernicious anemia is the most common cause of vitamin B12 deficiency. It is an autoimmune condition that results in the destruction of the gastric parietal cells that produce intrinsic factor. Intrinsic factor binds to vitamin B12 and is responsible for 99% of vitamin B12 absorption in the terminal ileum. Examination of the oral mucosa may reveal a smooth tongue, glossitis, or cheilosis. Neurologic changes include stocking-glove paresthesias, decreased vibratory and joint position sense, loss of fine touch sensation, loss of coordination, ataxia, or dementia. Gastrointestinal changes include anorexia and diarrhea. Patients with severe pernicious anemia and vitamin B12 deficiency may also have leukopenia and thrombocytopenia. Macrocytic anemia (MCV > 100 fL) is consistent with pernicious anemia, and the manual differential may show the presence of hypersegmented neutrophils and macro-ovalocytes. Both serum homocysteine and methylmalonic acid levels will be elevated. The treatment for pernicious anemia is lifelong vitamin B12 supplementation. Aplastic anemia (A) occurs when the bone marrow becomes hypoplastic and may result in symptoms of fatigue, weakness, and mucosal and skin bleeding. Hypersegmented neutrophils and macro-ovalocytes will not be seen on the peripheral smear in patients with aplastic anemia. Folate deficiency (B) is a macrocytic anemia similar to pernicious anemia. The differentiating laboratory finding is methylmalonic acid, which is normal in folate deficiency. Unlike pernicious anemia, neurologic symptoms are absent. G6PD deficiency (C) is an X-linked recessive disorder that may result in episodic hemolysis in the presence of oxidative drugs (e.g., aspirin, sulfonamides, nitrofurantoin), fava beans, and infection. The manual differential typically shows bite cells and Heinz bodies. Question: What parasite is the largest tapeworm that can infect humans and is a rare cause of vitamin B12 deficiency? Answer: Diphyllobothrium latum (the fish tapeworm).

A 60-year-old man with a history of coronary artery disease presents to his primary care provider for increasing fatigue and shortness of breath. He also reports a productive cough and trouble sleeping in his bed to the point he has switched to a recliner chair. Physical exam reveals 2+ pitting edema bilaterally, elevated jugular venous distention, bibasilar rales, and laterally displaced point of maximal impulse. What other physical exam finding is most commonly associated with the suspected diagnosis? Fixed split S2 Pericardial friction rub S3 gallop S4 gallop

S3 gallop This patient is presenting with systolic heart failure with reduced ejection fraction (< 55%). Left-sided heart failure is caused by coronary artery disease and hypertension, while right-sided heart failure is most commonly caused by left-sided heart failure or pulmonary disease. Systolic heart failure is associated with a reduced ejection fraction and is the most common form of heart failure. It is caused by myocardial infarctions and dilated cardiomyopathy. Patients with left-sided failure will present with dyspnea (e.g., orthopnea, dyspnea at rest, and paroxysmal nocturnal dyspnea) and complaints of pulmonary congestion (e.g., cough with pink frothy sputum and wheezing). Patients with right-sided failure will present with complaints of peripheral edema, anorexia, and nausea. Physical exam will reveal hypertension, bibasilar rales on auscultation, laterally displaced point of maximal impulse, S3 gallop, peripheral edema, jugular venous distention, and hepatojugular reflux. The S3 gallop is caused by the sudden deceleration of blood entering the left ventricle. Heart failure is diagnosed with echocardiogram. A fixed split S2 (A) is associated with an atrial septal defect. A pericardial friction rub (B) is associated with pericarditis. An S4 gallop (D) is typically associated with diastolic heart failure patients and is due to atrial contraction into a stiff left ventricle. Diastolic heart failure is caused by high blood pressure and left ventricular hypertrophy. These patients will have a normal ejection fraction and normal cardiac size. Question: Which type of pleural effusion is associated with heart failure: transudative or exudative? Answer: Transudative.

A 17-year-old boy presents to your office after experiencing a head injury while playing football 3 weeks ago. He lost consciousness for less than 5 seconds after hitting his head during a tackle. Upon regaining consciousness, he was taken to the hospital for evaluation due to experiencing symptoms of headache, nausea, and mild confusion. Computed tomography of the head was negative for any acute findings. The patient and his parents now have questions about return to activity and ongoing health maintenance after a concussion. Which of the following is the most appropriate guidance? A Annual brain imaging B Cognitive rehabilitation C Immediate return to activity D Stepwise return to activity

Stepwise return to activity Concussion is also referred to as mild traumatic brain injury and occurs as a result of head injury. It is often seen in adolescents and young adults in the context of playing sports, such as American football, soccer, and ice hockey. It is most often defined by a Glasgow Coma Scale score of 13 to 15 assessed at 30 minutes postinjury. A stepwise return to activity once the athlete is asymptomatic and under close supervision of medical providers, coaches, family, and athletic trainers will help to prevent any complications of the injury. Postconcussive syndrome is a sequelae of concussion, with symptoms including headache, dizziness, cognitive impairment, and neuropsychiatric symptoms. Most cases occur within the first 10 days after the injury with the majority being completely resolved within 3 months. Follow-up imaging of the head is dependent on the patient's ongoing symptoms and if there was imaging done at the time of injury. Most patients will have computed tomography or magnetic resonance imaging done initially as part of their acute workup. For those with ongoing severe symptoms, magnetic resonance imaging should be done to rule out any serious pathology. Annual brain imaging (A) is not recommended. Cognitive rehabilitation (B) is a controversial treatment option due to its cost and lack of research support in nonmilitary populations. More studies are needed before recommending this modality as treatment of sports-related concussions. Immediate return to activity (C) is not recommended in athletes who are suspected of having a concussion, as there is a greater risk of injury in these individuals. Players should be asymptomatic and off all medications before returning to play. Question: True or false: concussion risk is greater for female athletes participating in basketball or soccer. Answer: True.

A 32-year-old pregnant woman presents to the clinic complaining of acute onset myalgias, cough, and fever. She states she woke up this morning feeling okay and then all of a sudden felt as if she were "hit by a truck." She notes her coworker was just diagnosed with the flu a couple of days ago. Her vitals today are as follows: weight 158 pounds, body mass index 27 kg/m2, temperature 102°F, O2 98% on room air, and blood pressure 120/86 mm Hg. What is the most appropriate treatment for this patient? Azithromycin Baloxavir Oseltamivir Penicillin

The patient in the vignette above most likely has influenza. Influenza is a virus that is highly contagious and spreads via respiratory droplets. Those affected typically have a history of living or working in crowded areas such as schools, military bases, and small office spaces. Influenza most commonly presents from late fall to early spring, also known as flu season. Patients with influenza typically present with acute onset of fever, cough, myalgias, pharyngitis, and headache. Diagnosis is typically clinical, however, a rapid nasal influenza swab can be done to confirm diagnosis. Treatment should ideally be initiated within the first 48 hours of symptom onset. However, initiating therapy later than 48 hours of illness onset for those patients who are immunocompromised or pregnant up to two weeks postpartum have been shown to be beneficial. Azithromycin (A) and penicillin (D) are both antibiotics and are used as a treatment for various bacterial infections. Since influenza is a virus, the use of antibiotics is not warranted. Although baloxavir (B) is an appropriate treatment of choice for influenza, it is contraindicated in pregnancy. Since the patient in the vignette above is pregnant, baloxavir is not an appropriate treatment. Question: What is the most common strain of influenza in the human population? Answer: Type A.

A 63-year-old man presents to the clinic complaining of his "hands shaking." He notices whenever he is trying to do a specific task, such as reaching out to grab a drink or writing in his journal, his hands shake. The patient notes the tremor improves after drinking whiskey. On physical exam, there is no resting tremor noted, however, when performing the finger-to-nose exam, the patient's tremor increases as he approaches the target. The remainder of the neurologic exam is normal. Which of the following is the most appropriate first-line treatment for this patient's condition? AAlprazolam BCarbidopa CGabapentin DPropranolol

The patient in the vignette above has an essential tremor, which is also known as benign essential tremor or essential familial tremor. The cause of essential tremor is unknown, however, some studies show it as an autosomal dominant inherited disorder. There are two peaks of onset for essential tremor: during teenage years or in the sixth decade of life. Essential tremors are slow progressing and typically affect the upper extremities and head. An action tremor typically worsens in times of stress or high emotions and is present against gravity. This tremor improves with alcohol consumption. On a finger-to-nose test, the patient's tremor increases as the target approaches. Otherwise, the patient's neurological exam will have no other significant findings. The first-line medical treatment for an essential tremor is propranolol or primidone. If either propranolol or primidone alone fails, a trial of both medications combined may be effective. The third-line treatment involves using either gabapentin, topiramate, or alprazolam. Alprazolam (A) and gabapentin (C) are both third-line treatments for essential tremor and can be used once a patient has failed a trial of propranolol, primidone, and a combination of the two. Carbidopa (B) is the correct treatment for patients with Parkinson disease, which would present with a resting pill rolling tremor. It is not indicated for treatment of essential tremor. Question: What exam findings would be consistent with a finger-to-nose exam in a patient with known Parkinson disease? Answer: The tremor will improve as the patient's finger approaches the target.

A 28-year-old woman presents to the emergency department with complaints of palpitations and high fever. She had pneumonia one week ago, for which she was treated with antibiotics, and she socially drinks alcohol. Vitals are temperature 101.5°F, pulse 130 beats per minute, blood pressure 130/85 mm Hg, and respirations 20/min. Physical exam reveals an agitated patient, resting tremor, and peripheral edema. ECG reveals atrial fibrillation. What is the most likely diagnosis? Delirium tremens Myxedema coma Sepsis Thyroid storm

Thyroid Storm (hyperthyroid state + Acute event) Thyrotoxicosis, also known as thyroid storm, is a potentially fatal complication of hyperthyroidism. Thyroid storm is a rare complication but needs to be identified and treated emergently. It may be precipitated by surgery, trauma, or infection. Patients typically present with symptoms of a hypermetabolic state such as palpitations, tachycardia, high fever, tremors, or vomiting. On physical exam, they will be tachycardic, febrile, and exhibit evidence of heart failure or CNS disturbances. Ultimately, thyroid storm is a clinical diagnosis based on the presentation and history. Supporting lab work may reveal elevated free T4 and undetectable thyroid-stimulating hormone, and ECG may reveal atrial fibrillation. Thyrotoxicosis treatment involves the initiation of a beta-blocker such as propranolol to lessen the effects of thyroid storm. The next medication added is propylthiouracil or methimazole to prevent the synthesis of more thyroid hormones. The third key medication in the treatment of thyrotoxicosis is hydrocortisone. Hydrocortisone prevents the peripheral conversion of T4 to T3. The purpose of these medications is to stabilize the patient while preparing them for definitive therapy such as radioactive iodine ablation or thyroidectomy. Delirium tremens (A) is the result of alcohol withdrawal. The patient may present with tremors, tachycardia, fever, and hallucinations. This is an unlikely diagnosis for this clinical vignette, as the patient only drinks socially. Myxedema coma (B) is a complication of hypothyroidism not hyperthyroidism. These patients will present with bradycardia, obtundation, hypothermia, and hypotension. Sepsis (C) patients will present with fever, tachycardia, increased respirations, leukocytosis, and a source of infection. Septic patients will not typically present with a tremor and peripheral edema. Question: What type of heart failure is present in thyroid storm? Answer: High-output heart failure.

A 35-year-old patient presents to the hospital following a motor vehicle collision. When you arrive in the exam room, the patient is unresponsive. Which of the following exam findings most likely indicates early uncal herniation? Absent oculovestibular reflex Bilateral pinpoint pupils Gaze preference away from the lesion Unilateral dilated and fixed pupil

Unilateral dilated and fixed pupil Supratentorial lesions typically cause coma when the mass translates pressure inferiorly and causes herniation of the cerebrum through the tentorial notch. The increase in volume of the subtentorial compartment results in compression of the brainstem. This process can occur quickly or can be protracted (as is the case in slow-growing masses). The two most common herniation syndromes are central herniation (caused by a mass that directs a vertical vector of force toward the tentorial notch) and uncal herniation. Uncal herniation results from a unilateral mass causing a lateral force vector. This force vector pushes the temporal lobe over the edge of the tentorium and compresses the midbrain. Early transtentorial herniation will often present with a third cranial nerve palsy because the cranial nerve runs just inferior to the lateral edge of the tentorium. In the case of uncal herniation, only the ipsilateral eye will be affected, resulting in unilateral dilated and fixed pupil, as is the case in the patient in the vignette above. Question: What coma syndrome is caused by damage to the ventral pons? Answer: Locked-in syndrome.

A 23-year-old woman presents to the emergency department four days after surgical repair of her anterior cruciate ligament. She has no significant past medical history, and her only medication is a combined oral contraceptive pill. She reports increased pain and swelling in her left leg. On exam, her left calf is erythematous, and its circumference is 4 cm greater than her right calf. What is the initial diagnostic study of choice for the suspected condition? Computed tomography angiography Plain radiography Venous duplex ultrasound Ventilation-perfusion scan

Venous duplex ultrasound This patient presents with a deep vein thrombosis (DVT). A deep vein thrombosis results when a clot forms in large veins, most commonly the legs. Virchow triad describes the three biggest risk factors for a DVT: venous stasis, endothelial damage, and hypercoagulability. Other risk factors include increased age, malignancy, pregnancy, oral contraceptives, and stroke. Patients will present with complaints of unilateral extremity swelling and tenderness, and physical exam will reveal swelling, tenderness, warmth, and erythema.

A 32-year-old man presents to the office for a routine physical exam. He has a history of seasonal allergies but is otherwise healthy. He takes no medications, does not smoke, and drinks beer occasionally in social situations. He has an athletic build, however, he reports not exercising much due to shortness of breath while exercising. His vital signs show a blood pressure of 112/72 mm Hg, pulse of 87 bpm, temperature of 98.2°F, and respirations of 10 breaths per minute. On cardiac exam, you hear a crescendo-decrescendo systolic murmur beginning slightly after S1 that is best heard at the apex and left lower sternal border. The murmur increases with the Valsalva maneuver. His electrocardiogram is shown above. What is the most likely diagnosis? Aortic regurgitation Aortic stenosis Hypertrophic cardiomyopathy Patent ductus arteriosus

Hypertrophic cardiomyopathy Hypertrophic cardiomyopathy is a genetic heart condition that is characterized by left ventricular hypertrophy of the heart. The disease is caused by a mutation in the genes specific for cardiac sarcomere proteins and is transmitted as an autosomal dominant trait. Hypertrophic cardiomyopathy is the most common cardiac cause of death in patients younger than 35 years of age. There is a prevalence of 1:500 in the general population, and hypertrophic cardiomyopathy is found equally in men and women. There are two types of hypertrophic cardiomyopathy: obstructive and nonobstructive. Obstructive hypertrophic cardiomyopathy is caused by a narrowing of the left ventricular outflow tract due to a thickened interventricular septum. Nonobstructive hypertrophic cardiomyopathy is characterized by a stiff left ventricle, which reduces the volume of blood the ventricle can hold without impeding blood flow. In both types of hypertrophic cardiomyopathy, the histology reveals hypertrophied myocytes with associated interstitial fibrosis presenting in a chaotic and disorganized pattern. Often, patients with hypertrophic cardiomyopathy develop left ventricular outflow obstruction, diastolic dysfunction, myocardial ischemia, or mitral regurgitation. These abnormalities can lead to a variety of symptoms, including fatigue, dyspnea on exertion, chest pain, palpitations, presyncope, syncope, or sudden cardiac death. Heart failure, with dyspnea on exertion, affects over 90% of symptomatic patients and is the most common presentation in people with hypertrophic cardiomyopathy. Chest pain may occur in up to 30% of patients with hypertrophic cardiomyopathy. Chest pain is often exertional and is also commonly precipitated or worsened by heavy meals. Both supraventricular dysrhythmias and ventricular dysrhythmias occur in hypertrophic cardiomyopathy. Patients with a dysrhythmia may present with palpitations, increasing dyspnea, presyncope, or syncope, and occasionally patients will present with sudden cardiac death due to sustained ventricular dysrhythmias. EKG: A left ventricular hypertrophy pattern with or without abnormal Q waves is the most frequent electrocardiogram finding in patients with hypertrophic cardiomyopathy, and atrial fibrillation is the most common dysrhythmia seen. An echocardiogram typically reveals increased left ventricular wall thickness. Medical therapies include beta-blockers or calcium channel blockers. Diuretics should be used with caution due to the potential reduction in preload, which may exacerbate left ventricular outflow tract obstruction. Surgical management with myomectomy or alcohol septal ablation is an available option for patients who do not respond to medical management. Aortic regurgitation (A) also presents with symptoms of heart failure, however, the murmur associated with aortic stenosis, called an Austin Flint murmur, is described as a soft, high-pitched, early diastolic decrescendo murmur heard at the third intercostal space. Aortic regurgitation may also be accompanied by a "water hammer" pulse felt at the carotid arteries or by head bobbing with each heartbeat. Similar to hypertrophic cardiomyopathy, aortic stenosis (B) also presents with angina, syncope, and dyspnea. Aortic stenosis may also be detected by a crescendo-decrescendo systolic murmur, however, the murmur associated with aortic stenosis is best heard at the aorta and radiates up to the carotid arteries. Patent ductus arteriosus (D) is a common congenital heart defect where the ductus arteriosus, a structure that allows most of the blood leaving the right ventricle to bypass the pulmonary circulation and pass into the descending aorta, remains open. Similar to hypertrophic cardiomyopathy, this defect can result in cardiac dysrhythmias and symptoms of heart failure. The hallmark murmur of patent ductus arteriosus is a continuous machinery murmur that may be accentuated in systole and is best heard at the left upper chest. Question: What left ventricle thickness places a patient at increased risk for sudden cardiac death? Answer: 30 mm.

A 50-year-old man presents to a primary care provider with multiple episodes of severe headache every day for the past six days. He states that the pain is localized to his right eye and is accompanied by right-sided tearing, redness, and a runny nose. His symptoms last for 15-30 minutes, during which he is unable to sit or lie still. He denies fever, jaw claudication, nuchal rigidity, or a "thunderclap" headache sensation. Imaging is ordered to rule out other causes and is unremarkable. Which of the following is the most appropriate prophylactic therapy? 100% oxygen via nasal cannula Amitriptyline Dihydroergotamine mesylate Verapamil

Verapamil Primary headaches are divided into three categories: tension, migraine, and cluster. Cluster headaches are severe, unilateral, periorbital headaches that last between 15-180 minutes. They may occur multiple times per day for several weeks or months. Middle-aged men are most commonly affected. A family history of headaches or migraines is often absent. During a cluster headache attack, patients may be restless or aggressive. Ipsilateral lacrimation, conjunctival injection, rhinorrhea, miosis, ptosis, or eyelid edema may be present. The diagnosis for primary headaches is clinical. Red flags that warrant further investigation (e.g., brain MRI, CT scan, temporal biopsy) include age < 5 years or > 50 years, increasing frequency or intensity of headache, jaw claudication, motor or cognitive changes, the "worst headache ever," and a combination of stiff neck, fever, and malaise. The abortive treatment of choice for a cluster headache is 100% oxygen via nasal cannula. If 100% oxygen is not readily available, triptan medications (e.g., sumatriptan) or dihydroergotamine mesylate may be administered subcutaneously or intramuscularly as abortive treatment. For prophylactic treatment of cluster headaches, a short-term corticosteroid along with a calcium channel blocker (e.g., verapamil) may be given. The corticosteroid is then tapered as the calcium channel blocker takes full effect. For an acute attack, 100% oxygen via nasal cannula (A) is the treatment of choice, but it is not indicated as a prophylactic treatment of cluster headaches. Amitriptyline (B) is a tricyclic antidepressant that is useful in migraine and tension headache prophylaxis. Amitriptyline enhances the effects of serotonin and inhibits the transmission of pain signals. It is useful for patients with insomnia and should be taken at night due to its sedating effects. Amitriptyline is not used for cluster headache prophylaxis. Dihydroergotamine mesylate (C) is an ergot alkaloid that is used in the abortive treatment of cluster and migraine headaches. It is a second-line agent after triptans because of its side effect profile. Dihydroergotamine mesylate will only treat headaches that have already begun and will not prevent headaches or reduce the frequency of attacks. Question: What classes of antihypertensive medications other than beta-blockers may be used as prophylaxis for migraine headaches? Answer: Angiotensin-converting enzyme inhibitors, angiotensin II receptor blockers, and calcium channel blockers

A 60-year-old man presents to the clinic complaining of severe shortness of breath and orthopnea. An echocardiogram is significant for a pulmonary arterial pressure of 30 mm Hg, and his electrocardiography is significant for right ventricular and right atrial hypertrophy. Which of the following findings would you expect on this patient's physical exam? Accentuation of the pulmonic component of the second heart sound An opening snap Decreased jugular venous pressure No pitting edema

Accentuation of the pulmonic component of the second heart sound This patient presents with pulmonary hypertension, which is defined by a mean pulmonary arterial pressure equal to or greater than 25 mm Hg. The increase in pulmonary vascular resistance will lead to right ventricular hypertrophy and will eventually progress to right-sided heart failure. Primary or idiopathic pulmonary hypertension is rare and found in those with no history of pulmonary or cardiac disorders. Secondary causes of pulmonary hypertension are most commonly due to pulmonary diseases, such as COPD, left-sided heart failure, or due to chronic thromboembolic disease. Pulmonary hypertension typically presents with dyspnea on exertion, fatigue, and exertional chest pain. On electrocardiography, right ventricular hypertrophy with right axis deviation and right atrial abnormality are frequently present. An opening snap (B) is heard in the presence of mitral stenosis secondary to the thickened valve leaflets, not secondary to increased pulmonary pressures. Decreased jugular venous pressure (C) and no pitting edema (D) are inconsistent with physical exam findings for severe pulmonary hypertension. Pulmonary hypertension will lead to right-sided heart failure, which presents with pitting edema and increased jugular venous pressure. Question: What is the most common cause of secondary pulmonary hypertension? Answer: Chronic obstructive pulmonary disease.

A 28-year-old PA student of Asian descent presents to a primary care provider with fatigue and gross hematuria. He started having cold-like symptoms last night along with a mild sore throat and noticed bloody urine in the toilet this morning. He has not taken any recent medication other than pseudoephedrine. He reports no tobacco, alcohol, or drug use. A serum complement level is normal. What is the most likely cause of his hematuria? Berger disease Buerger disease Postinfectious glomerulonephritis Pseudoephedrine

Berger disease IgA nephropathy, also known as Berger disease, is a cause of nephritic syndrome and is the most common primary glomerular disease worldwide, particularly in those of East Asian (e.g., Chinese, Taiwanese, Korean, etc.) and Caucasian descent. It is commonly seen in boys and young men. IgA nephropathy is a primary kidney disease of IgA deposition in the glomerular mesangium. Patients classically present with sudden onset of gross hematuria 1 to 2 days after an upper respiratory infection. This synpharyngitic presentation distinguishes IgA nephropathy from postinfectious glomerulonephritis. Serological tests are nonspecific. Serum complement levels are normal. Kidney biopsy will show a focal glomerulonephritis with mesangial proliferation. Immunofluorescence will show diffuse mesangial IgA and complement C3 deposits. Buerger disease (B), also known as thromboangiitis obliterans, is a vaso-occlusive disease strongly associated with smoking. It is unrelated to Berger disease (IgA nephropathy). Postinfectious glomerulonephritis (C) typically occurs more than 1 week after a streptococcal infection, while IgA nephropathy may occur 1 to 2 days after a viral infection. Additionally, complement levels are usually decreased in postinfectious glomerulonephritis. Pseudoephedrine (D) is a medication that acts on alpha-adrenergic receptors and causes vasoconstriction. It may be used as a decongestant for cold-like symptoms. Medications that cause orange or red-orange discoloration of the urine include phenazopyridine (urinary analgesic) and rifampin (tuberculosis medication) but not pseudoephedrine. Question: What condition causes purple urine bag syndrome? Answer: Urinary tract infection in a patient with a long-term indwelling urinary catheter.

A 37-year-old man presents to his primary care provider for recurrent episodes of chest pain. He reports that the pain is midsternal and feels like an elephant is sitting on his chest. He started powerlifting four years ago and would periodically experience chest pain for a few minutes during his workouts. However, his chest pain now occurs more frequently, occasionally comes on at rest, and seems to last longer. He has a history of hypertension and hyperlipidemia. His current medication list includes sublingual nitroglycerin, aspirin, atorvastatin, and amlodipine. Which of the following in the patient's history would be most indicative of unstable angina? Angina relieved after two doses of sublingual nitroglycerin Angina responsive to amlodipine Chest pain lasting for more than 10 minutes Painful midsternal spasms at rest with preservation of exercise capacity

Chest pain lasting for more than 10 minutes Unstable angina, also known as acute coronary syndrome, is typically caused by unstable atherosclerotic plaque or a thrombus and is associated with progressively worsening angina. Patients with unstable angina may experience symptoms at rest, new onset of angina symptoms, angina brought on with less exertion than before, and an increase in frequency and duration of symptoms. Chest pain lasting for more than 10 minutes is a typical characteristic of unstable angina. Unstable angina is less responsive to sublingual nitrogen. Unstable angina is a clinical emergency and may result in cardiac dysrhythmias, cardiac arrest, or death. The American Heart Association views unstable angina and non-ST segment elevation myocardial infarction (NSTEMI) as a single entity. Positive cardiac biomarkers (e.g., troponin, CK-MB) with serial ECGs that do not show ST elevations of ≥ 1 mm in two contiguous leads are indicative of unstable angina. Risk stratification using tools such as the thrombolysis in myocardial infarction (TIMI) score aid in determining treatment. Conservative measures for unstable angina include dual antiplatelet therapy, careful monitoring with serial ECGs, echocardiography, and stress testing. Long-term pharmacologic therapy includes aspirin, nitrates, beta-blockers, statins, calcium channel blockers, angiotensin-converting enzyme (ACE) inhibitors, and ranolazine. Invasive measures for higher-risk patients include cardiac catheterization, angioplasty, and cardiac stenting. Angina relieved after two doses of sublingual nitroglycerin (A) is more typical in cases of stable angina and Prinzmetal (or variant) angina. Stable angina is typically caused by a fixed stenosis due to atherosclerotic plaque. It is predictably brought on by exertion, relieved with rest, typically lasts less than three minutes, and is responsive to sublingual nitroglycerin. Unstable angina is less likely to respond to nitroglycerin. Angina responsive to amlodipine (B) is consistent with a diagnosis of Prinzmetal angina. Prinzmetal angina is due to coronary vasospasm with characteristic painful midsternal spasms at rest with preservation of exercise capacity (D). Patients will often experience chest pain during the night or early in the morning. Prinzmetal angina is responsive to nitroglycerin and dihydropyridine calcium channel blockers (e.g., amlodipine). Beta-blockers will not alleviate chest pain and may worsen vasospasm in these patients. Question: What is a normal coronary artery calcium score? Answer: Zero, indicating no plaque deposits.

A 35-year-old woman presents to her primary care provider for a one-week history of dark urine. She has a past medical history of systemic lupus erythematosus but is not currently on medication due to a lapse in insurance. Her blood pressure is 165/105 mm Hg. Urine dipstick reveals 2+ protein and hematuria. Lab tests reveal low complement protein and azotemia. The patient is started on prednisone. What other medication is indicated for the treatment of this complication of systemic lupus erythematosus? Cyclophosphamide Furosemide Hydroxychloroquine Ibuprofen

Cyclophosphamide Systemic lupus erythematosus (SLE) is a chronic inflammatory disease that presents in the third decade of life and is most common in women. The pathophysiology of lupus is multifactorial and complex. It is occasionally induced by medications such as procainamide, hydralazine, isoniazid, or phenytoin. Patients with systemic lupus erythematosus will present with a triad of joint pain, fever, and malar rash. Other presenting symptoms may include discoid lupus rash (annular, erythematous patches on face and scalp), oral ulcers, alopecia, and fatigue. Laboratory workup will reveal a positive antinuclear antibody, positive double-stranded DNA, and positive anti-Smith antibody. Hydroxychloroquine aids in resolving skin lesions and joint pain. Complications of SLE include pericarditis, anemia, and glomerulonephritis. Lupus nephritis is treated with corticosteroids as well as an immunosuppressant such as cyclophosphamide, mycophenolate mofetil, or azathioprine. Furosemide (B) may be used in lupus nephritis if medication is indicated for blood pressure control or for edema. Hydroxychloroquine (C) is indicated for all patients with SLE, specifically to treat the skin lesions and joint pain associated with the disease. Hydroxychloroquine does not affect the outcome of kidney function in the presence of lupus nephritis. This medication would be explored in this patient once kidney function stabilized. Ibuprofen (D) is indicated for the management of arthritis pain secondary to SLE. However, it does not treat lupus nephritis. Question: What finding on renal biopsy is associated with rapidly progressive glomerulonephritis? Answer: Crescent formations.

A 12-year-old boy presents to your office for a routine sports physical exam. Family history reveals the patient had an uncle who died of a sudden unexplained cardiac event. You obtain a screening ECG, which is shown above. Which of the following best describes the pathophysiology of the suspected diagnosis? Accessory conduction pathway Atrioventricular nodal reentrant pathway Decreased atrioventricular node transmission Multiple atrial electrical foci

Accessory conduction pathway Wolff-Parkinson-White syndrome is caused by an accessory conduction pathway formed between the ventricles and atria of the heart. In Wolff-Parkinson-White syndrome, an accessory conduction pathway (also known as the bundle of Kent) allows the electrical impulse to bypass the AV node and, thus, cause preexcitation of the ventricular myocardium. The accessory pathway in Wolff-Parkinson-White syndrome can cause reentrant or supraventricular tachycardia (AFib or atrial flutter). Many patients with Wolff-Parkinson-White syndrome are asymptomatic. However, symptoms may include intermittent chest pain, palpitations, shortness of breath, or syncope. The diagnosis is usually made with an ECG. Characteristic ECG findings in Wolff-Parkinson-White syndrome include narrow complex tachycardia, a short PR interval, and a delta wave (slurred upstroke of QRS complex). AV nodal reentrant pathways (B) are the cause of paroxysmal supraventricular tachycardia. Paroxysmal supraventricular tachycardia occurs when an additional electrical conduction pathway is formed within the AV node. This new pathway causes a reentrant tachycardia. Decreased AV node transmission (C) is the cause of AV node block. AV nodal block occurs when the electrical impulse originating from the SA node is slowed or not transmitted through the AV node. AV nodal block results in mild to severe bradycardia, depending on the extent of the block. Multiple atrial electrical foci (D) cause atrial fibrillation. Atrial fibrillation causes tachycardia due to excitable atrial foci that emit electrical impulses at varying rates. Question: What antidysrhythmic medication classes are preferred when providing medical prophylaxis for reentrant tachycardia in Wolff-Parkinson-White syndrome? Answer: IA or IC.

A 32-year-old man with a history of type 1 diabetes presents with multiple complaints. He states that he has had a 15-pound unintentional weight loss over the last month and is feeling fatigued. He feels nauseous frequently and has had a loss of appetite. He has associated irritability and depression. His vital signs demonstrate a blood pressure of 89/52 mm Hg with orthostatic hypotension. You note hyperpigmentation of the elbows, knees, and knuckles. Which of the following is the most likely diagnosis? AAddison disease BConn syndrome CGastric cancer DIron deficiency anemia

Addison's disease Addison disease, or primary adrenal insufficiency, is most commonly caused by an autoimmune process but can be caused by a multitude of other processes such as infection, neoplasm, drugs, or adrenal infarction. Adrenal insufficiency causes a decrease in cortisol production, which can manifest as an acute crisis or a chronic condition that worsens over time. Patients with chronic adrenal insufficiency typically complain of unexplained or unintentional weight loss, fatigue, muscle weakness, abdominal pain, diarrhea, nausea or vomiting, loss of appetite, and irritability. Physical exam findings include hypotension that is worse with postural changes, tachycardia, sparse body hair, and hyperpigmentation of the skin, particularly over areas that endure friction. Initial lab findings may demonstrate hyponatremia, hypokalemia, and hypoglycemia. Patients may present in acute crisis and exhibit signs of shock. Conn syndrome (B), or primary hyperaldosteronism, causes increased blood pressure, muscle weakness, paresthesias, and fatigue. It is typically caused by an adrenal mass increasing the production of aldosterone. Gastric cancer (C) should always be considered when patients complain of nausea and unexplained weight loss, however, gastric cancer is typically associated with symptoms of abdominal pain and early satiety. Hyperpigmentation of the skin is not associated with gastric cancer. Iron deficiency anemia (D) may cause orthostatic hypotension and fatigue but is not associated with unexplained weight loss or hyperpigmentation. Question: What menstrual irregularity can occur in women with Addison disease? Answer: Amenorrhea.

A 23-year-old man with type 1 diabetes presents with acute onset of severe nausea, vomiting, and abdominal pain. He reports excessive thirst and frequent urination but reports no dysuria, hematuria, or urgency. On exam, he is hypotensive and tachycardic. Kussmaul respirations are also noted. Laboratory studies demonstrate serum glucose of 550 mg/dL, serum potassium of 3.1 mmol/L, a venous pH of 7.1, and a serum bicarbonate level of 13 mmol/L. A total of 2 L of 0.9% normal saline has been administered. Which of the following is the next step in management? Administration of dextrose Administration of insulin Administration of potassium chloride Oxygen therapy

Administration of potassium chloride Diabetic ketoacidosis (DKA) is characterized by ketoacidosis and hyperglycemia in a diabetic patient caused by a lack of insulin and a cycle of gluconeogenesis, lipolysis, and ketogenesis associated with elevated levels of insulin counterregulatory hormones (catecholamines, cortisol, and glucagon). Patients with early DKA present with the classic triad of polyuria, polydipsia, and weight loss. As the disease worsens, neurological symptoms (obtundation, coma, hemiparesis, seizures) may develop. Abdominal pain accompanied by nausea, vomiting, and diarrhea can also occur. A physical exam reveals signs of hypovolemia including decreased skin turgor, dry axillae and oral mucosa, low jugular pressure, tachycardia, and, if severe, hypotension. Fruity breath (due to exhaled acetone) and Kussmaul respirations (deep respirations reflecting the compensatory hyperventilation) may be present. The diagnostic criteria for DKA according to the American Diabetes Association include hyperglycemia (> 200 mg/dL), low venous pH (< 7.3) or low serum bicarbonate (< 15 mmol/L), and ketonemia (> 31 mg/dL) or ketonuria (> 80 mg/dL). ECG may show fatal dysrhythmia associated with hypokalemia and hypomagnesemia. Fluid resuscitation (normal saline 0.9% at 10 to 20 ml/kg over 1 to 2 hours) is the cornerstone of the initial management of DKA. If the patient's serum glucose falls below 200 mg/dL during fluid resuscitation, dextrose should be added to the fluids. All patients with DKA should be treated with insulin (at 005 to 0.1 U/kg/hour) 1 to 2 hours after the initial fluid resuscitation. Serum potassium should be monitored prior to the initiation of insulin. Levels lower than 3.3 mmol/L should prompt the administration of potassium chloride before insulin is started as insulin therapy causes potassium to enter the cells and can further exacerbate hypokalemia. Presence of hyperkalemia warrants the initiation of insulin. *The goal is to treat metabolic acidosis and close the anion gap, rather than treat hyperglycemia.* Patients may be discharged home when ketosis and acidosis have resolved. DKA is considered resolved when pH is > 7.3, serum bicarbonate is 18 mEq/L or greater, and glucose level is < 200 mg/dL. Newly diagnosed type 1 diabetics should be educated and exogenous insulin therapy initiated prior to discharge. Administration of dextrose (A) should be initiated if the serum glucose level falls below 200 mg/dL during the initial fluid resuscitation. This is to prevent the onset of hypoglycemia when insulin therapy is started. Administration of insulin (B) is indicated in all patients presenting with DKA. However, insulin should be held if the serum potassium is less than 3.3 mmol/L as it causes potassium to shift into the cells and will further exacerbate hypokalemia, leading to the evolution of cardiac dysrhythmia. Oxygen therapy (D) may be given in the setting of hypoxia but is not routinely indicated for the management of DKA. Question: Which serum test can be used to distinguish type 1 and type 2 diabetes mellitus? Answer: C-peptide protein, which is low or absent in type 1 diabetes.

A 32-year-old woman is being evaluated for infertility. She also complains of headaches and vision disturbance. On exam, she is noted to have bitemporal hemianopsia. Laboratory studies reveal a serum prolactin level of 210 ng/mL. Which of the following history findings would most likely be present? Amenorrhea Dysmenorrhea Increased libido Regular menses

Amenorrhea Functional hormone-secreting pituitary adenoma Most pituitary adenomas are slow-growing and benign, which means they are not cancer and do not spread to other parts of the body. However, as they grow big they can put pressure on nearby structures, such as the nerves that connect the eyes to the brain, and cause symptoms. This is known as the "mass effect." Dysmenorrhea (B), defined as painful menstruation, can be primary (caused by uterine production of prostaglandins) or secondary (due to endometriosis, adenomyosis, and uterine fibroids). Patients with a prolactinoma do not often develop dysmenorrhea but have amenorrhea or oligomenorrhea. Increased libido (C) is not seen in patients with a prolactinoma. High levels of serum prolactin inhibit the secretion of gonadotropin-releasing hormone, which in turn decreases the production of follicle-stimulating hormone and luteinizing hormone, thereby causing a decrease in the synthesis of estrogen and decreased libido. Irregular menses, not regular menses (D), also result from the decreased production of estrogen. Question: What is another name for prolactinoma? Answer: Lactotroph adenoma.

A 66-year-old man with a history of severe COPD, nonalcoholic steatohepatitis, and single-vessel coronary artery disease presents to your clinic with worsening angina. He reports that, over the last month, he has been getting two to three anginal episodes a week that are improved with sublingual nitroglycerin and rest. Vital signs show HR of 74 beats per minute and BP of 136/84 mm Hg. Physical exam is unremarkable. A recent echocardiogram showed an ejection fraction of 55%. What is the most appropriate treatment for this patient? Amlodipine Lisinopril Propranolol Ranolazine

Amlodipine for this stable angina CANNOT use beta-blockers in pts with COPD :( Lisinopril (B) is an angiotensin-converting enzyme (ACE) inhibitor used primarily as an antihypertensive agent and has additional morbidity and mortality benefits for patients with chronic kidney disease, diabetes mellitus, systolic heart failure, or personal history of myocardial infarction. It does not treat stable angina. Propranolol (C) is a beta-blocker and is usually a first-line long-term antianginal agent. However, it is contraindicated in this patient because of his severe COPD since beta-blockers may worsen airway obstruction. Ranolazine (D) is a long-term antianginal agent. It does not lower blood pressure or heart rate, and so it can be beneficial in patients who cannot tolerate beta-blockers, calcium channel blockers, or long-acting nitrates. Ranolazine is also used in patients with recent myocardial infarction. However, it is contraindicated in patients with hepatic impairment. Question: What are the primary indications for coronary artery bypass grafting in patients with coronary artery disease (CAD)? Answer: Three-vessel disease, > 50% stenosis in the left anterior descending (LAD) coronary artery, or left ventricular dysfunction.

A 42-year-old woman presents with vague complaints of fatigue and unexplained weight gain for the last several months. A thyroid-stimulating hormone assay is elevated, and free T4 is low. Which of the following physical exam findings would be most consistent with the diagnosis? Brittle fingernails Ophthalmopathy Pitting edema Skin hyperpigmentation

Brittle fingernails Clinical manifestations of hypothyroidism are caused by a general metabolic slowing secondary to a lack of thyroid hormone and an accumulation of matrix glycosaminoglycans in tissues. These manifestations can be seen across many organ systems. The skin may become cool and pale with a roughness to palpation. Sweating can be decreased, fingernails become brittle, and hair may become coarse and dry. Patients may complain of general thinning of the hair. Examination of the eyes may reveal periorbital edema. Decreased heart rate may be noted, and patients may exhibit dyspnea on exertion from a decrease in cardiac output. Macroglossia can cause sleep apnea to occur. Gastrointestinal symptoms include constipation and a decreased taste sensation. Women may complain of changes in menstrual cycles with either oligomenorrhea or hypermenorrhea. Musculoskeletal manifestations include muscle cramping, weakness, and myalgias. The diagnosis is typically made through testing of the thyroid-stimulating hormone, which will be increased in response to low levels of free T4 from the underactive thyroid gland. These signs and symptoms will be in proportion to the magnitude of the hormone disturbance at the time of diagnosis. An autoimmune cause is the most common reason for the development of hypothyroidism. Thyroid peroxidase antibodies will be positive in these patients. Treatment is dependent on the underlying cause but typically involves supplementation of T4.

A 26-year-old man with a past medical history of cystic fibrosis presents to the clinic complaining of a chronic cough with purulent, foul-smelling sputum. The patient has crackles noted to bilateral bases of lungs. A CT scan is significant for abnormal dilation of bronchial walls. Which of the following is the most likely diagnosis? Asthma Bronchiectasis Chronic bronchitis Emphysema

Bronchiectasis This patient presents with bronchiectasis, which is a disorder of the large bronchi characterized by permanent dilation and destruction of the bronchial walls. About half of all diagnosed cases are secondary to cystic fibrosis. Common symptoms include a chronic productive cough with large amounts of mucopurulent, foul-smelling sputum, wheezing, shortness of breath, and pleuritic chest pain. Typically, the physical exam findings are nonspecific, and this condition is diagnosed with a high-resolution CT scan, which will show dilated and thickened bronchi. This finding is commonly referred to as "tram tracks." The treatment for an acute exacerbation of bronchiectasis consists of antibiotic therapy based on sputum smears or prior cultures. Ampicillin, amoxicillin, trimethoprim-sulfamethoxazole, ciprofloxacin, or levaquin can be used for empirical treatment. In order to prevent exacerbation recurrences, chest physiotherapy and daily bronchodilators are the primary treatment. Asthma (A) is characterized by intermittent episodes of wheezing, cough, and chest tightness and, therefore, is not the correct diagnosis for the patient in the above vignette. Chronic bronchitis (C) is a type of chronic obstructive pulmonary disease that presents as a chronic cough with thick sputum and shortness of breath. However, this disease presents most commonly in previous smokers in their fifth or sixth decade of life. Emphysema (D) is the second type of chronic obstructive pulmonary disease that commonly presents as shortness of breath in a previous smoker, usually in the fifth or sixth decade of life. In emphysema, a cough is extremely rare and the patient's lungs are usually very quiet, meaning there are typically no crackles or rales present. Question: What is the most common organism found in a noncystic fibrosis patient with bronchiectasis? Answer: Haemophilus influenzae.

A 32-year-old woman presents to a primary care provider with complaints of back pain, hematuria, and headaches. She started keeping a blood pressure log 2 weeks ago after a high reading at a recent health fair and reports that her systolic blood pressure has been between 140-150 mm Hg. Physical examination reveals the presence of large, palpable kidneys bilaterally. The patient recalls that her 53-year-old mother and 34-year-old sister have a history of autosomal dominant polycystic kidney disease. Her estimated glomerular filtration rate is 75 ml/min/1.73 m2. What is the diagnostic study indicated at this time? CT scan Excretory infusion urography KUB radiograph Ultrasound

CT scan Polycystic kidney disease is characterized by the presence of numerous cysts in the kidney tubules and collecting system of the kidneys and is best diagnosed by a kidney ultrasound. CT scan or MRI is the test of choice for patients with typical symptoms of autosomal dominant polycystic kidney disease and a positive family history. A CT scan with and without contrast is recommended for patients with an estimated glomerular filtration rate ≥ 60 ml/min/1.73 m2 and allows for baseline imaging for future comparison, identification of complications of disease, and determination of prognosis and treatment plan. Excretory infusion urography (B) will reveal multiple lucencies in polycystic kidney disease. It may be used for urinary tract pathology but is not first-line for polycystic kidney disease. A KUB radiograph (C) will show enlarged kidneys but is not first-line for polycystic kidney disease. It is useful in identifying tumors and urinary obstructions in the kidneys, ureters, and bladder. An ultrasound (D) is the initial test of choice in asymptomatic patients with normal kidney function who have a positive family history in order to rule out the presence of polycystic kidney disease. The patient in the vignette above is symptomatic with normal kidney function, so a CT scan is the best test. Question: What two structures cause compression of the left renal vein in nutcracker syndrome? Answer: The aorta and the superior mesenteric artery.

A 62-year-old hospitalized man develops a fever and purulent sputum on day five of his admission. His oxygen desaturates to the point that he requires ventilatory support. On chest X-ray, a new infiltrate is seen. Which of the following is the best empiric antibiotic regimen? Cefepime, levofloxacin, and vancomycin Ceftriaxone and azithromycin Piperacillin-tazobactam and linezolid Piperacillin-tazobactam, amoxicillin, and vancomycin

Cefepime, levofloxacin, and vancomycin Hospital-acquired pneumonia (nosocomial pneumonia) is defined as pneumonia that occurs at least 48 hours after hospital admission. It is important to distinguish between community-acquired pneumonia and hospital-acquired pneumonia because they are caused by different pathogens and, therefore, require different antibiotic coverage. Hospital-acquired pneumonia is often caused by gram-negative bacilli, such as Escherichia coli, Klebsiella pneumoniae, Enterobacter species, Pseudomonas aeruginosa, or Acinetobacter species. Furthermore, Staphylococcus aureus and sometimes methicillin-resistant Staphylococcus aureus are also common pathogens. Patients with hospital-acquired pneumonia who have septic shock or require ventilatory support should be treated with a three-antibiotic regimen. The regimen includes 1) piperacillin-tazobactam, cefepime, ceftazidime, meropenem, or imipenem, plus 2) ciprofloxacin, levofloxacin, or aztreonam, plus 3) vancomycin or linezolid. Ceftriaxone and azithromycin (B) is the empiric treatment used for patients who are admitted to the hospital for community-acquired pneumonia who do not have risk factors for methicillin-resistant Staphylococcus aureus or Pseudomonas aeruginosa. Ceftriaxone provides coverage against typical pathogens and azithromycin provides coverage against atypical pathogens. Piperacillin-tazobactam and linezolid (C) provides coverage for gram-negative bacilli, including Pseudomonas aeruginosa. However, patients who require ventilatory support should be treated with two antibiotics that cover Pseudomonas aeruginosa. Piperacillin-tazobactam, amoxicillin, and vancomycin (D) is incorrect because the patient in the vignette needs to be treated with two agents that cover Pseudomonas aeruginosa. Amoxicillin does not cover Pseudomonas aeruginosa. Question: What are the typical bacterial pathogens that most frequently cause community-acquired pneumonia? Answer: Streptococcus pneumoniae, Haemophilus influenzae, and Moraxella catarrhalis.

A 28-year-old woman presents to the clinic with vaginal discharge and abdominal pain for one week. Pelvic exam is significant for cervical purulent drainage and no cervical motion tenderness. Gram stain of the vaginal discharge received the next day is shown above. Urine pregnancy test is negative. Which of the following is the most appropriate therapy? Ceftriaxone Ceftriaxone plus azithromycin Ceftriaxone plus doxycycline Doxycycline

Ceftriaxone plus doxycycline Neisseria gonorrhoeae is a gram-negative intracellular diplococci that can cause a variety of infections. It is commonly seen in young, sexually active individuals. In men and women, gonorrhea most commonly presents as a urogenital infection (e.g., cervicitis, urethritis, epididymitis, and prostatitis). However, there is an increasing incidence of pharyngeal and anorectal gonococcal infection in both men and women. Men who have sex with men are at increased risk for the development of anorectal and pharyngeal infections. A high proportion of individuals with pharyngeal and anorectal gonococcal infections and women with urogenital infections are asymptomatic. This lack of presenting symptoms contributes to the high prevalence of gonorrhea, which is the second most common bacterial sexually transmitted infection after Chlamydia trachomatis. Women with symptoms caused by a gonococcal infection may present with vaginal discharge, pelvic pain, dyspareunia, or intermenstrual bleeding. Men with gonorrhea will commonly present with testicular pain, dysuria, or penile discharge. Ceftriaxone alone (A) is not recommended for treatment of uncomplicated Neisseria gonorrhoeae infection because of the high prevalence of Chlamydia trachomatis coinfection. Therefore, additional coverage with doxycycline is recommended. Ceftriaxone plus azithromycin (B) is the recommended treatment in pregnant patients. Doxycycline alone (D) is not indicated for the treatment of uncomplicated Neisseria gonorrhoeae given the bacteria's rising resistance to tetracyclines. Question: What is Fitz-Hugh-Curtis syndrome? Answer: Perihepatitis caused by ascending gonococcal infection in women.

A 76-year-old woman presents to the emergency department after having a seizure. She has been vomiting for the past three days and has been lethargic since yesterday. A basic metabolic panel reveals a sodium level of 155 mEq/L. Which of the following complications should be considered when initiating volume repletion and serum sodium reduction? Cerebral edema Hypoglycemia Osmotic demyelination syndrome Prolonged QT interval

Cerebral Edema Hypernatremia is most often caused by the loss of free water from various causes. Dehydration and hypernatremia can be caused by vomiting, excessive sweating, or through uncontrolled diabetes causing increased urination. Typically, hypernatremia is balanced by thirst causing increased water intake, however, when the loss is greater than the intake, or when patients are unable to intake water freely, then dehydration and hypernatremia can occur. Hypernatremia is labeled as chronic if it has been present for more than 48 hours. Patients with chronic hypernatremia are at risk for cerebral edema with rapid correction of sodium levels. Cerebral edema is more commonly seen in children than in adults, however, slower correction rates are needed for all patients with chronic hypernatremia. Increased sodium levels initially cause osmotic water loss from brain cells and subsequent volume loss of the brain, which causes symptoms of lethargy, seizures, and potentially coma. After 48 hours, sodium and water from cerebrospinal fluid replaces the initial volume loss and the volume of the brain is restored. When plasma sodium concentrations are then rapidly corrected, additional water flows back into brain cells, causing cerebral edema. Cerebral edema can cause encephalopathy that can result in permanent brain damage. The goal for patients with chronic hypernatremia should be to lower serum sodium levels by 10 mEq/L in a 24-hour period, which is typically accomplished by administering 5% dextrose in water intravenously (at 1.35 ml/hour x the patient's weight in kilograms). In patients with acute hypernatremia, serum sodium levels can be lowered more rapidly with a goal of reaching normal levels within a 24-hour period. Hyperglycemia, not hypoglycemia (B), is a potential risk of administering 5% dextrose in water intravenously. Blood glucose levels should be monitored frequently to prevent hyperglycemia from occurring. Osmotic demyelination syndrome (C) can occur with rapid correction of hyponatremia. Administering hypertonic saline solution intravenously can cause this syndrome to occur. Prolonged QT interval (D) is associated with disturbances in serum potassium levels and is not typically associated with correction of hypernatremia. Question: How often should serum sodium and glucose levels be monitored during correction? Answer: Every two to three hours.

A 30-year-old physician assistant student on her pediatrics rotation presents to her primary care provider with a sore throat, tonsillar exudates, anterior cervical lymphadenopathy, abdominal pain, and a subjective fever. Rapid strep testing is positive, and she is treated with amoxicillin. Two weeks later, she presents with a fever of 102°F measured in the office. She has no previous history of rheumatic fever. Which of the following additional findings would be sufficient to make an initial diagnosis of acute rheumatic fever? ACarditis BChorea and reversible prolonged PR interval CErythema marginatum DPolyarthralgia and elevated erythrocyte sedimentation rate

Chorea and reversible prolonged PR interval The Jones criteria is a diagnostic tool for rheumatic fever. In patients with a recent streptococcal infection, an initial diagnosis of acute rheumatic fever can be made in the presence of two major criteria or one major and two minor criteria. Recurrent episodes of acute rheumatic fever with group A Streptococcus infection can be diagnosed with two major criteria, one major and two minor criteria, or three minor criteria. Chorea (major criteria) with a 102°F fever (minor criteria) and a reversible prolonged PR interval (minor criteria) would be sufficient to make the initial diagnosis of acute rheumatic fever. Other major criteria include carditis, erythema marginatum, subcutaneous nodules, and polyarthritis. Polyarthralgias, elevated ESR, and elevated CRP are minor criteria. The typical lesion seen on histology is a perivascular granuloma with vasculitis. Initial treatment includes strict bed rest until the fever, ESR, and heart rate have returned to normal and the patient is stable. Salicylates may be used for fever, joint pain, and swelling but have no effect on the disease course. Benzathine penicillin 1.2 million units IM once may be used to treat a current streptococcal infection and may be given once every four weeks for prophylactic prevention of recurrent acute rheumatic fever episodes. Corticosteroids may be used if joint pain persists after salicylate administration. Carditis (A) is a major criterion and along with fever would not be sufficient to make a diagnosis of acute rheumatic fever. Similarly, erythema marginatum (C) is a major criterion and along with fever would not be sufficient to make a diagnosis. Polyarthralgia and elevated ESR (D) are both minor criteria and along with fever would be sufficient to make a diagnosis of recurrent acute rheumatic fever but would not be sufficient for an initial diagnosis. Question: What are the enlarged macrophages called that are found within Aschoff bodies and are pathognomonic for acute rheumatic fever? Answer: Anitschkow cells (also known as "caterpillar cells" due to the appearance of chromatin).

A 47-year-old man presents to the clinic for evaluation of insomnia, fatigue, abdominal pain, and progressive lower extremity edema over the past nine months. He has a history of hypertension and major depressive disorder. His medications include lisinopril and duloxetine. He does not use tobacco and drinks one to two alcoholic beverages per month. He has a history of intravenous heroin use 18 years ago but does not currently use any illicit drugs. His vital signs are within normal limits. No abnormalities are heard on cardiopulmonary auscultation. His abdomen is soft and nontender with moderate distention. Physical examination is also significant for palmar erythema and pedal edema bilaterally. Abdominal ultrasound shows mild ascites, hypoechoic nodules within the liver, and splenomegaly. Which of the following is the most likely underlying cause for this patient's condition? Alcoholic liver disease Chronic hepatitis C Nonalcoholic fatty liver disease Nonalcoholic steatohepatitis

Chronic hepatitis C This patient's presentation is consistent with hepatic cirrhosis, most likely due to chronic hepatitis C virus (HCV) infection secondary to intravenous heroin use. Once a patient is infected with HCV, chronic HCV infection typically occurs, with up to 85% of patients developing chronic hepatitis. Chronic infection is slowly progressive and may not manifest as clinically apparent liver disease in many cases. Patients who are chronically infected can develop cirrhosis over a period of 20 to 30 years. Symptoms of chronic HCV infection are typically nonspecific and may be associated with cirrhosis or extrahepatic findings (e.g., renal disease, cryoglobulinemia) directly related to HCV infection. Generalized symptoms include fatigue, insomnia, abdominal pain, anorexia, arthralgia, myalgia, and weight loss. Neuropsychiatric symptoms (e.g., depression, cognitive impairment) are also commonly seen. Physical exam findings may include abdominal distention, enlarged superficial venous vasculature of the abdomen (caput medusa), blood per rectum, spider angiomas, splenomegaly, jaundice, ascites, asterixis, and palpable liver margin. Diagnosis is made through laboratory studies, but imaging may also show fibrotic changes, hypoechoic nodules, and significant coarseness of the liver serosa. Biopsy shows fibrotic changes and possibly lipid deposits. Diagnosis is made with reactive HCV antibody followed by confirmation with HCV RNA testing. Alcoholic liver disease (A) is unlikely based on this patient's limited alcohol consumption and appearance of hypoechoic nodules on ultrasonography. Steatosis is seen in up to 90% of patients with liver disease. Nonalcoholic fatty liver disease (C) and nonalcoholic steatohepatitis (D) appear hyperechoic on ultrasound. Question: What is the most frequent indication for liver transplantation in the United States? Answer: Chronic hepatitis C virus infection.

A 45-year-old man with progressive dyspnea is given a survival prediction of less than 10 years. His computed tomography scan is significant for diffuse reticular opacities. Considering the advanced stage of his current disease, what other findings would you expect on his physical exam? Bilateral lungs clear to auscultation Clubbing of his fingertips Decreased FEV1/FVC Pursed lip breathing

Clubbing of fingertips This patient presents with idiopathic pulmonary fibrosis, which is a chronic progressive interstitial scarring from persistent inflammation. The most significant risk factor for developing interstitial pulmonary fibrosis is cigarette smoking. The second most common risk factor is being exposed to metal dust, which can be seen with patients who work as coal miners or metal workers. A CT scan shows diffuse reticular opacities, also known as "honeycombing," and ground-glass opacities. A biopsy can be performed and would also show honeycombing, fibrosis, and scarring. They would have clubbing of the fingers and inspiratory crackles on physical exam. Bilateral lungs clear to auscultation (A) is incorrect. Due to advanced disease, the patient in the above vignette would have significant inspiratory crackles to bilateral lung bases. Decreased FEV1/FVC (C) is consistent with obstructive pulmonary disease, not restrictive. Pursed lip breathing (D) is more consistent with physical exam findings of a patient with emphysema. Question: What would a pulmonary function test for a patient with restrictive lung disease show? Answer: It would show a normal or increased FEV1/FVC, a decreased total lung capacity, and a decreased pulmonary reserve volume.

A 65-year-old woman with a history of deep vein thrombosis who is taking warfarin reports to her primary care provider with a three-week history of lower leg swelling, pain, and an itchy rash on the leg that was previously affected by the deep vein thrombosis. Physical exam reveals 2+ pitting edema of the affected leg, brownish discoloration, and a diffuse, flaking, weeping rash over her affected calf. Venous ultrasound does not show new deep vein thrombosis. What initial intervention is recommended? Altering anticoagulation therapy Compression stockings Radioablation therapy Warm compresses

Compression stockings This patient presents with chronic venous insufficiency, which is a vascular incompetence of either the deep veins, the superficial veins, or both. Most commonly, it occurs after a deep vein thrombosis or trauma to the leg. Other risk factors include advancing age, obesity, prolonged standing, and pregnancy. Patients will present with leg pain that is described as burning, aching, or cramping. They will report increased pain and changes in the skin color with prolonged standing or sitting, and the pain improves with leg elevation and walking. On physical exam, there will be peripheral edema with increased calf circumference, pitting edema, and varicosities. Patients may also have stasis dermatitis, which is an eczema-like rash with scaling and weeping erosions. The skin will appear brownish-red, which is the result of hemosiderin deposition in the skin from injured vessels in the legs. The most severe complication of chronic venous insufficiency is venous stasis ulcers over the medial malleolus. Of note, peripheral pulses and leg temperature should be within normal limits. Compression stockings and leg elevation are the mainstays of treatment. Surgical ablation of incompetent veins is an option for patients with significant or refractory symptoms. It is important to also maintain control of ulcer progression by promoting healing with wet to dry dressings and controlling edema. Recurrence is common, and lifelong compression therapy is recommended for all patients. Altering anticoagulation therapy (A) might be indicated if the patient in the above vignette developed a new deep vein thrombosis while on her current medication regimen, but the ultrasound ruled this out. Radioablation therapy (C) is indicated in severe, refractory cases of chronic venous insufficiency. The patient in the vignette has not tried conservative therapy yet, so compression stockings should be utilized initially. Warm compresses (D) are indicated for the treatment of superficial thrombophlebitis, which presents with localized tenderness, pain, and erythema over a palpable vein. Question: What vascular condition does hair loss on the distal lower extremities indicate? Answer: Arterial insufficiency.

A 53-year-old man presents for a routine complete physical. Screening blood work is completed that demonstrates a fasting blood glucose of 189 mg/dL and hemoglobin A1C of 7.8%. The patient is prescribed metformin. Which of the following is the most common side effect of this medication? Diarrhea Hypoglycemia Lactic acidosis Vitamin B12 deficiency

Diarrhea The adverse effects that are associated with metformin include gastrointestinal upset, including diarrhea, nausea, a metallic taste in the mouth, and anorexia. These are the most common side effects associated with this medication, and patients should be aware of these before beginning this drug. These side effects are often transient in nature and can be controlled by reducing the dosage of the medication or discontinuing the drug if the side effects are intolerable. Hypoglycemia (B) is not a side effect of metformin, which makes it a more desirable choice than other antidiabetic medications. Hypoglycemia is more common with insulin and sulfonylurea medications. Lactic acidosis (C) is a serious but uncommon adverse event that can occur while taking metformin. This drug should not be administered to those at risk of developing lactic acidosis, including those with impaired kidney or liver function, a history of alcohol abuse, or a past history of lactic acidosis with metformin use. Vitamin B12 deficiency (D) can occur while taking metformin, however, this side effect is less common and is typically not severe enough to cause a megaloblastic anemia.

A 46-year-old man from New Mexico with a past medical history of a renal transplant presents to the emergency department with one week of cough and fever. Vital signs are T 100.6°F, HR 108 bpm, BP 116/76 mm Hg, RR 20.min, and oxygen saturation 97% on room air. Chest X-ray shows an upper lobe infiltrate with hilar adenopathy. A fungal pathogen is suspected. Which of the following is the recommended treatment? Amphotericin B Ceftriaxone and azithromycin Fluconazole Supportive care

Fluconazole Coccidioidomycosis is a fungal infection that most commonly manifests clinically with pulmonary disease. It is endemic in the southwestern part of the United States, including Arizona, New Mexico, central and southern California, and western Texas. The treatment of coccidioidomycosis varies based on the immune status of the patient and the severity of illness. Healthy patients with mild to moderate illness do not require antifungal treatment. Healthy and nonpregnant patients with severe illness (severe dyspnea or hypoxia on room air) are treated with fluconazole or itraconazole. Amphotericin B should be used in patients with respiratory compromise requiring mechanical ventilation. Immunocompromised patients should be treated with fluconazole or itraconazole regardless of disease severity. Patients with coccidioidomycosis should be monitored following treatment for the resolution or return of symptoms. Sequelae may include prolonged fatigue or the development of pulmonary nodules.

A 55-year-old woman is brought to her primary care provider by her daughter, who reports the patient has been exhibiting some strange behavior and personality changes over the past year. The daughter says the patient has been more withdrawn and has lashed out at family members and pets. The daughter also notes the patient has been caught multiple times with inedible objects in her mouth. The patient has no history of psychiatric illness or previous behavioral problems. The physical examination is significant for the presence of palmar grasp and rooting reflexes. A Mini-Mental State Examination is normal. MRI is ordered, and a neurology consult is made. Which of the following is the most likely diagnosis? Alzheimer disease Frontotemporal dementia Pseudodementia Vascular dementia

Frontotemporal dementia Frontotemporal dementia is a neurodegenerative disease that occurs as a result of a pathologic buildup of proteins in the frontal and temporal lobes of the brain. The usual onset is between 40-75 years of age. Frontotemporal dementia has two clinical subtypes: behavioral variant and primary progressive aphasia. Behavioral variant frontotemporal dementia is the most common subtype and presents with changes in the patient's behavior, personality, and social conduct. Patients may display impulsivity, engage in criminal behavior, make inappropriate sexual advances, revert to a child-like sense of humor, or fail to recognize social cues. Family members may notice the patient seems more withdrawn, apathetic, or emotionally disconnected. Patients can become uncaring, unsympathetic, and lash out at family members or pets. Hyperorality symptoms are common and include binge eating, excessive consumption of sugary products, placing inedible objects in the mouth, or eating only one type of food. An increase in alcohol or tobacco use may also occur. Repetitive and compulsive behaviors and speech are associated with frontotemporal dementia. Primary progressive aphasia is another subtype of frontotemporal dementia and is further classified into progressive nonfluent aphasia, semantic dementia, and logopenic phonological aphasia. Definitive diagnosis is done with direct visualization of neuronal protein accumulation in the brain during autopsy. There are no FDA-approved treatments for frontotemporal dementia. Alzheimer disease (A) is uncommon in patients < 65 years old. Intellectual decline is more prominent in Alzheimer disease, and patients will typically have a poor score on mental status screening tests. Pseudodementia (C) is used to describe patients with a psychiatric illness (commonly major depressive disorder) who display features of dementia. Primitive reflexes are not present with pseudodementia. Vascular dementia (D) is associated with hypertension and is caused by lacunar or multiple cortical infarctions. Social behaviors are typically normal in vascular dementia. Question: What is Pick disease? Answer: A type of frontotemporal dementia caused by an abnormal amount of tau proteins (Pick cells).

A 68-year-old man presents to the clinic complaining of abdominal distention and shortness of breath. He takes no daily medications, has not seen a healthcare provider in decades, and has consumed 12 beers per day for the past 20 years. On physical exam, vital signs are within normal limits. Skin is nonicteric, and body habitus demonstrates central adiposity. Physical exam reveals a firm, irregular liver border just below the right costal margin, abdominal fluid wave, and dullness to percussion with decreased breath sounds on the right chest. Which of the following represents appropriate initial management of this patient's complaints? AFurosemide 20 mg daily and salt restriction BInsertion of a thoracostomy tube in the right chest and abdominal paracentesis CLiver transplantation DMetoprolol tartrate 50 mg bid and potassium restriction

Furosemide (Lasix) 20 mg daily and salt restriction Clinically, many factors point to cirrhosis in an individual at increased risk for the disease. These include ascites, platelets under 160,000/µL, spider angiomata, a diminished ALT:AST ratio, and a prolonged international normalized ratio. Hepatomegaly may be evident on exam, although advanced cirrhosis can lead to liver consolidation. The liver border may feel nodular on exam. Complications of cirrhosis are many and include ascites, spontaneous peritonitis, hepatorenal syndrome, hepatic heart failure, pleural effusion (generally on the right), hepatic encephalopathy, splenomegaly, bleeding, variceal formation, and variceal rupture. Patients with ascites can be treated with diuretics such as furosemide and salt restriction. Occasional therapeutic paracentesis may be necessary for relief of distention. Patients with cirrhosis must be counseled to abstain from alcohol, hepatotoxic medications such as acetaminophen, and certain hepatotoxic supplements and herbal remedies. Although this patient has signs of a pleural effusion, insertion of a thoracostomy tube in the right chest (B) should be avoided because this can lead to infection, severe electrolyte imbalance, renal failure, and bleeding. Right-sided pleural effusion in a patient with hepatic failure and cirrhosis is known as hepatic hydrothorax and can be treated with diuretics and salt restriction. Question: What is midodrine and how is it used in the treatment of cirrhosis-induced hyponatremia? Answer: Vasoconstrictor used to maintain mean arterial pressure in hyponatremic patients with low blood pressure due to third-spacing of ascites.

A 50-year-old man presents with a gradual onset of weight loss, malaise, and abdominal pain. On exam, he is noted to have painful violaceous plaques surrounded by livedo reticularis on the extremities. Laboratory studies demonstrate elevated C-reactive protein and erythrocyte sedimentation rate. Which of the following diseases should be screened for based on the suspected diagnosis? Acute cholecystitis Hepatitis A Hepatitis B Hepatitis E

Hepatitis B Polyarteritis nodosa is necrotizing inflammation of medium-sized vessels that predominantly affects the skin, the heart, peripheral nerves, mesenteric vessels, and the brain. The lungs are spared. Although the exact cause is unknown, it is believed to be due to the deposition of immune complexes that results in vessel injury. Men are more commonly affected than women, with the onset of disease generally between 40 and 60 years of age. Polyarteritis nodosa is associated with hepatitis B, hepatitis C, and a mutation in the gene for adenosine deaminase 2. Patients present with gradual onset of fever, malaise, weight loss, abdominal pain, peripheral neuropathy, arthralgia, and arthritis. Skin findings include livedo reticularis, subcutaneous nodules, and skin ulcers. When the kidneys are affected, hypertension, edema, oliguria, and uremia may be present Acute cholecystitis (A), inflammation of the gallbladder caused by obstruction of the cystic duct, is not associated with polyarteritis nodosa. Hepatitis A (B) and hepatitis E (D) are not associated with polyarteritis nodosa. Hepatitis A is a viral infection that is transmitted by fecal-oral contamination by hepatitis A virus. Hepatitis E is transmitted by fecal-oral contamination by hepatitis E virus. Both infections are mild and self-limiting and can be prevented by proper hand-washing and good water supply. Question: Which large vessel vasculitis is characterized by temporal artery inflammation, jaw claudication, and amaurosis fugax? Answer: Giant cell arteritis.

A 45-year-old woman with a past medical history of diabetes mellitus presents to the emergency department with one week of cough and dyspnea. She is from Mississippi and works on a farm. She denies any history of smoking cigarettes. Chest X-ray shows a unilateral infiltrate with mediastinal adenopathy. An angiotensin-converting enzyme level is checked and is normal. Which of the following is the most likely diagnosis? ALung cancer BPulmonary coccidioidomycosis CPulmonary histoplasmosis DSarcoidosis

Histoplasmosis is the most prevalent endemic fungal infection in the United States. It is caused by the fungus Histoplasma capsulatum. Histoplasma capsulatum grows best in soil contaminated with bird or bat droppings. Common sites of exposure include chicken coops, farm buildings, caves, and bird roost sites. Commonly associated activities or occupations include construction, demolition, remodeling, excavation, and exploring caves. Histoplasmosis is endemic in the states along the Ohio and Mississippi River valleys. Most cases of histoplasmosis are asymptomatic or manifest with self-limited and mild symptoms. However, individuals who are immunocompromised or exposed to large inoculums are more likely to have significant clinical symptoms. Symptoms of mild to moderate acute pulmonary histoplasmosis include cough, dyspnea, chest pain, fever, chills, headache, anorexia, and myalgias. Severe cases may progress to respiratory failure requiring ventilatory support. Lung cancer (A) may also present with an infiltrate and thoracic adenopathy, however, the patient in this vignette does not smoke cigarettes and has an acute cough. In addition, she has risk factors for exposure to histoplasmosis, therefore, histoplasmosis is the most likely diagnosis. Pulmonary coccidioidomycosis (B) is endemic in the southwestern United States, such as Arizona, New Mexico, and parts of California and Texas. The patient in this vignette lives in an area endemic for histoplasmosis. Sarcoidosis (D) can manifest with similar clinical manifestations and imaging findings as histoplasmosis. The patient in this vignette had a normal angiotensin-converting enzyme level, which is elevated in 75% of patients with sarcoidosis. Furthermore, sarcoidosis is most common in young adults and African Americans. In addition, this patient has clear risk factors for histoplasmosis. Question: What is the risk of mistakenly diagnosing histoplasmosis as sarcoidosis? Answer: Treating histoplasmosis with corticosteroids or other immunosuppressive agents can lead to disseminated disease.

A 40-year-old woman presents to her primary care provider with complaints of hand pain. She states her wrists and hands hurt and feel very stiff every morning. She does not notice the pain as much later in the day after she has been moving around for a while. Physical exam reveals ulnar deviation at the metacarpophalangeal joint bilaterally, as well as swollen, tender interphalangeal joints. What physical exam finding is most likely, given the suspected diagnosis? Hard, fixed nodules at the distal interphalangeal joint Hyperextension of the distal interphalangeal joint with flexion of the proximal interphalangeal joint Inability to extend the distal interphalangeal joint Unstable first metacarpophalangeal joint in the neutral position

Hyperextension of the distal interphalangeal joint with flexion of the proximal interphalangeal joint The joint stiffness usually lasts for more than 60 minutes each morning. Joint complaints will classically be symmetric. On physical exam, swollen and tender joints with erythema will be noted bilaterally. A boutonnière deformity may be noted in their digits. This deformity involves flexion at the proximal interphalangeal joint and hyperextension at the distal interphalangeal joint. The swan neck deformity is also classic to rheumatoid arthritis. This deformity is characterized by flexion at the distal interphalangeal joint and hyperextension at the proximal interphalangeal joint. Finally, ulnar deviation at the metacarpophalangeal joint is a classic physical exam finding. The best initial test is the rheumatoid factor, which is sensitive but not specific. A hard, fixed nodule at the distal interphalangeal joint (A) is referred to as a Heberden node and is classically seen in patients with osteoarthritis. The inability to extend the distal interphalangeal (C) is seen in patients with mallet finger. They are unable to extend this joint due to an avulsion of the extensor tendon. Instability of the first metacarpophalangeal (D) is seen in patients with skier's thumb. These patients have torn their ulnar collateral ligament. This is an acute injury due to forced abduction of the thumb. Question: What class of biologic medication should be avoided in patients with multiple sclerosis? Answer: Tumor necrosis factor inhibitors.

A previously healthy 50-year-old man presents to your office for his annual physical exam. He wants to make sure he is up to date on all of his preventive screenings. He asks about the need for hepatic cancer screening because one of his friends was recently diagnosed. Which of the following groups should be screened for hepatic cancer? Individuals aged 50 years and older Individuals with a 30 pack-year history of smoking Individuals with alcohol use disorder Individuals with liver cirrhosis

Individuals with liver cirrhosis Hepatocellular carcinoma is a leading cause of cancer-related death worldwide. In patients with cirrhosis of the liver, this statistic increases to being the leading cause of death. The prognosis of hepatocellular cancer depends on the stage of the disease at the time of diagnosis, with early stages having curative options and advanced stages having high mortality rates. High-risk patients, such as those with known cirrhosis and those with chronic hepatitis B infection, should be screened at regular intervals for hepatocellular carcinoma. Some medical societies also recommend screening for patients with chronic hepatitis C infection or advanced liver fibrosis. Screening for most at-risk patients involves abdominal ultrasound every 6 months, with a specific focus on the liver and sometimes the spleen. Computed tomography or magnetic resonance imaging is sometimes used, however in general, further imaging should be reserved for patients who have a lesion on the liver that is greater than 1 cm in size. Hepatocellular carcinoma screening has been associated with early detection of tumors, increased rates of curative treatment, and improved survival rates, which highlights the importance of screening high-risk patients. Patients with positive findings on screening should be referred for treatment immediately to avoid progression of the disease into stages that are not curable. Individuals aged 50 years and older (A) without a history of liver cirrhosis or chronic hepatitis infection do not require screening for hepatocellular cancer. Preventive screening for this age group includes screening for colorectal cancer with either colonoscopy or fecal occult blood test. Lung cancer screening is recommended for individuals aged 55-80 years with a 30 pack-year history of smoking (B). This screening is done with low-dose computed tomography of the chest. A careful history should be taken in individuals with alcohol use disorder (C) to determine the presence of liver cirrhosis or chronic hepatitis B or C infection, however, alcohol use disorder on its own is not an indication for hepatocellular carcinoma screening. Question: True or false: alpha-fetoprotein testing alone is an acceptable alternative for hepatocellular carcinoma screening. Answer: False.

A 62-year-old man with chronic obstructive pulmonary disease uses an albuterol inhaler as needed for symptoms. Despite using the albuterol inhaler, the patient has frequent daily symptoms. He denies any exacerbations in the past year. Which of the following additions to his therapeutic regimen would be most appropriate? Inhaled ipratropium Inhaled tiotropium Oral prednisone Oral theophylline

Inhaled tiotropium The diagnosis of chronic obstructive pulmonary disease is confirmed when a patient with compatible symptoms is found to have irreversible obstructive airflow on spirometry. Chronic obstructive pulmonary disease is staged according to the GOLD criteria using spirometry. Smoking cessation is the most important intervention in the treatment of chronic obstructive pulmonary disease. Inhaled ipratropium (A) is a short-acting muscarinic antagonist. The patient described in this vignette is a group B patient, which means that he has more severe symptoms but is at low risk for future exacerbations. The best treatment for a group B patient is a short-acting bronchodilator (usually albuterol) and a long-acting bronchodilator, such as tiotropium. Oral prednisone (C) is a systemic steroid. Systemic steroids are regularly used to treat exacerbations of chronic obstructive pulmonary disease but are not recommended for long-term chronic use due to the adverse effects and possible increase in morbidity and mortality. In rare circumstances when systemic steroids are used long-term to treat chronic obstructive pulmonary disease, the systemic steroid should be used at the lowest possible dose and typically while under the care of a pulmonologist. Oral theophylline (D) is an oral bronchodilator that is less effective and has more side effects than inhaled bronchodilators. Theophylline works by modest bronchodilation and enhancing respiratory muscle function. Due to reduced efficacy and increased adverse effects, theophylline should not be used as a first- or second-line therapy. Question: Which antibiotic is used chronically in rare chronic obstructive pulmonary disease patients who have frequent exacerbations despite optimal therapy? Answer: Azithromycin.

A 72-year-old man with a past medical history significant for hypertension presents for a routine yearly physical exam. He has been feeling well and has no complaints. His physical exam is benign, and a basic metabolic panel is ordered as part of a routine series of blood tests. The results show a calcium level that is 0.8 mg/dL above normal limits. When compared to last year's results, this is a new lab finding. Which of the following studies should be ordered next to confirm the most likely diagnosis? 1,25-dihydroxyvitamin D Intact parathyroid hormone Phosphate Thyroid ultrasound

Intact parathyroid hormone Primary hyperparathyroidism describes a condition where the regulation of serum ionized calcium by parathyroid hormone is abnormal. Normally, parathyroid hormone is secreted in response to lower calcium levels. This hormone acts to increase bone resorption, increase intestinal calcium absorption, and decrease urinary calcium excretion to raise serum calcium levels back into the normal range and maintain homeostasis. In primary hyperparathyroidism, there is hypersecretion of parathyroid hormone and, therefore, increased serum calcium levels. This hypersecretion may be caused by previous radiation exposure to the head or neck, genetic defects, adenomas of the parathyroid gland, glandular hyperplasia, or carcinoma, with adenomas being the most common cause. Measuring intact parathyroid hormone confirms the diagnosis. Other testing includes a 24-hour urine calcium test to assess renal function and genetic testing if a genetic predisposition or abnormality is suspected. Bone mineral density testing should be considered as part of the evaluation. The most definitive management of primary hyperparathyroidism is through a parathyroidectomy. Calcitriol, or 1,25-dihydroxyvitamin D (A), may be elevated in patients with primary hyperparathyroidism since more calcitriol is being converted in these patients from calcidiol. However, this would not lend a diagnosis. Phosphate (C) is inversely proportional to serum calcium levels. In patients with primary hyperparathyroidism, serum phosphate levels are typically on the low end of the normal range or decreased. This finding would be expected in a patient with elevated calcium levels, however, it would not help confirm the most likely diagnosis. A thyroid ultrasound (D) would not be helpful in determining the etiology of hypercalcemia, however, if an adenoma was suspected, imaging of the parathyroid may be useful in determining that diagnosis after serum parathyroid studies showed an elevation in parathyroid hormone levels. Question: What is the most common presentation of patients with primary hyperparathyroidism? Answer: Patients are almost always asymptomatic at the time of diagnosis.

A 54-year-old woman is being discharged on warfarin after being admitted for new-onset atrial fibrillation. Which of the following foods should she be advised to avoid while on warfarin? Bananas Kale Oats Pasta

Kale Atrial fibrillation is the most common cardiac dysrhythmia. It can lead to significantly decreased cardiac output. Atrial fibrillation can be asymptomatic, however, symptoms of new-onset atrial fibrillation include palpitations, angina, fatigue, altered mental status, chest pain, dyspnea, and symptoms of heart failure. Diagnosis is made via 12-lead ECG showing an irregularly irregular rhythm without any discernible P waves and can be confirmed with echocardiogram. Initial treatment of symptomatic patients involves rate control, generally using drugs that slow conduction across the atrioventricular node, such as beta blockers (metoprolol), non-dihydropyridine calcium channel blockers (diltiazem or verapamil), or digoxin. Rhythm control should be considered once the risk of thrombus production has been assessed and ruled out. Patients who have been in atrial fibrillation for less than 48 hours or are at low risk of thrombus formation can receive rhythm controlling interventions at presentation. These patients should be anticoagulated with heparin or enoxaparin and warfarin or dabigatran prior to cardioversion. Those who have been in atrial fibrillation for more than 48 hours are at increased risk for thrombus formation and should be anticoagulated with the above-mentioned medications for three to four weeks before cardioversion. Patients on warfarin should avoid foods that are high in vitamin K such as kale and other dark leafy greens, ethanol, cranberry juice, and other foods high in vitamin E due to the risk of altering the efficacy of warfarin. Question: What is the mechanism of action of apixaban? Answer: Direct factor Xa inhibitors help to prevent the breakdown of prothrombin to thrombin by inhibiting factor Xa.

Which of the following joints are most commonly affected in patients with pseudogout? First metatarsophalangeal joint Hip Knee Pubic symphysis

Knee Pseudogout, also known as acute calcium pyrophosphate deposition, is an inflammatory arthropathy that commonly occurs in large joints. The risk of developing pseudogout increases with age and typically presents in patients over 65 years of age. Other factors indicated in the development of the disease include osteoarthritis, rheumatoid arthritis, trauma, and history of surgery to the affected joint. It is also associated with conditions such as hemochromatosis, hypomagnesemia, and hyperparathyroidism. It occurs equally in men and women. Crystal deposition is thought to be precipitated by trauma, microtrauma, sepsis, changes in serum calcium levels, and surgery to the joint. Patients will present with acute joint pain to one or more joints, with associated effusion, erythema, and warmth. The knee is the most commonly affected joint. Question: What type of crystals are found in joints affected by gout? Answer: Monosodium urate crystals.

A 45-year-old African American woman presents to the ED, brought by her daughter, who reports the patient had a sudden-onset severe headache while playing the clarinet, followed by nausea, vomiting, and brief loss of consciousness. A noncontrast head CT scan is ordered, as seen above. Which of the following diagnostic studies is indicated next? Digital subtraction angiography Head CT scan with contrast Lumbar puncture Magnetic resonance angiography

Lumbar Puncture The patient in the vignette above has a presentation concerning for a spontaneous subarachnoid hemorrhage. Patients with subarachnoid hemorrhage may report a sudden "thunderclap" headache that is described as "the worst headache of their life." Nausea, vomiting, and loss of consciousness may occur after the initial headache. The patient may regain consciousness or may progress to a coma. Patients who regain consciousness may present with altered mental status or signs of meningeal irritation (e.g., nuchal rigidity). The most common cause of nontraumatic subarachnoid hemorrhage is a ruptured saccular aneurysm. In patients with a normal CT scan (as seen above), a lumbar puncture for cerebrospinal fluid (CSF) evaluation should be performed. Four CSF tubes are obtained, and an elevated red blood cell count that does not clear from the first to the fourth tube is indicative of a subarachnoid hemorrhage. Digital subtraction angiography (A) is useful for evaluating the entire vasculature after the patient is stabilized because up to 20% of patients will have multiple aneurysms, however, a lumbar puncture is indicated in the acute setting for emergent diagnosis of subarachnoid hemorrhage in the presence of an unremarkable noncontrast head CT scan. A head CT with contrast (B) is not indicated to confirm a subarachnoid hemorrhage. The contrast can interfere with the detection of blood since blood and contrast appear similar on a CT scan. Magnetic resonance angiography (D) is not as sensitive as digital subtraction angiography in the detection of other aneurysms. It is not indicated in the acute setting because it is a time-consuming test. Question: What are the four types of cerebral aneurysms? Answer: Saccular, fusiform, dissecting, and mycotic.

A 60-year-old man presents to the ER for acute abdominal pain. The patient has a significant past medical history of hepatitis B. The patient reports a history of recent malaise, 10 lb weight loss, and fevers leading up to this episode of abdominal pain. Physical exam reveals elevated blood pressure, diffusely tender abdomen, and a rash on the patient's legs. Computed tomography with IV contrast of the abdomen reveals microaneurysms with sudden cutoffs and evidence of mesenteric ischemia. What description of the rash would be consistent with the suspected diagnosis? Diffuse maculopapular Mottled, lace-like purple discoloration Palpable purpura Violaceous, raised discoloration

Mottled, lace-like purple discoloration Polyarteritis nodosa is a rare systemic vasculitis of medium-sized arteries. The vasculitis leads to necrotizing lesions. The vasculitis also leads to an increased incidence of microaneurysms, which can rupture or hemorrhage, causing organ ischemia. There is an increased association with viral hepatitis B, hepatitis C, and hairy cell leukemia. Manifestations of the disease will depend on which arteries are affected. Patients may present with myalgias, arthritis, weight loss, symptoms of peripheral neuropathy, and skin changes. There are multiple different dermatologic manifestations such as livedo reticularis, ulcers, gangrene, and Raynaud phenomenon. Livedo reticualris is a mottled, lace-like purple discoloration of the skin. The physical exam will reveal hypertension, fever, and skin changes. Diagnostic workup requires looking at inflammatory markers. Unlike many other types of vasculitis, classic polyarteritis nodosa is ANCA negative. A diffuse maculopapular (A) rash is not specific to any one disease and is seen in conjunction with multiple ailments such as allergic reactions or viral infections. Palpable purpura (C) is noted in other types of vasculitis. It is classic in microscopic polyangiitis and Henoch-Schönlein purpura. A violaceous, raised discoloration (D) describes lupus pernio, which is classically seen with sarcoidosis. Question: Which organ involvement is classically spared in polyarteritis nodosa? Answer: The lungs.

A 20-year-old college student presents to the clinic complaining of a low-grade fever and right-sided ear pain that started three days ago. She also notes a persistent nonproductive cough and fatigue that she has had for about a month. The patient attributes the lingering cough and fatigue to a previous cold-like illness. On exam, her right tympanic membrane is slightly erythematous but not bulging, and slight crackles are noted diffusely to bilateral lung fields. A Gram stain is obtained with no organisms noted. Chest X-ray is significant for bilateral diffuse patchy infiltrates. Which of the following is the most likely diagnosis? Aspiration pneumonia Legionella pneumophila pneumonia Mycoplasma pneumoniae pneumonia Streptococcus pneumoniae pneumonia

Mycoplasma pneumoniae pneumonia The patient in this vignette has atypical pneumonia, which is also known as "walking pneumonia." Mycoplasma pneumoniae is the most common cause of atypical pneumonia. Other causes of atypical pneumonias include Chlamydophilia pneumoniae, Legionella pneumophila, or viruses. As the term "atypical" implies, this type of pneumonia presents differently than the classic case of pneumonia. These patients commonly present with a low-grade fever, mild pulmonary symptoms including a nonproductive cough, and extrapulmonary symptoms (such as a sore throat, ear pain, myalgias, or malaise). These symptoms usually present over time and are insidious in nature. The individuals who are affected with Mycoplasma pneumoniae are typically either the young and healthy patients or the very old patients with comorbidities. Diagnosing atypical pneumonia is typically done via chest radiography which will show bilateral diffuse infiltrates. On a Gram stain or sputum culture, organisms are not visible. Treatment for atypical pneumonia is similar to that of community-acquired pneumonia, which is treated with azithromycin. Aspiration pneumonia (A) is more consistent with a Gram stain significant for anaerobes or any typical mouth flora, such as Staphylococcus, or if chest X-ray is consistent with infiltrates to the right lower lung. Legionella pneumophila pneumonia (B) presents with gastrointestinal symptoms, such as diarrhea, elevated liver enzymes, nausea, and abdominal pain, which the patient in the vignette does not have. Streptococcus pneumoniae pneumonia (D) will appear as gram-positive cocci in pairs on Gram stain. This is the most common cause of community-acquired pneumonia, which commonly presents with acute onset of a productive cough, shortness of breath, and pleuritic chest pain. Question: How many hours does a patient have to be hospitalized to be diagnosed with hospital-acquired pneumonia? Answer: 48 hours. A diagnosis of pneumonia prior to 48 hours of hospitalization would still be classified as community-acquired pneumonia.

A 65-year-old man with a history of hyperlipidemia who is a current everyday smoker presents to the emergency department with chest pain that started two hours ago. The pain began when he was shoveling snow. Physical exam is unremarkable. His initial ECG is shown above. His initial troponin level is 0.5 ng/mL, and his serum troponin two hours later is 1.2 ng/mL. Which of the following is the most likely diagnosis? ANon-ST elevation myocardial infarction BPrinzmetal angina CST elevation myocardial infarction DUnstable angina

NSTEMI This patient is presenting with a non-ST elevation myocardial infarction (NSTEMI). An NSTEMI is the result of atherosclerosis in coronary arteries or coronary artery vasospasm. Risk factors include hyperlipidemia, cigarette smoking, obesity, and diabetes mellitus. Patients will present with retrosternal chest pressure that radiates to the arm or jaw, is worse with exertion, and is not relieved by rest or nitrates. Other presenting symptoms include nausea, diaphoresis, and dizziness. Physical examination is typically unremarkable. When a patient has an NSTEMI, diagnosis is based on elevated troponin values on two occasions taken two hours apart. The electrocardiogram may show ST depressions or T wave inversions, or it may be normal. Initial therapy in the emergency department should include aspirin 325 mg, heparin drip, supplemental oxygen if oxygen saturation is below 90%, and sublingual nitroglycerin, unless the patient is hypotensive or there is evidence of right-sided infarction. Prinzmetal angina (B) is characterized by nonexertional chest pain that typically occurs early in the morning. It is caused by coronary artery vasospasms. Patients will have transiently elevated ST segments and are often treated with calcium channel blockers. ST elevation myocardial infarction (C) presents similarly to an NSTEMI. Patients will present with crushing, radiating chest pain that is aggravated by exertion and unrelieved with nitrates, however, ST elevations would be noted on the ECG. Immediate PCI should be performed for these patients. Unstable angina (D) also presents similarly to an NSTEMI, however, these patients do not have ECG changes or elevated troponin levels.

A 50-year-old woman presents with muscle weakness of the shoulder and pelvic girdles that has progressively worsened over the last several months. Physical exam reveals a three out of five muscle strength in the hip flexors and the deltoids. Laboratory studies demonstrate elevated aldolase and creatine kinase. Which of the following positive laboratory results would be expected? AAnti-double stranded DNA antibodies BAnti-Jo-1 antibodies CAnti-La antibodies DAnti-SCL-70 antibodies

Only anti-Jo-1 antibodies are a diagnostic marker for polymyositis and is associated with interstitial lung disease, arthritis, mechanic's hands, and Raynaud phenomenon. Anti-La antibodies (C) are seen in patients with lupus and Sjӧgren syndrome. Sjӧgren syndrome is an autoimmune disorder that affects the salivary and lacrimal glands leading to xerostomia and keratoconjunctivitis sicca. Lupus is an autoimmune disease that is characterized by multiorgan involvement including the skin, brain, kidneys, joints, and other organs. Anti-SCL-70 antibodies (D) are considered a specific marker for the diffuse type of systemic scleroderma but can also be seen in a subset of patients with lupus. Their presence in scleroderma carries a poorer prognosis. Diffuse systemic scleroderma is characterized by collagen deposition in the skin, heart, lungs, gastrointestinal tract, and kidneys. Anti-double stranded DNA antibodies (A) are antinuclear antibodies that target double-stranded DNA. They are associated with systemic lupus erythematosus, an autoimmune disorder characterized by chronic inflammation, positive ANA, and multiorgan involvement.

A 63-year-old woman presents to the emergency department for an initial presentation of a moderately severe temporal headache, scalp tenderness, and pain with chewing for six days. She has no change to her vision. She has a history of polymyalgia rheumatica with specific symptoms of neck and shoulder stiffness. Lab results indicate an elevated erythrocyte sedimentation rate and C-reactive protein. Imaging studies are pending. Which of the following is the first priority in caring for this patient, even before the diagnosis is confirmed? Arterial biopsy Hospital admission Immediate ophthalmology referral Oral corticosteroid treatment

Oral corticosteroid treatment Arterial biopsy (A) should be collected within two to three days after initiating corticosteroids. Biopsy specimens demonstrating segmented vasculitis confirm the diagnosis. For patients with severe symptoms, including vision loss, hospital admission (B) is necessary for formal evaluation and treatment with IV steroids. Immediate ophthalmology consult (C) should be obtained for any patients presenting with changes in vision. Question: What findings are most common on eye exam for a patient with vision impairment due to temporal arteritis? Answer: Cotton-wool spots, a sign of impaired perfusion of the retina, are precursors of ischemic occlusion and appear as fluffy white patches on the retina.

A 53-year-old woman presents to a primary care provider for right knee pain. She reports a gradual onset over the last four months and describes the pain as dull and achy. She reports joint stiffness in her right knee that typically occurs every morning, but it does not last more than 15 minutes. It is aggravated by activity and alleviated with rest and acetaminophen. She denies any acute or previous injury to her right knee. She also reports joint stiffness in her fingers. She has otherwise been healthy. Physical examination of the right knee reveals decreased flexion and extension, crepitus, and mild swelling. No erythema, warmth, or ecchymosis is present. Bony enlargements over the carpometacarpal joint of the thumb and the distal interphalangeal joints of the remaining digits are present bilaterally. Her right knee radiograph is shown above. What is the most likely diagnosis? Osteoarthritis Pseudogout Reactive arthritis Rheumatoid arthritis

Osteoarthritis Osteoarthritis is the most common arthropathy among adults. The joints most affected are the hips, knees, proximal interphalangeal (Bouchard nodes), distal interphalangeal (Heberden nodes), carpometacarpal of the thumb, and cervical and lumbar spine. The underlying pathophysiology is due to degeneration of the joint cartilage and bone hypertrophy at articular margins secondary to repetitive use over many years. Risk factors for osteoarthritis include age > 40 years old, obesity, and repetitive movements. Osteoarthritis does not typically result in elevated inflammatory markers (e.g., erythrocyte sedimentation rate, C-reactive protein), and synovial fluid is noninflammatory. An X-ray of the affected joint is the best method for confirming a diagnosis of osteoarthritis. X-ray findings will often show asymmetric joint space narrowing, subchondral sclerosis, and marginal osteophytes. Bony cysts may occasionally be present. Pseudogout (B) is a crystalopathy caused by calcium pyrophosphate dihydrate crystal deposition. It is often seen in patients > 60 years of age and mostly occurs in the larger joints (e.g., knees). Pseudogout has an acute onset and is characterized by erythema, warmth, and pain with movement. Reactive arthritis (C) is an autoimmune inflammatory condition typically occurring after a bacterial infection that affects the knees, ankles, or feet. It is characterized by the triad of conjunctivitis, urethritis, and arthritis. The patient in the above vignette has been otherwise healthy and does not have any ocular or genitourinary symptoms. Rheumatoid arthritis (D) is a chronic autoimmune inflammatory disease that has an insidious onset similar to osteoarthritis and is usually diagnosed in women in their third to fifth decade of life. Rheumatoid arthritis pain and stiffness generally lasts > 1 hour in the morning and mostly affects the wrist and metacarpophalangeal and joints of the hand while sparing the distal interphalangeal joints. Ulnar deviations, swan neck, deformities, and boutonnière deformities are frequently seen in affected phalangeal joints. The affected joints in rheumatoid arthritis are typically spongy and warm to the touch as opposed to the hard and cool joints affected by osteoarthritis. Question: What disease is avascular necrosis of the lunate bone also known as? Answer: Kienböck disease.

A 44-year-old woman presents with complaints of gradually worsening dyspnea on exertion, fatigue, and dry cough over the course of several months. Her vital signs are within normal limits. A chest radiograph is obtained that shows symmetric, bilateral hilar adenopathy. Which of the following would be the most likely finding on physical exam? Digital clubbing Erythema nodosum Peripheral lymphadenopathy Rales

Peripheral lymphadenopathy Sarcoidosis describes a granulomatous disease with multiple manifestations. The exact pathophysiology of sarcoidosis is unknown. The disease most frequently affects the lungs but can create extrapulmonary manifestations, including ocular, cutaneous, cardiovascular, or kidney involvement. Sarcoidosis most often presents in patients between 20 and 60 years of age. It is more common in African American patients and presents approximately 10 years earlier in those individuals. Patients usually present with cough, dyspnea, fatigue, and general malaise. They may have chest pain, exercise intolerance, and muscle weakness may be present. A careful history should include questions about new skin lesions around tattoos or scars, dry mucous membranes, parotid swelling, arthralgias, and muscle weakness to rule out extrapulmonary manifestations. Peripheral lymphadenopathy is present in approximately 40 percent of patients. Tuberculosis testing should be completed to rule out tuberculosis as a cause of the granulomatous pulmonary changes. A chest radiograph will show hilar adenopathy that is typically symmetric bilaterally. Other changes such as reticular or ground-glass opacities and nodular scarring may also be present. A high-resolution computed tomography scan may be necessary to further evaluate pulmonary findings. Digital clubbing (A) is associated with advanced disease and would not be present in a patient with newly diagnosed sarcoidosis. Erythema nodosum (B) is seen in patients with Lofgren syndrome. Lofgren syndrome is the combination of hilar adenopathy, polyarthralgias, fever, and erythema nodosum. It is less common than other presentations of sarcoidosis, although it is the most common nonspecific cutaneous finding associated with sarcoidosis. Rales (D) are associated with fluid in the smaller airway spaces that cause a crackling sound with breathing. These are not associated with sarcoidosis but rather with congestive heart failure and pulmonary edema. Question: What are the most common presenting symptoms of sarcoidosis in children? Answer: Uveitis, skin rash, and arthritis.

A 75-year-old woman presents to the emergency department after collapsing at work. She reports this is her third syncopal episode in the past month. The electrocardiogram above is obtained during the initial workup. What definitive clinical intervention is indicated for this patient? Amiodarone Flecainide Observation Permanent pacemaker

Permanent pacemaker This patient is presenting with sick sinus syndrome, which is a dysrhythmia characterized by periods of sinus arrest, persistent bradycardia, or chronotropic incompetence, indicating that the heart is not reacting appropriately to increased metabolic demands. Sinoatrial node disease and previous corrective cardiac surgery are risk factors for developing sick sinus syndrome. Patients may present with a variety of complaints related to sick sinus syndrome, including syncope, palpitations, fatigue, lightheadedness, dyspnea, and exercise intolerance. Diagnosis is confirmed by electrocardiogram, which shows bradycardia alternating with tachycardia as well as extended periods of sinus inactivity. If the initial ECG is not diagnostic, ambulatory ECG monitoring is required for diagnosis. A permanent pacemaker is indicated for symptomatic patients. If patients are hemodynamically unstable, dopamine, epinephrine, or atropine should be administered and transcutaneous pacing may be required. Asymptomatic patients typically do not require intervention but should be routinely evaluated for deterioration. Amiodarone (A) is a class III antidysrhythmic medication that is used to treat dysrhythmias in patients with coronary artery disease and left-side heart failure. It is also utilized in advanced cardiac life support management of ventricular tachycardia and ventricular fibrillation. Flecainide (B) is a class IC antidysrhythmic, and its mechanism of action depends on decreasing sodium influx in cardiac cells. Flecainide is used to treat and prevent paroxysmal atrial fibrillation, atrial flutter, and paroxysmal supraventricular tachycardia in patients with normal heart structure. Observation (C) is not indicated in the patient in the vignette above because she is symptomatic. Question: What class of antidysrhythmics are beta-blockers? Answer: Class II.

A 50-year-old woman is brought to the ED by her husband for sudden-onset nausea, abdominal pain, and blurred vision that began 30 minutes ago while she was sitting in the car at a drive-in movie theater. She reports that she saw halos around all the street lights on the way over to the ED. Physical examination reveals a right eye with ciliary injection, a hazy cornea, and a mid-dilated, fixed pupil that is nonreactive to light. An intraocular pressure of 65 mm Hg is obtained (normal 12-22 mm Hg). A bolus of acetazolamide 500 mg IV brings the intraocular pressure down to 22 mm Hg. The patient is discharged with acetazolamide 250 mg PO four times daily and instructed to follow up with an ophthalmologist the following morning. Which of the following medications is most appropriate in the management of the patient's condition? Brimonidine 0.2% ophthalmic solution Glycerin PO 2 g/kg Pilocarpine 4% ophthalmic solution Prednisolone acetate 1% eye drops

Pilocarpine 4% ophthalmic solution Acute angle-closure glaucoma is a medical emergency that may result in permanent vision loss within two to five days of symptom onset if untreated. Inuit and Asian populations are predisposed to developing acute angle-closure glaucoma. A shallow anterior chamber or enlarged lens increases the risk of acute angle-closure glaucoma. As the pupil is acutely dilated, the iris tissue is folded into the angle between the cornea and iris. This prevents the aqueous fluid from flowing out through the trabecular meshwork and results in a dangerous increase in intraocular pressure, typically > 50 mm Hg. The most common presentation occurs after acute pupillary dilation (e.g., sitting in a darkened movie theater, acute stress, administration of anticholinergic or sympathomimetic agents) and results in a sudden onset of severe pain, blurred vision, decreased visual acuity, and nausea. In certain patients, the pain is not well localized and may present as abdominal pain. Patients often report vision changes such as seeing halos around lights. Physical examination of the affected eye may reveal erythema, ciliary injection, a hazy or steamy cornea, and a mid-dilated fixed pupil that is nonreactive to light. Shining a light laterally into the affected eye may reveal a widened iris shadow over at least a quarter of the iris, which indicates a shallow anterior chamber. Initial treatment of acute angle-closure glaucoma involves rapidly decreasing the intraocular pressure. A bolus of acetazolamide 500 mg IV should be given immediately upon diagnosis. Osmotic diuretics (e.g., oral glycerin, IV urea, IV mannitol) may be used if patients are unresponsive to acetazolamide. Acetazolamide 250 mg PO four times daily, along with topical ophthalmic medications, should follow. One drop of pilocarpine 4% ophthalmic solution may be given four times daily. Pilocarpine works by causing constriction of the pupillary sphincter muscle which pulls the iris away from the angle and allows the trapped aqueous fluid to flow out through the trabecular meshwork. Clinicians should be aware that corticosteroid use can worsen acute angle-closure glaucoma by causing an increase in intraocular pressure secondary to an increase in aqueous fluid retention and a decrease in aqueous fluid outflow. Definitive treatment of acute angle-closure glaucoma is surgical and includes laser peripheral iridotomy or surgical peripheral iridectomy. Brimonidine 0.2% ophthalmic solution (A) is a selective alpha-2 agonist used in the treatment of chronic glaucoma and open-angle glaucoma. It is not used for treatment of acute angle-closure glaucoma. Glycerin PO 2 g/kg (B) is an osmotic diuretic that may be used if acetazolamide fails to reduce intraocular pressure. The patient in the above vignette responded to acetazolamide. Prednisolone acetate 1% eye drops (D) are a corticosteroid and are contraindicated in the treatment of glaucoma because they may cause an increase in intraocular pressure. Steroid drops may be used to reduce ocular inflammation in patients with iritis. Question: What ocular structure produces aqueous fluid? Answer: The ciliary body.

A 40-year-old woman presents to the office complaining of weakness in her muscles that for the past six months began in her legs, arms, and neck. She does not complain of any pain or changes in the skin. Her physical exam reveals bilateral weakness to proximal muscles in the hips, shoulders, and neck bilaterally without tenderness to palpation. Serologic testing is positive for anti-Jo-1 antibodies, and a muscle biopsy is ordered to confirm diagnosis. Which of the following is the most likely diagnosis? Dermatomyositis Polymyalgia rheumatica Polymyositis Systemic lupus erythematosus

Polymyositis Polymyositis is a systemic inflammatory disease of the muscles that is primarily characterized by muscle weakness. It is idiopathic but commonly associated with other autoimmune diseases (e.g., systemic lupus erythematosus), as well as certain viral infections (e.g., influenza, coxsackievirus), cancers, and connective tissue disorders. It affects all age groups but peaks at ages 40-60. It is more common in women than men and in those of African American descent than Caucasian. Patients will present classically with progressive bilateral proximal muscle weakness over the course of weeks to months, often without associated pain. The muscles involved typically include the muscles in the neck and upper and lower extremities. Weakness may fluctuate on different days. Lower extremity symptoms typically occur before upper extremity symptoms. Dermatomyositis (A) is similar to polymyositis but has additional dermatologic signs and symptoms, which the patient in the vignette above does not have evidence of. Patients with polymyalgia rheumatica (B) will not have muscle weakness but will complain of pain and stiffness in the proximal muscles. Patients with systemic lupus erythematosus (D) may have similar symptoms as those with polymyositis (e.g., fatigue, fever, arthralgias, myalgias), but it is not characterized by the presence of anti-Jo-1-antibodies. Question: What are some drugs that can cause myopathy? Answer: Hydroxychloroquine, colchicine, hydralazine, phenytoin, and angiotensin-converting enzyme inhibitors.

A 36-year-old diabetic health care worker is being treated for latent tuberculosis after a positive tuberculin skin test and a negative chest radiograph. The health care provider prescribes isoniazid daily for nine months. Which of the following medications should be prescribed in addition to this medication? Pyrazinamide Pyridoxine Rifampin Rifapentine

Pyridoxine There are several acceptable drug regimens available for the treatment of latent tuberculosis. Treatment regimens for active tuberculosis are different than those for latent tuberculosis. The acceptable regimens for treatment include rifampin daily for four months, isoniazid and rifampin daily for three months, isoniazid and rifapentine weekly for three months, or isoniazid daily for nine months. There has been no difference in efficacy of these regimens in treating latent infection, however, rates of efficacy may differ based on compliance. Isoniazid can cause peripheral neuropathy due to the interference with pyridoxine metabolism, and therefore, administration of pyridoxine while taking isoniazid is recommended in those who have a predisposition to developing neuropathy. Liver function tests must be monitored during the time that isoniazid is being prescribed, and patients should be monitored at least monthly. Pyrazinamide (A) is an antituberculosis agent that is used to treat active infections, not latent infections. Rifampin (C) and isoniazid are not typically combined to treat latent tuberculosis infection. Some studies have shown efficacy with a regimen of rifampin and isoniazid for three months, however, this regimen has not been studied well in large-scale studies. Rifapentine (D) is typically combined with isoniazid for a three-month regimen but is not used in combination with isoniazid for the nine-month regimen. Question: What is the most important side effect of isoniazid? Answer: Hepatitis.

A 25-year-old man presents to urgent care with general fatigue, slight fever, and pain, warmth, and swelling to his left knee for the past two weeks. He reports an occurrence of gastroenteritis nearly one month before these symptoms presented. On exam of the left knee, there is erythema, swelling, warmth, and extreme tenderness at locations of tendon insertion. Which of the following is the most likely diagnosis? Pseudogout Reactive arthritis Rheumatoid arthritis Septic arthritis

Reactive Arthritis Reactive arthritis is an autoimmune inflammatory asymmetric peripheral oligoarthritis that develops as a reaction to an infection in another part of the body, most often the gastrointestinal or genitourinary systems. The most common organisms causing infection in the gastrointestinal system include Salmonella, Shigella, Yersinia, or Campylobacter. Chlamydia trachomatis is the most prevalent in the genitourinary system. Reactive arthritis occurs more frequently in men ages 20-50. Symptoms may be self-limited and typically present as acute, asymmetric pain of the large weight-bearing joints (usually knee or ankle), erythema, and swelling that occurs several weeks after the initial infection. Systemic symptoms of fever, fatigue, and weight loss may also be present. Tx: NSAIDS. "Can't see, can't pee, can't climb a tree" Pseudogout (A) presents with pain, swelling, and erythema of the peripheral joints, is diagnosed with calcium pyrophosphate dihydrate crystals in the synovial fluid, and is treated with anti-inflammatories. Rheumatoid arthritis (C) begins with inflammation of multiple small joints in the hands and feet and progresses to larger joint involvement as the disease advances. Diagnosis is by the presence of rheumatoid factor on serology, and treatment includes disease-modifying antirheumatic drugs. Septic arthritis (D) is a true acute joint infection that presents with fever and chills along with pain, warmth, and swelling of the affected joint over hours. Synovial fluid analysis usually reveals high leukocytes (50,000-150,000 cells/mcL), consisting of predominantly neutrophils and most often a positive culture. Question: Which genetic marker has been linked to reactive arthritis? Answer: HLA-B27.

A 14-year-old girl presents to her primary care provider for an annual sports physical. Her past medical history is unremarkable. On physical exam, a left-sided rib hump is present on the Adam forward bend test. Examination of the chest wall reveals pectus excavatum. A midsystolic click is heard on cardiac auscultation. A Cobb angle of 15° is calculated from spine radiography. Her ECG rhythm strip is shown above. A transthoracic echocardiogram reveals a left ventricular ejection fraction of 70%, bowing of the mitral leaflets into the left atrium during systole, and trace to mild tricuspid regurgitation. Which of the following is the most appropriate next step in management? Reassurance and clearance for sports participation Referral to a cardiologist for further evaluation Repeat echocardiography annually for three years Surgical consult for valve repair

Reassurance and clearance for sports participation Mitral valve prolapse (MVP) is a valvular condition that occurs when the mitral leaflets bulge upward (prolapse) into the left atrium during systole. These "floppy" valves are most commonly a result of degenerative processes, but they can also be a normal anatomic variant most often seen in healthy, thin, young women. MVP can be associated with systemic collagen disorders (e.g., Marfan or Ehler-Danlos syndrome) and skeletal changes such as scoliosis and pectus excavatum. MVP is a common cause of mitral regurgitation. Isolated MVP and MVP with mild mitral regurgitation are commonly asymptomatic, though some patients may have mild symptoms such as nonspecific angina, dyspnea, and fatigue. Chronic MVP that results in moderate to severe mitral valve regurgitation can lead to an increased left atrial pressure, pulmonary edema, and progressive exertional dyspnea with fatigue. A midsystolic click on cardiac auscultation is heard when the mitral valve leaflets prolapse into the left atrium. A holosystolic murmur best heard at the apex is consistent with mitral regurgitation. A transthoracic echocardiogram is the best initial diagnostic tool to evaluate valvular conditions. CT or MRI angiography is indicated in patients with MVP who may have aortic root disease or aortic dilation. In patients with isolated MVP, such as the patient in this case, no treatment or follow-up is needed unless they become symptomatic. Referral to a cardiologist for further evaluation (B) is indicated in patients with MVP and audible mitral regurgitation. Only a midsystolic click consistent with MVP was heard on auscultation in this patient, and echocardiography showed no evidence of mitral regurgitation. Repeat echocardiography annually for three years (C) is not necessary for this patient since she only has trace to mild tricuspid regurgitation. The cardiologist would manage periodic echocardiography in patients with moderate to severe valvular dysfunction. A surgical consult for valve repair (D), while favored over valve replacement, is not indicated for isolated MVP Question: What is the Gorlin sign? Answer: The ability to touch the tongue to the nose, which is most commonly seen in patients with Ehler-Danlos syndrome.

A 67-year-old man with a history of alcohol use disorder is admitted to the hospital and subsequently diagnosed with cirrhosis. His chemistry panel and blood gas show the following results: Sodium: 142 mmol/L Potassium: 4.0 mmol/L Chloride: 110 mmol/L Albumin: 4.0 g/L pH: 7.55 PaCO2: 32 mm Hg PaO2: 90 mm Hg Bicarbonate: 23 mmol/L What is the most likely diagnosis? Anion gap metabolic acidosis Metabolic alkalosis Non-anion gap metabolic acidosis Respiratory alkalosis

Respiratory alkalosis An increased pH, a decreased PaCO2, and a decreased or normal bicarbonate are indicative of respiratory alkalosis. The most common cause of respiratory alkalosis is hyperventilation. Other pulmonary conditions (e.g., pneumonia, asthma, restrictive lung disease), sepsis, liver disease (e.g., cirrhosis), heart failure, and salicylate intoxication may also cause respiratory alkalosis. Anion gap metabolic acidosis (A) is characterized by a decreased pH, normal or increased PaCO2, decreased bicarbonate, and increased anion gap. Causes of anion gap metabolic acidosis can be remembered by the acronym MUDPILES (methanol, uremia, diabetic ketoacidosis, paraldehyde or propylene glycol, isoniazid or iron, lactic acidosis, ethanol, salicylates). Metabolic alkalosis (B) is characterized by an increased pH, normal or decreased PaCO2, and increased bicarbonate. Metabolic alkalosis can be caused by vomiting, thiazide or loop diuretics, mineralocorticoid excess, chloride restriction, severe potassium depletion, bicarbonate administration, or rapid correction of chronic hypercapnia syndrome. Non-anion gap metabolic acidosis (C) is characterized by a decreased pH, a normal or increased PaCO2, a decreased bicarbonate, and a normal anion gap. Causes of non-anion gap metabolic acidosis can be remembered by the acronym HARDUPS (hyperalimentation, acetazolamide, renal tubular acidosis, diarrhea, ureteropelvic shunt, posthypocapnia, spironolactone). Question: What is the formula for calculating the anion gap? Answer: Anion gap = Na+ + K+ − (Cl− + HCO3−).

A 68-year-old man presents to a primary care provider with complaints of fatigue, anorexia, insomnia, and muscle cramps. A complete laboratory workup is ordered and is significant for a hemoglobin of 9.1 g/dL, potassium of 5.4 mmol/L, phosphate of 1.8 mmol/L, and glomerular filtration rate of 29 mL/min/1.73 m2. Which of the following dietary modifications is most appropriate for this patient? AIncreased phosphate intake to > 1,000 mg/day BIncreased water intake to > 2 L/day CRestricted calcium intake to < 1,000 mg/day DRestricted protein intake to < 0.8 g/kg/day

Restricted protein intake to <0.8 g/kg/day for stage 4 Chronic Kidney Disease (CKD) Increased phosphate intake to > 1,000 mg/day (A), increased water intake to > 2 L/day (B), and restricted calcium intake to < 1,000 mg/day (C) are all incorrect. Patients with chronic kidney disease should have an adequate caloric intake, calcium and vitamin D supplementation, and limitation of fluids, sodium, potassium, and phosphorus. Question: What low-molecular-weight protein is a cysteine protease inhibitor that can be used as a serum biomarker for detecting acute kidney injury? Answer: Cystatin C.

A 65-year-old woman with a history of hypertension is four days post-total knee replacement. She reports a red, swollen leg that started last night. Today, she is experiencing shortness of breath and pleuritic chest pain. Her electrocardiogram is shown above. What dysrhythmia is consistent with the patient's underlying condition? ALeft anterior fascicular block BLeft bundle branch block CLeft posterior fascicular block DRight bundle branch block

Right bundle branch block This electrocardiogram shows a right bundle branch block. RBBB indicates slowed conduction through the right ventricle. RBBB may be caused by pulmonary embolism, chronic obstructive pulmonary disease, cor pulmonale, cardiomyopathy, or pulmonary hypertension, or it may be idiopathic. ECG findings associated with RBBB include a wide QRS complex (> 120 ms), QRS complex with sR′ or rsR′ in V1 and V2, and S waves that are slurred in V6 and I. There is no specific treatment for RBBB other than treating the underlying condition. Prognosis is good if the RBBB is not associated with acute coronary syndrome or a pulmonary embolism. A left anterior fascicular block (A) is characterized by left axis deviation; small Q waves with tall R waves in I and aVL; small R with deep S waves in II, III, and aVF; prolonged R wave peak time in aVL > 45 ms; and increased QRS voltage in the limb leads. A left bundle branch block (B) is characterized by QRS > 120 ms, dominant S wave in V1, broad monophasic waves in lateral leads, absence of Q waves in lateral leads, and prolonged R wave peak time > 60 ms in V5 and V6. A left posterior fascicular block (C) is characterized by right axis deviation; small R waves with deep S waves in I and aVL; small Q with tall R waves in II, III, and aVF; prolonged R wave peak time in aVF; no evidence of right ventricular hypertrophy; and no evidence of other etiologies for right axis deviation. Question: What are the parameters for right axis deviation? Answer: QRS axis greater than +90°.

A 45-year-old woman presents to the clinic. She appears quite somnolent and states she is in pain. She reports pain in her hands, forearms, and knees along with abdominal discomfort. She also describes sensitivity to cold in her hands, which she says makes her hands turn white and become painful. She reports her symptoms have been present on and off for the last few years but have begun to significantly worsen. Her medical history is significant for hypertension and interstitial lung disease with home oxygen use. She denies tobacco or illicit drug use. She takes lisinopril, metoprolol, hydrochlorothiazide, and mycophenolate. Physical exam reveals puffy, thickened skin over her fingers on both hands, tenderness over multiple joints, rales, labored respirations, general body atrophy, abdominal rigidity, and a systolic regurgitation murmur of grade 3 over the tricuspid area. Her vitals are a temperature of 99.5°F, HR of 95 bpm, RR of 20 breaths per minute, BP of 140/100 mm Hg, O2 saturation of 93%, and body mass index of 14 kg/m2. Which of the following is the most likely diagnosis? Acromegaly Polymyalgia rheumatica Polymyositis Scleroderma

Scleroderma Systemic sclerosis (scleroderma) is classically defined as a chronic, progressive, fibrotic disease that affects multiple systems. Broadly, it commonly involves vascular dysfunction (leading to ischemia) and fibrosis of skin and internal organs. The pathogenesis is complex and broad but generally involves fibroblasts as effector cells in the disease process. There are four subtypes of the disease: limited cutaneous systemic sclerosis, diffuse cutaneous systemic sclerosis, systemic sclerosis sine scleroderma, and systemic sclerosis with overlap syndrome. Limited cutaneous systemic sclerosis classically involves manifestations from the mnemonic CREST (calcinosis cutis, Raynaud phenomenon, esophageal dysmotility, sclerodactyly, and telangiectasia). History of a patient with suspected scleroderma includes severe fatigue, pruritus, edema, hyperpigmentation or depigmentation, dry skin, hair loss, joint or muscle pains, muscle weakness, stiffness and immobility, dysphagia, heartburn, constipation or diarrhea, dyspnea on exertion, chest pain, hypertension, pulmonary edema, and headaches, among others. Physical exam may show edematous digits of upper extremities, skin thickening, perioral skin tightening, digital ulcers in upper extremities, abnormal nail bed capillaries, telangiectasia, crepitus, or tendon friction rub, among many other possible exam findings. Acromegaly (A) is not likely in this patient since this condition typically involves progression over many years of generally increasing size of limbs, feet, hands, head, and other areas due to growth hormone hypersecretion. Polymyalgia rheumatica (B) is a rheumatic condition involving symmetric ache and stiffness of more proximal musculoskeletal regions, such as shoulders, hips, neck, and torso, especially in the morning. It does not have skin, lung, or cardiac manifestations. Polymyositis (C) can present with interstitial disease, cardiac involvement, myalgias, and other similar presentations to systemic sclerosis but usually does not involve skin thickening. It is characterized by muscle weakness but not pain. Question: What is the modestly helpful treatment for erectile dysfunction in men with systemic sclerosis? Answer: Long-acting PDE5 inhibitors.

A 26-year-old woman presents to the emergency department with a chief complaint of hematemesis. Emesis is described as bright red blood. She had two episodes prior to presenting to the hospital. She reports no known history of stomach ulcer. She denies abdominal pain. Vital signs indicate a blood pressure of 102/76 mm Hg and heart rate of 52 bpm. Her body mass index is 16.6 kg/m2. Physical examination reveals lanugo, calluses on the dorsum of the right hand, and a small left subconjunctival hemorrhage. An endoscopy is performed that demonstrates a longitudinal tear in her esophagus with minimal active bleeding. Which of the following would you expect to learn from her history, given her history and likely underlying medical condition? She has a history of alcohol use disorder She has a history of chronic NSAID use She has a history of rumination syndrome She has a history of self-induced vomiting

She has a history of self-induced vomiting Mallory-Weiss tears, longitudinal tears w/in esophagus or proximal stomach. Hx will include self-induced vomiting, alcohol use, or hiatal hernia She has a history of alcohol use disorder (A) is incorrect. Alcohol use is associated with Mallory-Weiss syndrome, but in this particular vignette, the patient does not have a history of alcohol use and has physical examination findings that are suspicious for self-induced vomiting. She has a history of chronic NSAID use (B) is incorrect. Chronic NSAID use would raise suspicion for a stomach ulcer or gastritis. The patient's endoscopy findings are consistent with a Mallory-Weiss tear. She has a history of rumination syndrome (C) is incorrect. Rumination syndrome is associated with the regurgitation of partially digested food. It is not included as a possible cause of Mallory-Weiss tear. The patient's physical examination findings are more consistent with a self-induced vomiting etiology. Question: What condition affecting pregnant women may result in a predisposition towards Mallory-Weiss tears? Answer: Hyperemesis gravidarum.

A 69-year-old woman with a 30 pack-year history and recent travel overseas presents to the emergency department with chest pain, shortness of breath, and cough. There is concern for pulmonary embolism, and she has a computed tomography scan of the chest completed that shows a 9 mm nodule in the left lower lobe. The rest of the scan is unremarkable, and she is diagnosed with bronchitis. Which of the following is the best management of this lesion? Order a follow-up CT scan in three months Order positron emission tomography scan Refer the patient for biopsy Send sputum for cytology

Since the most concerning differential is primary neoplasm, a careful history should be taken from patients with pulmonary nodules. Smoking history is a key concern and risk factor in the development of primary lung cancer. Patients with a smoking history and a large nodule (> 8 mm in diameter) should be followed by pulmonology and referred for biopsy of the lesion since there is a greater chance that the nodule is a primary malignancy. Nodules less than 8 mm in size can be followed with serial CT studies to ensure the lesion is not growing. Growing lesions are concerning and should be biopsied to rule out cancer. Newer recommendations from the US Preventive Services Task Force (USPSTF) include screening for lung cancer with a low-dose CT in patients ages 55 to 74 who are current or former smokers with a 30 pack-year history. Ordering a follow-up computed tomography (CT) scan in three months (A) might be appropriate for a patient with a nodule > 8 mm if the patient was younger and had no risk factors for lung cancer, however, in those with a smoking history and advanced age, the risk of malignancy is higher. Ordering a positron emission tomography (PET) scan (B) might be helpful in staging malignancy and also for determining whether a nodule in a patient with intermediate risk for malignancy may be cancerous. With patients at high risk for malignancy, the PET scan is less helpful in ruling out a primary lung cancer. Sending sputum for cytology (D) is not helpful in this case. Sputum cytology is not a sensitive test, particularly for malignancies that are not in the main airways. Question: What are some features that are used to characterize pulmonary nodules on CT scan? Answer: Size, attenuation, borders, calcification, fat content, and enhancement.

A 38-year-old woman has been diagnosed with papillary thyroid cancer. The tumor is 2.3 cm in diameter with metastasis to regional lymph nodes and no distant metastasis. Which of the following best describes the stage of this cancer? Stage I Stage II Stage III Stage IV

Stage 1 Papillary thyroid cancer is a thyroid follicular epithelial-derived cancer that is considered a differentiated type of cancer. The tumor, node, metastasis (TNM) system for staging is typically used for papillary thyroid cancer. If the age is less than 55 years, then patients with any tumor size and any node metastasis without distant metastasis are classified as stage I. If there is distant metastasis, this group of patients would be classified as stage II. As the age of the patient increases, then the size of the tumor, the number of nodes involved, and the presence or absence of distant metastasis are weighed more heavily in the staging process. Surgical intervention is the primary method of treatment. Stage II (B) would be accurate if this patient, who is younger than 55 years of age, had distant metastasis. The size of the tumor and the number of nodes involved would not change the staging. Stage III (C) would be diagnosed in a patient older than 55 years of age with a tumor that extends into the adjacent soft tissues without distant metastasis. Stage IV (D) is designated in a patient older than 55 years of age when the tumor invades prevertebral fascia or encaseates the carotid without metastasis or when there is any tumor or node involvement with distant metastasis. Question: Which types of papillary thyroid cancers have a poorer prognosis? Answer: Tall cell, insular, and hobnail variants.

A 23-year-old man with no significant past medical history presents to the clinic with an acute cough for 10 days. He states that he initially had rhinorrhea and a sore throat, but these symptoms have improved. Vital signs are normal and lungs are clear to auscultation. Which of the following is the best treatment? Amoxicillin Azithromycin Prednisone Supportive care

Supportive care Acute bronchitis is a lower respiratory tract infection that involves the large airways (bronchi) without evidence of pneumonia or underlying chronic obstructive pulmonary disease. Respiratory viruses cause the majority of cases of acute bronchitis. Cough lasting more than five days is the predominant clinical manifestation. The cough often lasts one to three weeks and may be productive or nonproductive. Nasal symptoms such as rhinorrhea and nasal congestion, sore throat, and headache may be present during the beginning of acute bronchitis. Acute bronchitis may also cause chest pain when coughing or hemoptysis. Patients with acute bronchitis typically do not have a fever or tachycardia. Auscultation of the lungs may reveal wheezing or rhonchi. Acute bronchitis is a clinical diagnosis. The differential diagnosis may include viral upper respiratory tract infections (common cold), influenza, or pneumonia. Viral upper respiratory tract infections and acute bronchitis initially present similarly with nasal congestion, cough, sore throat, and headache, however, the cough persists and becomes the predominant symptom in patients with acute bronchitis. Any of the following clinical manifestations raise suspicion for pneumonia: fever, tachycardia, respiratory distress, rales, and decreased breath sounds. Amoxicillin (A) is a beta-lactam antibiotic frequently used to cover the typical organisms causing community-acquired pneumonia, such as Streptococcus pneumoniae. However, antibiotics are not indicated in the treatment of acute bronchitis because respiratory viruses cause the majority of cases. Azithromycin (B) is a macrolide antibiotic that may be used to cover atypical organisms causing community-acquired pneumonia, such as Mycoplasma pneumoniae. Antibiotics are not indicated in the treatment of acute bronchitis. Prednisone (C) is a corticosteroid. Corticosteroids are often prescribed for symptom improvement in patients with acute bronchitis, however, studies have not shown clinical benefit. Therefore, corticosteroids are not recommended in the treatment of acute bronchitis. Question: What are common causes of a cough lasting more than four weeks? Answer: Asthma, chronic obstructive pulmonary disease, angiotensin-converting enzyme inhibitor use, gastroesophageal reflux disease, and postnasal drip.

A 35-year-old woman with central obesity, a large hump in her supraclavicular area, striae of her skin, and easy bruising presents to the health clinic for worsening proximal muscle weakness. She is noted to be hypertensive, has gained weight, and has darkening of the skin on the back of her neck with a velvety texture. She is also noted to have hirsutism and acne on her face. Laboratory studies show a normal glucose level and hypokalemia. High-dose dexamethasone suppression test shows ACTH suppression, and an MRI shows a pituitary adenoma. What is the best clinical intervention for this patient, based on the most likely diagnosis? Adrenalectomy Ketoconazole Steroid taper Transsphenoidal surgery

Transsphenoidal surgery Cushing disease is caused by excess ACTH produced most commonly by a benign pituitary adenoma. Increased ACTH causes excess cortisol and glucocorticoid production, resulting in the clinical manifestations of Cushing disease. Symptoms include truncal obesity, moon facies, buffalo hump, wasting of extremity muscles, proximal muscle weakness, and striae of the abdomen. Patients can also develop hypertension, weight gain, hirsutism, acanthosis nigricans, and acne. Diagnosis of Cushing disease is made by high-dose dexamethasone suppression test, which will show ACTH suppression, ruling out other causes of increased ACTH or cortisol production. MRI of the brain can show a pituitary mass, which is suggestive of Cushing disease. Treatment is transsphenoidal surgery to remove the pituitary adenoma. Question: What is the most common cause of Cushing syndrome? Answer: Long-term high-dose corticosteroid therapy.

A 30-year-old woman with a recent diagnosis of HIV and history of smoking presents to your office for her annual exam. She wants to make sure she is up to date with the screenings recommended with her new diagnosis. Which of the following preventive screening tests should be done? Fecal occult blood test Low-dose computed tomography of the chest Mammogram Tuberculin skin test

Tuberculin skin test Most deaths in patients with HIV who are on antiretroviral therapy are now attributed to other causes and not to AIDS. HIV infection increases the risk of a number of non-AIDS-related diseases, including liver disease, renal disease, cardiovascular disease, and certain malignancies. With that in mind, preventive health care is an important component of treating the patient with HIV infection and differs somewhat from preventive care in a patient without HIV infection. Routine sex- and age-specific preventive screenings should be done in patients with HIV infection, but there are additional screenings and differences in the frequency of screenings that need to be considered for these patients. Screening for cardiovascular risk is recommended with more frequency in patients with HIV. Fasting lipid profile and glucose or hemoglobin A1C should be monitored at baseline and then every 6-12 months. Bone density testing is recommended in both men and women with HIV, with women being screened after menopause and men being screened after age 50 years. Cervical cancer screening in women with HIV is more frequent than in those without. There are some differences in vaccination recommendations for patients with HIV, especially those with a CD4 count less than 200 cells/microL who cannot receive live vaccinations. Tuberculosis screening after HIV diagnosis should be done in all patients. Those without a history of tuberculosis or positive screening are important to screen because tuberculosis can occur with any CD4 level. Fecal occult blood testing (A) or colonoscopy for colon cancer screening is recommended for patients with HIV starting at age 50 years, or younger if there is a family history that merits earlier screening. Low-dose computed tomography of the chest (B) is used to screen patients for lung cancer starting at age 55 years when there is a 30 pack-year history of smoking and current smoking or a history of quitting in the last 15 years. Mammogram (C) is the diagnostic tool of choice to screen for breast cancer in women starting at age 40-50 years. HIV infection does not change the initiation and frequency of recommendations for colon cancer screening, lung cancer screening, or breast cancer screening. Question: True or false: prophylaxis against opportunistic infections is recommended for all patients with HIV. Answer: False.


Set pelajaran terkait

Chapter 6 Accounting 212 Smartbook Stuff

View Set

Biology Chapter 11: Cell Communication

View Set

Foundations of Project Management: W2

View Set

AP CSP Unit 6 College Board Questions

View Set

Chapter 4: ABO and Hh Blood Group Systems

View Set